Download UPSC CMS Paper I 2020 Question Paper with Answer Key

Download UPSC Combined Medical Services (CMS) CMS Paper I 2020 Question Paper with Answer Key

Combined Medical Services Examination-2020
Paper-I
1.
A chronic alcoholic develops a paroxysm of palpitations after alcohol binge. Which of the
following Arrhythmia is most likely?
(a) Ventricular fibrillation
(b) Ventricular premature complex
(c) Atrial flutter
(d) Atrial fibrillation
2.
An elderly-man with history of Diabetes mellitus and Coronary Artery Disease comes for
follow-up, with complaints of muscle pains. Which one of the following drugs could be the
most likely cause?
(a) Aspirin
(b) Glimepiride
(c) Enalapril
(d) Atorvastatin
3.
A 40-year old lady comes to outdoor clinic with complaints of sudden onset chest pain. The
chest X-ray shows bilateral Pneumothorax. Examination reveals abnormalities of body
habitus-including long arms, legs and finger (arachnodactyly), scoliosis, high arched palate,
joint hypermobility, and a pansystolic murmur at cardiac apex. Which one of the following
diseases is most likely?
(a) Takayasu?s arteritis
(b) Raynaud?s syndrome
(c) Marfan?s syndrome
(d) Rheumatic heart disease-Mitral regurgitation
4.
All of the following are indications for treadmill testing/exercise-testing EXCEPT:
(a) To confirm the diagnosis of angina
(b) To evaluate stable angina
(c) To assess outcome after coronary revascularization
(d) To evaluate the treatment efficacy of antianginal drugs
5.
Kussmaul?s sign is present in all of the following conditions, EXCEPT:

(a) Massive Pulmonary Embolism
(b) Restrictive Cardiomyopathy
(c) Hypertrophic Cardiomyopathy
(d) Right Ventricular Infarction

6.
Which one of the following is NOT a common cause of atrial fibrillation?

(a) Mitral regurgitation
(b) Hypothyroidism
(c) Hypertension
(d) Acute myocardial infarction

FirstRanker.com - FirstRanker's Choice
Combined Medical Services Examination-2020
Paper-I
1.
A chronic alcoholic develops a paroxysm of palpitations after alcohol binge. Which of the
following Arrhythmia is most likely?
(a) Ventricular fibrillation
(b) Ventricular premature complex
(c) Atrial flutter
(d) Atrial fibrillation
2.
An elderly-man with history of Diabetes mellitus and Coronary Artery Disease comes for
follow-up, with complaints of muscle pains. Which one of the following drugs could be the
most likely cause?
(a) Aspirin
(b) Glimepiride
(c) Enalapril
(d) Atorvastatin
3.
A 40-year old lady comes to outdoor clinic with complaints of sudden onset chest pain. The
chest X-ray shows bilateral Pneumothorax. Examination reveals abnormalities of body
habitus-including long arms, legs and finger (arachnodactyly), scoliosis, high arched palate,
joint hypermobility, and a pansystolic murmur at cardiac apex. Which one of the following
diseases is most likely?
(a) Takayasu?s arteritis
(b) Raynaud?s syndrome
(c) Marfan?s syndrome
(d) Rheumatic heart disease-Mitral regurgitation
4.
All of the following are indications for treadmill testing/exercise-testing EXCEPT:
(a) To confirm the diagnosis of angina
(b) To evaluate stable angina
(c) To assess outcome after coronary revascularization
(d) To evaluate the treatment efficacy of antianginal drugs
5.
Kussmaul?s sign is present in all of the following conditions, EXCEPT:

(a) Massive Pulmonary Embolism
(b) Restrictive Cardiomyopathy
(c) Hypertrophic Cardiomyopathy
(d) Right Ventricular Infarction

6.
Which one of the following is NOT a common cause of atrial fibrillation?

(a) Mitral regurgitation
(b) Hypothyroidism
(c) Hypertension
(d) Acute myocardial infarction

7.
In an ECG recording, P wave is produced by:

(a) Atrial depolarisation
(b) Atrial repolarisation
(c) Ventricular depolarisation
(d) Ventricular repolarisation
8.
Which one of the following modalities is NOT used in the treatment of ventricular
tachycardia?

(a) DC Cardioversion
(b) Injection Lignocaine
(c) Injection Amiodarone
(d) Injection Adenosine
9.
Which one of the following drugs is a Direct Renin Inhibitor?

(a) Benedipine
(b) Azilsartan
(c) Aliskiren
(d) Lisinopril
10.
All of the following are causes of systolic hypertension with wide pulse pressure EXCEPT:

(a) Aortic regurgitation
(b) Thyrotoxicosis
(c) Patent ductus arteriosus
(d) Aortic stenosis
11.
Signet-ring sign on CT-chest is suggestive of:
(a) Bronchiectasis
(b) Active alveolitis
(c) Aspergilloma
(d) Sarcoidosis
12.
Oral corticosteroids are best introduced in the treatment of chronic bronchial asthma when it
is:
(a) Mild persistent
(b) Moderate persistent
(c) Severe persistent
(d) Very severe persistent
13.
An 18-year old male has insulin dependent diabetes, with malabsorption syndrome and
bilateral upper lobe bronchiectasis. A diagnostic work-up will include all EXCEPT:
(a) 24 hour fecal fat-estimation
(b) CT-chest
(c) Lung biopsy
(d) DNA sequencing study
FirstRanker.com - FirstRanker's Choice
Combined Medical Services Examination-2020
Paper-I
1.
A chronic alcoholic develops a paroxysm of palpitations after alcohol binge. Which of the
following Arrhythmia is most likely?
(a) Ventricular fibrillation
(b) Ventricular premature complex
(c) Atrial flutter
(d) Atrial fibrillation
2.
An elderly-man with history of Diabetes mellitus and Coronary Artery Disease comes for
follow-up, with complaints of muscle pains. Which one of the following drugs could be the
most likely cause?
(a) Aspirin
(b) Glimepiride
(c) Enalapril
(d) Atorvastatin
3.
A 40-year old lady comes to outdoor clinic with complaints of sudden onset chest pain. The
chest X-ray shows bilateral Pneumothorax. Examination reveals abnormalities of body
habitus-including long arms, legs and finger (arachnodactyly), scoliosis, high arched palate,
joint hypermobility, and a pansystolic murmur at cardiac apex. Which one of the following
diseases is most likely?
(a) Takayasu?s arteritis
(b) Raynaud?s syndrome
(c) Marfan?s syndrome
(d) Rheumatic heart disease-Mitral regurgitation
4.
All of the following are indications for treadmill testing/exercise-testing EXCEPT:
(a) To confirm the diagnosis of angina
(b) To evaluate stable angina
(c) To assess outcome after coronary revascularization
(d) To evaluate the treatment efficacy of antianginal drugs
5.
Kussmaul?s sign is present in all of the following conditions, EXCEPT:

(a) Massive Pulmonary Embolism
(b) Restrictive Cardiomyopathy
(c) Hypertrophic Cardiomyopathy
(d) Right Ventricular Infarction

6.
Which one of the following is NOT a common cause of atrial fibrillation?

(a) Mitral regurgitation
(b) Hypothyroidism
(c) Hypertension
(d) Acute myocardial infarction

7.
In an ECG recording, P wave is produced by:

(a) Atrial depolarisation
(b) Atrial repolarisation
(c) Ventricular depolarisation
(d) Ventricular repolarisation
8.
Which one of the following modalities is NOT used in the treatment of ventricular
tachycardia?

(a) DC Cardioversion
(b) Injection Lignocaine
(c) Injection Amiodarone
(d) Injection Adenosine
9.
Which one of the following drugs is a Direct Renin Inhibitor?

(a) Benedipine
(b) Azilsartan
(c) Aliskiren
(d) Lisinopril
10.
All of the following are causes of systolic hypertension with wide pulse pressure EXCEPT:

(a) Aortic regurgitation
(b) Thyrotoxicosis
(c) Patent ductus arteriosus
(d) Aortic stenosis
11.
Signet-ring sign on CT-chest is suggestive of:
(a) Bronchiectasis
(b) Active alveolitis
(c) Aspergilloma
(d) Sarcoidosis
12.
Oral corticosteroids are best introduced in the treatment of chronic bronchial asthma when it
is:
(a) Mild persistent
(b) Moderate persistent
(c) Severe persistent
(d) Very severe persistent
13.
An 18-year old male has insulin dependent diabetes, with malabsorption syndrome and
bilateral upper lobe bronchiectasis. A diagnostic work-up will include all EXCEPT:
(a) 24 hour fecal fat-estimation
(b) CT-chest
(c) Lung biopsy
(d) DNA sequencing study
14.
Which one of the following antigens is commonly associated with causation of Farmer?s
lung?
(a) Penicillium
(b) Actinomycetes
(c) Aspergillus
(d) Candida
15.
The Gene X-pert test used for MTB detection has the additional advantage of detection of
which of the following?

(a) INH resistance
(b) Rifampicin resistance
(c) Multi drug resistance
(d) Ethambutol resistance
16.
The most common cause of chronic type II respiratory failure is:
(a) Severe pneumonia
(b) Severe COPD
(c) Severe bronchial asthma
(d) Severe pulmonary thromboembolism
17.
The most frequent symptom in respiratory diseases is:
(a) Breathlessness
(b) Chest pain
(c) Hemoptysis
(d) Cough
18.
A 26-year old young lady attends medical emergency and is labelled as acute severe asthma
by the emergency physician. Which one of the following is unlikely to be a part of the
prescription?

(a) Albuterol
(b) Anti-leukotrienes
(c) Aminophylline
(d) Magnesium sulfate
19.
Consider the following statements with regard to respiratory examination:

1. Change in note, when patient phonates ?EEE? (Egophony) is characteristic of
interstitial fibrosis
2. Whispered pectoriloquy is characteristic of lung consolidation
3. Monophonic wheeze is characteristic of asthma
4. Hyper-resonant note on percussion is characteristic of pnemothorax

Which of the above statements are correct?
(a) 1 and 3
(b) 2 and 4 only
(c) 1, 2 and 4
(d) 2, 3 and 4
FirstRanker.com - FirstRanker's Choice
Combined Medical Services Examination-2020
Paper-I
1.
A chronic alcoholic develops a paroxysm of palpitations after alcohol binge. Which of the
following Arrhythmia is most likely?
(a) Ventricular fibrillation
(b) Ventricular premature complex
(c) Atrial flutter
(d) Atrial fibrillation
2.
An elderly-man with history of Diabetes mellitus and Coronary Artery Disease comes for
follow-up, with complaints of muscle pains. Which one of the following drugs could be the
most likely cause?
(a) Aspirin
(b) Glimepiride
(c) Enalapril
(d) Atorvastatin
3.
A 40-year old lady comes to outdoor clinic with complaints of sudden onset chest pain. The
chest X-ray shows bilateral Pneumothorax. Examination reveals abnormalities of body
habitus-including long arms, legs and finger (arachnodactyly), scoliosis, high arched palate,
joint hypermobility, and a pansystolic murmur at cardiac apex. Which one of the following
diseases is most likely?
(a) Takayasu?s arteritis
(b) Raynaud?s syndrome
(c) Marfan?s syndrome
(d) Rheumatic heart disease-Mitral regurgitation
4.
All of the following are indications for treadmill testing/exercise-testing EXCEPT:
(a) To confirm the diagnosis of angina
(b) To evaluate stable angina
(c) To assess outcome after coronary revascularization
(d) To evaluate the treatment efficacy of antianginal drugs
5.
Kussmaul?s sign is present in all of the following conditions, EXCEPT:

(a) Massive Pulmonary Embolism
(b) Restrictive Cardiomyopathy
(c) Hypertrophic Cardiomyopathy
(d) Right Ventricular Infarction

6.
Which one of the following is NOT a common cause of atrial fibrillation?

(a) Mitral regurgitation
(b) Hypothyroidism
(c) Hypertension
(d) Acute myocardial infarction

7.
In an ECG recording, P wave is produced by:

(a) Atrial depolarisation
(b) Atrial repolarisation
(c) Ventricular depolarisation
(d) Ventricular repolarisation
8.
Which one of the following modalities is NOT used in the treatment of ventricular
tachycardia?

(a) DC Cardioversion
(b) Injection Lignocaine
(c) Injection Amiodarone
(d) Injection Adenosine
9.
Which one of the following drugs is a Direct Renin Inhibitor?

(a) Benedipine
(b) Azilsartan
(c) Aliskiren
(d) Lisinopril
10.
All of the following are causes of systolic hypertension with wide pulse pressure EXCEPT:

(a) Aortic regurgitation
(b) Thyrotoxicosis
(c) Patent ductus arteriosus
(d) Aortic stenosis
11.
Signet-ring sign on CT-chest is suggestive of:
(a) Bronchiectasis
(b) Active alveolitis
(c) Aspergilloma
(d) Sarcoidosis
12.
Oral corticosteroids are best introduced in the treatment of chronic bronchial asthma when it
is:
(a) Mild persistent
(b) Moderate persistent
(c) Severe persistent
(d) Very severe persistent
13.
An 18-year old male has insulin dependent diabetes, with malabsorption syndrome and
bilateral upper lobe bronchiectasis. A diagnostic work-up will include all EXCEPT:
(a) 24 hour fecal fat-estimation
(b) CT-chest
(c) Lung biopsy
(d) DNA sequencing study
14.
Which one of the following antigens is commonly associated with causation of Farmer?s
lung?
(a) Penicillium
(b) Actinomycetes
(c) Aspergillus
(d) Candida
15.
The Gene X-pert test used for MTB detection has the additional advantage of detection of
which of the following?

(a) INH resistance
(b) Rifampicin resistance
(c) Multi drug resistance
(d) Ethambutol resistance
16.
The most common cause of chronic type II respiratory failure is:
(a) Severe pneumonia
(b) Severe COPD
(c) Severe bronchial asthma
(d) Severe pulmonary thromboembolism
17.
The most frequent symptom in respiratory diseases is:
(a) Breathlessness
(b) Chest pain
(c) Hemoptysis
(d) Cough
18.
A 26-year old young lady attends medical emergency and is labelled as acute severe asthma
by the emergency physician. Which one of the following is unlikely to be a part of the
prescription?

(a) Albuterol
(b) Anti-leukotrienes
(c) Aminophylline
(d) Magnesium sulfate
19.
Consider the following statements with regard to respiratory examination:

1. Change in note, when patient phonates ?EEE? (Egophony) is characteristic of
interstitial fibrosis
2. Whispered pectoriloquy is characteristic of lung consolidation
3. Monophonic wheeze is characteristic of asthma
4. Hyper-resonant note on percussion is characteristic of pnemothorax

Which of the above statements are correct?
(a) 1 and 3
(b) 2 and 4 only
(c) 1, 2 and 4
(d) 2, 3 and 4
20.
All of the following are the causes of exudative Pleural Effusion, EXCEPT:
(a) SVC obstruction
(b) Fungal infection
(c) SLE
(d) Meig syndrome
21.
Which one of the following is NOT true about Ghrelin?
(a) It stimulates appetite
(b) It decreases gastric emptying
(c) It increases acid secretion
(d) Fasting increases its secretion
22.
Fecal elastase test is used for diagnosing which one of the following conditions?
(a) Lactose intolerance
(b) Mucosal inflammation
(c) Bile acid secretory defects
(d) Pancreatic dysfunction
23.
The recent classification system for listing a patient as a candidate for liver transplantation is:
(a) Child Pugh score
(b) APACHE score
(c) MELD score
(d) Metavir score
24.
The blood supply of liver consists of:
(a) 50 % hepatic artery and 50 % portal vein
(b) 80 % portal vein and 20 % hepatic artery
(c) 80 % hepatic artery and 20 % portal vein
(d) 70 % hepatic artery and 30 % portal vein
25.
All of the following conditions are associated with Glomerulonephritis with low complement
level, EXCEPT:
(a) Subacute bacterial endocarditis
(b) IgA nephropathy
(c) Systemic lupus erythematosus
(d) Cryoglobulinaemia













FirstRanker.com - FirstRanker's Choice
Combined Medical Services Examination-2020
Paper-I
1.
A chronic alcoholic develops a paroxysm of palpitations after alcohol binge. Which of the
following Arrhythmia is most likely?
(a) Ventricular fibrillation
(b) Ventricular premature complex
(c) Atrial flutter
(d) Atrial fibrillation
2.
An elderly-man with history of Diabetes mellitus and Coronary Artery Disease comes for
follow-up, with complaints of muscle pains. Which one of the following drugs could be the
most likely cause?
(a) Aspirin
(b) Glimepiride
(c) Enalapril
(d) Atorvastatin
3.
A 40-year old lady comes to outdoor clinic with complaints of sudden onset chest pain. The
chest X-ray shows bilateral Pneumothorax. Examination reveals abnormalities of body
habitus-including long arms, legs and finger (arachnodactyly), scoliosis, high arched palate,
joint hypermobility, and a pansystolic murmur at cardiac apex. Which one of the following
diseases is most likely?
(a) Takayasu?s arteritis
(b) Raynaud?s syndrome
(c) Marfan?s syndrome
(d) Rheumatic heart disease-Mitral regurgitation
4.
All of the following are indications for treadmill testing/exercise-testing EXCEPT:
(a) To confirm the diagnosis of angina
(b) To evaluate stable angina
(c) To assess outcome after coronary revascularization
(d) To evaluate the treatment efficacy of antianginal drugs
5.
Kussmaul?s sign is present in all of the following conditions, EXCEPT:

(a) Massive Pulmonary Embolism
(b) Restrictive Cardiomyopathy
(c) Hypertrophic Cardiomyopathy
(d) Right Ventricular Infarction

6.
Which one of the following is NOT a common cause of atrial fibrillation?

(a) Mitral regurgitation
(b) Hypothyroidism
(c) Hypertension
(d) Acute myocardial infarction

7.
In an ECG recording, P wave is produced by:

(a) Atrial depolarisation
(b) Atrial repolarisation
(c) Ventricular depolarisation
(d) Ventricular repolarisation
8.
Which one of the following modalities is NOT used in the treatment of ventricular
tachycardia?

(a) DC Cardioversion
(b) Injection Lignocaine
(c) Injection Amiodarone
(d) Injection Adenosine
9.
Which one of the following drugs is a Direct Renin Inhibitor?

(a) Benedipine
(b) Azilsartan
(c) Aliskiren
(d) Lisinopril
10.
All of the following are causes of systolic hypertension with wide pulse pressure EXCEPT:

(a) Aortic regurgitation
(b) Thyrotoxicosis
(c) Patent ductus arteriosus
(d) Aortic stenosis
11.
Signet-ring sign on CT-chest is suggestive of:
(a) Bronchiectasis
(b) Active alveolitis
(c) Aspergilloma
(d) Sarcoidosis
12.
Oral corticosteroids are best introduced in the treatment of chronic bronchial asthma when it
is:
(a) Mild persistent
(b) Moderate persistent
(c) Severe persistent
(d) Very severe persistent
13.
An 18-year old male has insulin dependent diabetes, with malabsorption syndrome and
bilateral upper lobe bronchiectasis. A diagnostic work-up will include all EXCEPT:
(a) 24 hour fecal fat-estimation
(b) CT-chest
(c) Lung biopsy
(d) DNA sequencing study
14.
Which one of the following antigens is commonly associated with causation of Farmer?s
lung?
(a) Penicillium
(b) Actinomycetes
(c) Aspergillus
(d) Candida
15.
The Gene X-pert test used for MTB detection has the additional advantage of detection of
which of the following?

(a) INH resistance
(b) Rifampicin resistance
(c) Multi drug resistance
(d) Ethambutol resistance
16.
The most common cause of chronic type II respiratory failure is:
(a) Severe pneumonia
(b) Severe COPD
(c) Severe bronchial asthma
(d) Severe pulmonary thromboembolism
17.
The most frequent symptom in respiratory diseases is:
(a) Breathlessness
(b) Chest pain
(c) Hemoptysis
(d) Cough
18.
A 26-year old young lady attends medical emergency and is labelled as acute severe asthma
by the emergency physician. Which one of the following is unlikely to be a part of the
prescription?

(a) Albuterol
(b) Anti-leukotrienes
(c) Aminophylline
(d) Magnesium sulfate
19.
Consider the following statements with regard to respiratory examination:

1. Change in note, when patient phonates ?EEE? (Egophony) is characteristic of
interstitial fibrosis
2. Whispered pectoriloquy is characteristic of lung consolidation
3. Monophonic wheeze is characteristic of asthma
4. Hyper-resonant note on percussion is characteristic of pnemothorax

Which of the above statements are correct?
(a) 1 and 3
(b) 2 and 4 only
(c) 1, 2 and 4
(d) 2, 3 and 4
20.
All of the following are the causes of exudative Pleural Effusion, EXCEPT:
(a) SVC obstruction
(b) Fungal infection
(c) SLE
(d) Meig syndrome
21.
Which one of the following is NOT true about Ghrelin?
(a) It stimulates appetite
(b) It decreases gastric emptying
(c) It increases acid secretion
(d) Fasting increases its secretion
22.
Fecal elastase test is used for diagnosing which one of the following conditions?
(a) Lactose intolerance
(b) Mucosal inflammation
(c) Bile acid secretory defects
(d) Pancreatic dysfunction
23.
The recent classification system for listing a patient as a candidate for liver transplantation is:
(a) Child Pugh score
(b) APACHE score
(c) MELD score
(d) Metavir score
24.
The blood supply of liver consists of:
(a) 50 % hepatic artery and 50 % portal vein
(b) 80 % portal vein and 20 % hepatic artery
(c) 80 % hepatic artery and 20 % portal vein
(d) 70 % hepatic artery and 30 % portal vein
25.
All of the following conditions are associated with Glomerulonephritis with low complement
level, EXCEPT:
(a) Subacute bacterial endocarditis
(b) IgA nephropathy
(c) Systemic lupus erythematosus
(d) Cryoglobulinaemia













26.
Consider the following statements with regard to oral aphthous ulcers:

1. They are superficial and painful
2. They may occur in women just prior to menstruation
3. Oral glucocorticoids may be needed for the treatment of severe, recurrent cases
4. They are pre-malignant and progress over few years to squamous cell carcinoma of
the oral cavity

Which of the above statements are correct?

(a) 1 and 4 only
(b) 2 and 3 only
(c) 1, 2 and 3 only
(d) 1, 2, 3 and 4
27.
Which one of the following statements about Barrett?s Oesophagus is NOT correct?

(a) It is a pre-malignant condition
(b) Normal squamous cells lining the lower oesophagus are replaced by columnar
cells
(c) Normal columnar cells lining the lower oesophagus are replaced by squamous
cells
(d) It is an adaptive response to chronic gastro-oesophageal reflux

28.
The most common cause of acute hepatitis outbreaks in India is:

(a) Hepatitis E
(b) Hepatitis C
(c) Hepatitis B
(d) Hepatitis A

29.
Spontaneous oesophageal perforation after a bout of forceful vomiting or retching is
characteristically seen in which one of the following conditions?

(a) Menetrier?s disease
(b) Boerhaave?s syndrome
(c) Achalasia of the oesophagus
(d) Barrett?s oesophagus
30.
Eradication of helicobacter pylori infection may prove beneficial in the following extra-
gastric disorders EXCEPT:

(a) Unexplained vitamin B12 deficiency
(b) Idiopathic thrombocytopenic purpura
(c) Iron deficiency anaemia without gastro-intestinal bleeding
(d) Acute glomerulonephritis

FirstRanker.com - FirstRanker's Choice
Combined Medical Services Examination-2020
Paper-I
1.
A chronic alcoholic develops a paroxysm of palpitations after alcohol binge. Which of the
following Arrhythmia is most likely?
(a) Ventricular fibrillation
(b) Ventricular premature complex
(c) Atrial flutter
(d) Atrial fibrillation
2.
An elderly-man with history of Diabetes mellitus and Coronary Artery Disease comes for
follow-up, with complaints of muscle pains. Which one of the following drugs could be the
most likely cause?
(a) Aspirin
(b) Glimepiride
(c) Enalapril
(d) Atorvastatin
3.
A 40-year old lady comes to outdoor clinic with complaints of sudden onset chest pain. The
chest X-ray shows bilateral Pneumothorax. Examination reveals abnormalities of body
habitus-including long arms, legs and finger (arachnodactyly), scoliosis, high arched palate,
joint hypermobility, and a pansystolic murmur at cardiac apex. Which one of the following
diseases is most likely?
(a) Takayasu?s arteritis
(b) Raynaud?s syndrome
(c) Marfan?s syndrome
(d) Rheumatic heart disease-Mitral regurgitation
4.
All of the following are indications for treadmill testing/exercise-testing EXCEPT:
(a) To confirm the diagnosis of angina
(b) To evaluate stable angina
(c) To assess outcome after coronary revascularization
(d) To evaluate the treatment efficacy of antianginal drugs
5.
Kussmaul?s sign is present in all of the following conditions, EXCEPT:

(a) Massive Pulmonary Embolism
(b) Restrictive Cardiomyopathy
(c) Hypertrophic Cardiomyopathy
(d) Right Ventricular Infarction

6.
Which one of the following is NOT a common cause of atrial fibrillation?

(a) Mitral regurgitation
(b) Hypothyroidism
(c) Hypertension
(d) Acute myocardial infarction

7.
In an ECG recording, P wave is produced by:

(a) Atrial depolarisation
(b) Atrial repolarisation
(c) Ventricular depolarisation
(d) Ventricular repolarisation
8.
Which one of the following modalities is NOT used in the treatment of ventricular
tachycardia?

(a) DC Cardioversion
(b) Injection Lignocaine
(c) Injection Amiodarone
(d) Injection Adenosine
9.
Which one of the following drugs is a Direct Renin Inhibitor?

(a) Benedipine
(b) Azilsartan
(c) Aliskiren
(d) Lisinopril
10.
All of the following are causes of systolic hypertension with wide pulse pressure EXCEPT:

(a) Aortic regurgitation
(b) Thyrotoxicosis
(c) Patent ductus arteriosus
(d) Aortic stenosis
11.
Signet-ring sign on CT-chest is suggestive of:
(a) Bronchiectasis
(b) Active alveolitis
(c) Aspergilloma
(d) Sarcoidosis
12.
Oral corticosteroids are best introduced in the treatment of chronic bronchial asthma when it
is:
(a) Mild persistent
(b) Moderate persistent
(c) Severe persistent
(d) Very severe persistent
13.
An 18-year old male has insulin dependent diabetes, with malabsorption syndrome and
bilateral upper lobe bronchiectasis. A diagnostic work-up will include all EXCEPT:
(a) 24 hour fecal fat-estimation
(b) CT-chest
(c) Lung biopsy
(d) DNA sequencing study
14.
Which one of the following antigens is commonly associated with causation of Farmer?s
lung?
(a) Penicillium
(b) Actinomycetes
(c) Aspergillus
(d) Candida
15.
The Gene X-pert test used for MTB detection has the additional advantage of detection of
which of the following?

(a) INH resistance
(b) Rifampicin resistance
(c) Multi drug resistance
(d) Ethambutol resistance
16.
The most common cause of chronic type II respiratory failure is:
(a) Severe pneumonia
(b) Severe COPD
(c) Severe bronchial asthma
(d) Severe pulmonary thromboembolism
17.
The most frequent symptom in respiratory diseases is:
(a) Breathlessness
(b) Chest pain
(c) Hemoptysis
(d) Cough
18.
A 26-year old young lady attends medical emergency and is labelled as acute severe asthma
by the emergency physician. Which one of the following is unlikely to be a part of the
prescription?

(a) Albuterol
(b) Anti-leukotrienes
(c) Aminophylline
(d) Magnesium sulfate
19.
Consider the following statements with regard to respiratory examination:

1. Change in note, when patient phonates ?EEE? (Egophony) is characteristic of
interstitial fibrosis
2. Whispered pectoriloquy is characteristic of lung consolidation
3. Monophonic wheeze is characteristic of asthma
4. Hyper-resonant note on percussion is characteristic of pnemothorax

Which of the above statements are correct?
(a) 1 and 3
(b) 2 and 4 only
(c) 1, 2 and 4
(d) 2, 3 and 4
20.
All of the following are the causes of exudative Pleural Effusion, EXCEPT:
(a) SVC obstruction
(b) Fungal infection
(c) SLE
(d) Meig syndrome
21.
Which one of the following is NOT true about Ghrelin?
(a) It stimulates appetite
(b) It decreases gastric emptying
(c) It increases acid secretion
(d) Fasting increases its secretion
22.
Fecal elastase test is used for diagnosing which one of the following conditions?
(a) Lactose intolerance
(b) Mucosal inflammation
(c) Bile acid secretory defects
(d) Pancreatic dysfunction
23.
The recent classification system for listing a patient as a candidate for liver transplantation is:
(a) Child Pugh score
(b) APACHE score
(c) MELD score
(d) Metavir score
24.
The blood supply of liver consists of:
(a) 50 % hepatic artery and 50 % portal vein
(b) 80 % portal vein and 20 % hepatic artery
(c) 80 % hepatic artery and 20 % portal vein
(d) 70 % hepatic artery and 30 % portal vein
25.
All of the following conditions are associated with Glomerulonephritis with low complement
level, EXCEPT:
(a) Subacute bacterial endocarditis
(b) IgA nephropathy
(c) Systemic lupus erythematosus
(d) Cryoglobulinaemia













26.
Consider the following statements with regard to oral aphthous ulcers:

1. They are superficial and painful
2. They may occur in women just prior to menstruation
3. Oral glucocorticoids may be needed for the treatment of severe, recurrent cases
4. They are pre-malignant and progress over few years to squamous cell carcinoma of
the oral cavity

Which of the above statements are correct?

(a) 1 and 4 only
(b) 2 and 3 only
(c) 1, 2 and 3 only
(d) 1, 2, 3 and 4
27.
Which one of the following statements about Barrett?s Oesophagus is NOT correct?

(a) It is a pre-malignant condition
(b) Normal squamous cells lining the lower oesophagus are replaced by columnar
cells
(c) Normal columnar cells lining the lower oesophagus are replaced by squamous
cells
(d) It is an adaptive response to chronic gastro-oesophageal reflux

28.
The most common cause of acute hepatitis outbreaks in India is:

(a) Hepatitis E
(b) Hepatitis C
(c) Hepatitis B
(d) Hepatitis A

29.
Spontaneous oesophageal perforation after a bout of forceful vomiting or retching is
characteristically seen in which one of the following conditions?

(a) Menetrier?s disease
(b) Boerhaave?s syndrome
(c) Achalasia of the oesophagus
(d) Barrett?s oesophagus
30.
Eradication of helicobacter pylori infection may prove beneficial in the following extra-
gastric disorders EXCEPT:

(a) Unexplained vitamin B12 deficiency
(b) Idiopathic thrombocytopenic purpura
(c) Iron deficiency anaemia without gastro-intestinal bleeding
(d) Acute glomerulonephritis

31.
What is the target blood Hemoglobin level when treating anaemia in Stage-4 chronic kidney
disease patient?
(a) 8 to 10 gm/dL
(b) 10 to 12 gm/dL
(c) 12 to 13 gm/dL
(d) 13 to 14 gm/dL
32.
Which one of the following complications of chronic kidney disease is observed in patients
with low parathyroid hormone levels?
(a) Adynamic bone disease
(b) Tumoral calcinosis
(c) Osteitis fibrosa cystic
(d) Calciphylaxis
33.
All of the following statements about adult polycystic kidney disease are true, EXCEPT:
(a) It is inherited as autosomal dominant trait
(b) It is usually associated with marked proteinuria
(c) It is usually associated with systematic hypertension, from young age
(d) It is associated with Berry aneurysm
34.
Consider the following statements in relation to an adult patient:

1. Oliguria is defined as passage of urine less than 300 ml per day
2. Anuria is said to exist when less than 50 ml urine is passed per day
3. Polyuria is defined as urine volume in excess of 3 litre per day

Which of the statements given above are correct?
(a) 1 and 3 only
(b) 2 and 3 only
(c) 1 and 2 only
(d) 1, 2 and 3

35.
White cell casts in urine examination are strongly suggestive of:
(a) Nephritis
(b) Pyelonephritis
(c) Renal stone disease
(d) Papillitis
36.
A 13-year old boy presents with hematuria, oliguria, edema and hypertension. He has history
of sore throat two weeks prior to presentation. Laboratory investigations are remarkable for
low C
3
and increased titres of ASO and antiDNase. Which one of the following statements is
NOT correct about management for this condition?

(a) Renal biopsy is rarely required for making diagnosis
(b) Antibiotic treatment is given for streptococcal infection
(c) Treatment is largely supportive
(d) Immunosuppressants are to be used for crescentic glomerulonephritis
FirstRanker.com - FirstRanker's Choice
Combined Medical Services Examination-2020
Paper-I
1.
A chronic alcoholic develops a paroxysm of palpitations after alcohol binge. Which of the
following Arrhythmia is most likely?
(a) Ventricular fibrillation
(b) Ventricular premature complex
(c) Atrial flutter
(d) Atrial fibrillation
2.
An elderly-man with history of Diabetes mellitus and Coronary Artery Disease comes for
follow-up, with complaints of muscle pains. Which one of the following drugs could be the
most likely cause?
(a) Aspirin
(b) Glimepiride
(c) Enalapril
(d) Atorvastatin
3.
A 40-year old lady comes to outdoor clinic with complaints of sudden onset chest pain. The
chest X-ray shows bilateral Pneumothorax. Examination reveals abnormalities of body
habitus-including long arms, legs and finger (arachnodactyly), scoliosis, high arched palate,
joint hypermobility, and a pansystolic murmur at cardiac apex. Which one of the following
diseases is most likely?
(a) Takayasu?s arteritis
(b) Raynaud?s syndrome
(c) Marfan?s syndrome
(d) Rheumatic heart disease-Mitral regurgitation
4.
All of the following are indications for treadmill testing/exercise-testing EXCEPT:
(a) To confirm the diagnosis of angina
(b) To evaluate stable angina
(c) To assess outcome after coronary revascularization
(d) To evaluate the treatment efficacy of antianginal drugs
5.
Kussmaul?s sign is present in all of the following conditions, EXCEPT:

(a) Massive Pulmonary Embolism
(b) Restrictive Cardiomyopathy
(c) Hypertrophic Cardiomyopathy
(d) Right Ventricular Infarction

6.
Which one of the following is NOT a common cause of atrial fibrillation?

(a) Mitral regurgitation
(b) Hypothyroidism
(c) Hypertension
(d) Acute myocardial infarction

7.
In an ECG recording, P wave is produced by:

(a) Atrial depolarisation
(b) Atrial repolarisation
(c) Ventricular depolarisation
(d) Ventricular repolarisation
8.
Which one of the following modalities is NOT used in the treatment of ventricular
tachycardia?

(a) DC Cardioversion
(b) Injection Lignocaine
(c) Injection Amiodarone
(d) Injection Adenosine
9.
Which one of the following drugs is a Direct Renin Inhibitor?

(a) Benedipine
(b) Azilsartan
(c) Aliskiren
(d) Lisinopril
10.
All of the following are causes of systolic hypertension with wide pulse pressure EXCEPT:

(a) Aortic regurgitation
(b) Thyrotoxicosis
(c) Patent ductus arteriosus
(d) Aortic stenosis
11.
Signet-ring sign on CT-chest is suggestive of:
(a) Bronchiectasis
(b) Active alveolitis
(c) Aspergilloma
(d) Sarcoidosis
12.
Oral corticosteroids are best introduced in the treatment of chronic bronchial asthma when it
is:
(a) Mild persistent
(b) Moderate persistent
(c) Severe persistent
(d) Very severe persistent
13.
An 18-year old male has insulin dependent diabetes, with malabsorption syndrome and
bilateral upper lobe bronchiectasis. A diagnostic work-up will include all EXCEPT:
(a) 24 hour fecal fat-estimation
(b) CT-chest
(c) Lung biopsy
(d) DNA sequencing study
14.
Which one of the following antigens is commonly associated with causation of Farmer?s
lung?
(a) Penicillium
(b) Actinomycetes
(c) Aspergillus
(d) Candida
15.
The Gene X-pert test used for MTB detection has the additional advantage of detection of
which of the following?

(a) INH resistance
(b) Rifampicin resistance
(c) Multi drug resistance
(d) Ethambutol resistance
16.
The most common cause of chronic type II respiratory failure is:
(a) Severe pneumonia
(b) Severe COPD
(c) Severe bronchial asthma
(d) Severe pulmonary thromboembolism
17.
The most frequent symptom in respiratory diseases is:
(a) Breathlessness
(b) Chest pain
(c) Hemoptysis
(d) Cough
18.
A 26-year old young lady attends medical emergency and is labelled as acute severe asthma
by the emergency physician. Which one of the following is unlikely to be a part of the
prescription?

(a) Albuterol
(b) Anti-leukotrienes
(c) Aminophylline
(d) Magnesium sulfate
19.
Consider the following statements with regard to respiratory examination:

1. Change in note, when patient phonates ?EEE? (Egophony) is characteristic of
interstitial fibrosis
2. Whispered pectoriloquy is characteristic of lung consolidation
3. Monophonic wheeze is characteristic of asthma
4. Hyper-resonant note on percussion is characteristic of pnemothorax

Which of the above statements are correct?
(a) 1 and 3
(b) 2 and 4 only
(c) 1, 2 and 4
(d) 2, 3 and 4
20.
All of the following are the causes of exudative Pleural Effusion, EXCEPT:
(a) SVC obstruction
(b) Fungal infection
(c) SLE
(d) Meig syndrome
21.
Which one of the following is NOT true about Ghrelin?
(a) It stimulates appetite
(b) It decreases gastric emptying
(c) It increases acid secretion
(d) Fasting increases its secretion
22.
Fecal elastase test is used for diagnosing which one of the following conditions?
(a) Lactose intolerance
(b) Mucosal inflammation
(c) Bile acid secretory defects
(d) Pancreatic dysfunction
23.
The recent classification system for listing a patient as a candidate for liver transplantation is:
(a) Child Pugh score
(b) APACHE score
(c) MELD score
(d) Metavir score
24.
The blood supply of liver consists of:
(a) 50 % hepatic artery and 50 % portal vein
(b) 80 % portal vein and 20 % hepatic artery
(c) 80 % hepatic artery and 20 % portal vein
(d) 70 % hepatic artery and 30 % portal vein
25.
All of the following conditions are associated with Glomerulonephritis with low complement
level, EXCEPT:
(a) Subacute bacterial endocarditis
(b) IgA nephropathy
(c) Systemic lupus erythematosus
(d) Cryoglobulinaemia













26.
Consider the following statements with regard to oral aphthous ulcers:

1. They are superficial and painful
2. They may occur in women just prior to menstruation
3. Oral glucocorticoids may be needed for the treatment of severe, recurrent cases
4. They are pre-malignant and progress over few years to squamous cell carcinoma of
the oral cavity

Which of the above statements are correct?

(a) 1 and 4 only
(b) 2 and 3 only
(c) 1, 2 and 3 only
(d) 1, 2, 3 and 4
27.
Which one of the following statements about Barrett?s Oesophagus is NOT correct?

(a) It is a pre-malignant condition
(b) Normal squamous cells lining the lower oesophagus are replaced by columnar
cells
(c) Normal columnar cells lining the lower oesophagus are replaced by squamous
cells
(d) It is an adaptive response to chronic gastro-oesophageal reflux

28.
The most common cause of acute hepatitis outbreaks in India is:

(a) Hepatitis E
(b) Hepatitis C
(c) Hepatitis B
(d) Hepatitis A

29.
Spontaneous oesophageal perforation after a bout of forceful vomiting or retching is
characteristically seen in which one of the following conditions?

(a) Menetrier?s disease
(b) Boerhaave?s syndrome
(c) Achalasia of the oesophagus
(d) Barrett?s oesophagus
30.
Eradication of helicobacter pylori infection may prove beneficial in the following extra-
gastric disorders EXCEPT:

(a) Unexplained vitamin B12 deficiency
(b) Idiopathic thrombocytopenic purpura
(c) Iron deficiency anaemia without gastro-intestinal bleeding
(d) Acute glomerulonephritis

31.
What is the target blood Hemoglobin level when treating anaemia in Stage-4 chronic kidney
disease patient?
(a) 8 to 10 gm/dL
(b) 10 to 12 gm/dL
(c) 12 to 13 gm/dL
(d) 13 to 14 gm/dL
32.
Which one of the following complications of chronic kidney disease is observed in patients
with low parathyroid hormone levels?
(a) Adynamic bone disease
(b) Tumoral calcinosis
(c) Osteitis fibrosa cystic
(d) Calciphylaxis
33.
All of the following statements about adult polycystic kidney disease are true, EXCEPT:
(a) It is inherited as autosomal dominant trait
(b) It is usually associated with marked proteinuria
(c) It is usually associated with systematic hypertension, from young age
(d) It is associated with Berry aneurysm
34.
Consider the following statements in relation to an adult patient:

1. Oliguria is defined as passage of urine less than 300 ml per day
2. Anuria is said to exist when less than 50 ml urine is passed per day
3. Polyuria is defined as urine volume in excess of 3 litre per day

Which of the statements given above are correct?
(a) 1 and 3 only
(b) 2 and 3 only
(c) 1 and 2 only
(d) 1, 2 and 3

35.
White cell casts in urine examination are strongly suggestive of:
(a) Nephritis
(b) Pyelonephritis
(c) Renal stone disease
(d) Papillitis
36.
A 13-year old boy presents with hematuria, oliguria, edema and hypertension. He has history
of sore throat two weeks prior to presentation. Laboratory investigations are remarkable for
low C
3
and increased titres of ASO and antiDNase. Which one of the following statements is
NOT correct about management for this condition?

(a) Renal biopsy is rarely required for making diagnosis
(b) Antibiotic treatment is given for streptococcal infection
(c) Treatment is largely supportive
(d) Immunosuppressants are to be used for crescentic glomerulonephritis
37.
Which one of the following statements is NOT correct regarding the risk factors for
nephrolithiasis?

(a) Higher dietary calcium increases risk
(b) Higher animal protein intake increases risk
(c) Higher dietary potassium intake lowers risk
(d) Lower urine citrate increases the risk
38.
A 60-year old patient is admitted in emergency with seizure, aphasia and altered sensorium.
He has history of fever and headache for the preceding 3 days. The CSF examination is
unremarkable. What would be the probable diagnosis?
(a) Multiple sclerosis
(b) Tubercular meningitis
(c) Pyogenic (bacterial) meningitis
(d) Viral encephalitis
39.
Drug of choice for the treatment of Trigeminal Neuralgia is:
(a) Carbamazepine
(b) Aceclofenac
(c) Thiamine
(d) Prednisdone
40.
Constructional skills is a function of which lobe?
(a) Dominant temporal lobe
(b) Non dominant parietal lobe
(c) Frontal lobe
(d) Non dominant temporal lobe

41.
Vertical gaze palsy with convergence retraction nystagmus is seen in:
(a) Weber syndrome
(b) Millard Gubler syndrome
(c) Claude syndrome
(d) Parinaud syndrome
42.
A patient on looking forward was found to have his right eye deviated downwards and
outwards with pupil dilated. He is suffering from:
(a) Left 3
rd
nerve palsy
(b) Left 6
th
nerve palsy
(c) Right 4
th
nerve palsy
(d) Right 3
rd
nerve palsy
43.
All of the following are features of Cauda Equina syndrome EXCEPT:

(a) Low back pain
(b) Lower limb areflexia
(c) Loss of bladder function
(d) Extensor plantar response

FirstRanker.com - FirstRanker's Choice
Combined Medical Services Examination-2020
Paper-I
1.
A chronic alcoholic develops a paroxysm of palpitations after alcohol binge. Which of the
following Arrhythmia is most likely?
(a) Ventricular fibrillation
(b) Ventricular premature complex
(c) Atrial flutter
(d) Atrial fibrillation
2.
An elderly-man with history of Diabetes mellitus and Coronary Artery Disease comes for
follow-up, with complaints of muscle pains. Which one of the following drugs could be the
most likely cause?
(a) Aspirin
(b) Glimepiride
(c) Enalapril
(d) Atorvastatin
3.
A 40-year old lady comes to outdoor clinic with complaints of sudden onset chest pain. The
chest X-ray shows bilateral Pneumothorax. Examination reveals abnormalities of body
habitus-including long arms, legs and finger (arachnodactyly), scoliosis, high arched palate,
joint hypermobility, and a pansystolic murmur at cardiac apex. Which one of the following
diseases is most likely?
(a) Takayasu?s arteritis
(b) Raynaud?s syndrome
(c) Marfan?s syndrome
(d) Rheumatic heart disease-Mitral regurgitation
4.
All of the following are indications for treadmill testing/exercise-testing EXCEPT:
(a) To confirm the diagnosis of angina
(b) To evaluate stable angina
(c) To assess outcome after coronary revascularization
(d) To evaluate the treatment efficacy of antianginal drugs
5.
Kussmaul?s sign is present in all of the following conditions, EXCEPT:

(a) Massive Pulmonary Embolism
(b) Restrictive Cardiomyopathy
(c) Hypertrophic Cardiomyopathy
(d) Right Ventricular Infarction

6.
Which one of the following is NOT a common cause of atrial fibrillation?

(a) Mitral regurgitation
(b) Hypothyroidism
(c) Hypertension
(d) Acute myocardial infarction

7.
In an ECG recording, P wave is produced by:

(a) Atrial depolarisation
(b) Atrial repolarisation
(c) Ventricular depolarisation
(d) Ventricular repolarisation
8.
Which one of the following modalities is NOT used in the treatment of ventricular
tachycardia?

(a) DC Cardioversion
(b) Injection Lignocaine
(c) Injection Amiodarone
(d) Injection Adenosine
9.
Which one of the following drugs is a Direct Renin Inhibitor?

(a) Benedipine
(b) Azilsartan
(c) Aliskiren
(d) Lisinopril
10.
All of the following are causes of systolic hypertension with wide pulse pressure EXCEPT:

(a) Aortic regurgitation
(b) Thyrotoxicosis
(c) Patent ductus arteriosus
(d) Aortic stenosis
11.
Signet-ring sign on CT-chest is suggestive of:
(a) Bronchiectasis
(b) Active alveolitis
(c) Aspergilloma
(d) Sarcoidosis
12.
Oral corticosteroids are best introduced in the treatment of chronic bronchial asthma when it
is:
(a) Mild persistent
(b) Moderate persistent
(c) Severe persistent
(d) Very severe persistent
13.
An 18-year old male has insulin dependent diabetes, with malabsorption syndrome and
bilateral upper lobe bronchiectasis. A diagnostic work-up will include all EXCEPT:
(a) 24 hour fecal fat-estimation
(b) CT-chest
(c) Lung biopsy
(d) DNA sequencing study
14.
Which one of the following antigens is commonly associated with causation of Farmer?s
lung?
(a) Penicillium
(b) Actinomycetes
(c) Aspergillus
(d) Candida
15.
The Gene X-pert test used for MTB detection has the additional advantage of detection of
which of the following?

(a) INH resistance
(b) Rifampicin resistance
(c) Multi drug resistance
(d) Ethambutol resistance
16.
The most common cause of chronic type II respiratory failure is:
(a) Severe pneumonia
(b) Severe COPD
(c) Severe bronchial asthma
(d) Severe pulmonary thromboembolism
17.
The most frequent symptom in respiratory diseases is:
(a) Breathlessness
(b) Chest pain
(c) Hemoptysis
(d) Cough
18.
A 26-year old young lady attends medical emergency and is labelled as acute severe asthma
by the emergency physician. Which one of the following is unlikely to be a part of the
prescription?

(a) Albuterol
(b) Anti-leukotrienes
(c) Aminophylline
(d) Magnesium sulfate
19.
Consider the following statements with regard to respiratory examination:

1. Change in note, when patient phonates ?EEE? (Egophony) is characteristic of
interstitial fibrosis
2. Whispered pectoriloquy is characteristic of lung consolidation
3. Monophonic wheeze is characteristic of asthma
4. Hyper-resonant note on percussion is characteristic of pnemothorax

Which of the above statements are correct?
(a) 1 and 3
(b) 2 and 4 only
(c) 1, 2 and 4
(d) 2, 3 and 4
20.
All of the following are the causes of exudative Pleural Effusion, EXCEPT:
(a) SVC obstruction
(b) Fungal infection
(c) SLE
(d) Meig syndrome
21.
Which one of the following is NOT true about Ghrelin?
(a) It stimulates appetite
(b) It decreases gastric emptying
(c) It increases acid secretion
(d) Fasting increases its secretion
22.
Fecal elastase test is used for diagnosing which one of the following conditions?
(a) Lactose intolerance
(b) Mucosal inflammation
(c) Bile acid secretory defects
(d) Pancreatic dysfunction
23.
The recent classification system for listing a patient as a candidate for liver transplantation is:
(a) Child Pugh score
(b) APACHE score
(c) MELD score
(d) Metavir score
24.
The blood supply of liver consists of:
(a) 50 % hepatic artery and 50 % portal vein
(b) 80 % portal vein and 20 % hepatic artery
(c) 80 % hepatic artery and 20 % portal vein
(d) 70 % hepatic artery and 30 % portal vein
25.
All of the following conditions are associated with Glomerulonephritis with low complement
level, EXCEPT:
(a) Subacute bacterial endocarditis
(b) IgA nephropathy
(c) Systemic lupus erythematosus
(d) Cryoglobulinaemia













26.
Consider the following statements with regard to oral aphthous ulcers:

1. They are superficial and painful
2. They may occur in women just prior to menstruation
3. Oral glucocorticoids may be needed for the treatment of severe, recurrent cases
4. They are pre-malignant and progress over few years to squamous cell carcinoma of
the oral cavity

Which of the above statements are correct?

(a) 1 and 4 only
(b) 2 and 3 only
(c) 1, 2 and 3 only
(d) 1, 2, 3 and 4
27.
Which one of the following statements about Barrett?s Oesophagus is NOT correct?

(a) It is a pre-malignant condition
(b) Normal squamous cells lining the lower oesophagus are replaced by columnar
cells
(c) Normal columnar cells lining the lower oesophagus are replaced by squamous
cells
(d) It is an adaptive response to chronic gastro-oesophageal reflux

28.
The most common cause of acute hepatitis outbreaks in India is:

(a) Hepatitis E
(b) Hepatitis C
(c) Hepatitis B
(d) Hepatitis A

29.
Spontaneous oesophageal perforation after a bout of forceful vomiting or retching is
characteristically seen in which one of the following conditions?

(a) Menetrier?s disease
(b) Boerhaave?s syndrome
(c) Achalasia of the oesophagus
(d) Barrett?s oesophagus
30.
Eradication of helicobacter pylori infection may prove beneficial in the following extra-
gastric disorders EXCEPT:

(a) Unexplained vitamin B12 deficiency
(b) Idiopathic thrombocytopenic purpura
(c) Iron deficiency anaemia without gastro-intestinal bleeding
(d) Acute glomerulonephritis

31.
What is the target blood Hemoglobin level when treating anaemia in Stage-4 chronic kidney
disease patient?
(a) 8 to 10 gm/dL
(b) 10 to 12 gm/dL
(c) 12 to 13 gm/dL
(d) 13 to 14 gm/dL
32.
Which one of the following complications of chronic kidney disease is observed in patients
with low parathyroid hormone levels?
(a) Adynamic bone disease
(b) Tumoral calcinosis
(c) Osteitis fibrosa cystic
(d) Calciphylaxis
33.
All of the following statements about adult polycystic kidney disease are true, EXCEPT:
(a) It is inherited as autosomal dominant trait
(b) It is usually associated with marked proteinuria
(c) It is usually associated with systematic hypertension, from young age
(d) It is associated with Berry aneurysm
34.
Consider the following statements in relation to an adult patient:

1. Oliguria is defined as passage of urine less than 300 ml per day
2. Anuria is said to exist when less than 50 ml urine is passed per day
3. Polyuria is defined as urine volume in excess of 3 litre per day

Which of the statements given above are correct?
(a) 1 and 3 only
(b) 2 and 3 only
(c) 1 and 2 only
(d) 1, 2 and 3

35.
White cell casts in urine examination are strongly suggestive of:
(a) Nephritis
(b) Pyelonephritis
(c) Renal stone disease
(d) Papillitis
36.
A 13-year old boy presents with hematuria, oliguria, edema and hypertension. He has history
of sore throat two weeks prior to presentation. Laboratory investigations are remarkable for
low C
3
and increased titres of ASO and antiDNase. Which one of the following statements is
NOT correct about management for this condition?

(a) Renal biopsy is rarely required for making diagnosis
(b) Antibiotic treatment is given for streptococcal infection
(c) Treatment is largely supportive
(d) Immunosuppressants are to be used for crescentic glomerulonephritis
37.
Which one of the following statements is NOT correct regarding the risk factors for
nephrolithiasis?

(a) Higher dietary calcium increases risk
(b) Higher animal protein intake increases risk
(c) Higher dietary potassium intake lowers risk
(d) Lower urine citrate increases the risk
38.
A 60-year old patient is admitted in emergency with seizure, aphasia and altered sensorium.
He has history of fever and headache for the preceding 3 days. The CSF examination is
unremarkable. What would be the probable diagnosis?
(a) Multiple sclerosis
(b) Tubercular meningitis
(c) Pyogenic (bacterial) meningitis
(d) Viral encephalitis
39.
Drug of choice for the treatment of Trigeminal Neuralgia is:
(a) Carbamazepine
(b) Aceclofenac
(c) Thiamine
(d) Prednisdone
40.
Constructional skills is a function of which lobe?
(a) Dominant temporal lobe
(b) Non dominant parietal lobe
(c) Frontal lobe
(d) Non dominant temporal lobe

41.
Vertical gaze palsy with convergence retraction nystagmus is seen in:
(a) Weber syndrome
(b) Millard Gubler syndrome
(c) Claude syndrome
(d) Parinaud syndrome
42.
A patient on looking forward was found to have his right eye deviated downwards and
outwards with pupil dilated. He is suffering from:
(a) Left 3
rd
nerve palsy
(b) Left 6
th
nerve palsy
(c) Right 4
th
nerve palsy
(d) Right 3
rd
nerve palsy
43.
All of the following are features of Cauda Equina syndrome EXCEPT:

(a) Low back pain
(b) Lower limb areflexia
(c) Loss of bladder function
(d) Extensor plantar response

44.
A 68-year old male presented in OPD with complaints of progressive small handwriting. On
examination, he had resting tremor, bradykinesia, rigidity and postural instability. This
patient is suffering most likely from:

(a) Parkinson?s disease
(b) Vascular dementia
(c) Alzheimer?s disease
(d) Frunto temporal dementia
45.
An elderly patient presents with abnormal gait. He has a wide base freezing gait with
imbalance, comprising short strides, shuffling along the floor and difficulty with starts and
turns. Heal-Shin test is normal. Which one of the following is most likely disorder?
(a) Cerebellar ataxia
(b) Sensory ataxia
(c) Frontal gait disorder
(d) Parkinson?s disease
46.
Cushing reflex, seen in conditions of raised intra cranial pressure includes all of the following
EXCEPT:
(a) Bradycardia
(b) Hypertension
(c) Irregular respiration
(d) Hypothermia

47.
A 35 year old man presents with history of low grade fever and headache for last five weeks.
Clinical examination is remarkable for signs of meningismus. He undergoes lumbar puncture
(LP) and a day later reports worsening of headache. Which one of the following features is
NOT consistent with diagnosis of ?Post-LP? headache?

(a) Post-LP headache usually begins within 48 hours
(b) Post-LP headache worsens in sitting position
(c) Post-LP headache is most severe upon waking up
(d) Post-LP headache may improve with caffeine intake
48.
Which one of the following investigations is NOT appropriate in a case of recurrent
thrombosis?
(a) Antiphospholipid antibodies
(b) Bcr-Abl assay
(c) Protein C and S assays
(d) Antithrombin level
49.
All of the following are causes of reactive thrombocytosis, EXCEPT:
(a) Hemolytic anemia
(b) Megaloblastic anemia
(c) Post splenectomy
(d) Chronic inflammatory disorders


FirstRanker.com - FirstRanker's Choice
Combined Medical Services Examination-2020
Paper-I
1.
A chronic alcoholic develops a paroxysm of palpitations after alcohol binge. Which of the
following Arrhythmia is most likely?
(a) Ventricular fibrillation
(b) Ventricular premature complex
(c) Atrial flutter
(d) Atrial fibrillation
2.
An elderly-man with history of Diabetes mellitus and Coronary Artery Disease comes for
follow-up, with complaints of muscle pains. Which one of the following drugs could be the
most likely cause?
(a) Aspirin
(b) Glimepiride
(c) Enalapril
(d) Atorvastatin
3.
A 40-year old lady comes to outdoor clinic with complaints of sudden onset chest pain. The
chest X-ray shows bilateral Pneumothorax. Examination reveals abnormalities of body
habitus-including long arms, legs and finger (arachnodactyly), scoliosis, high arched palate,
joint hypermobility, and a pansystolic murmur at cardiac apex. Which one of the following
diseases is most likely?
(a) Takayasu?s arteritis
(b) Raynaud?s syndrome
(c) Marfan?s syndrome
(d) Rheumatic heart disease-Mitral regurgitation
4.
All of the following are indications for treadmill testing/exercise-testing EXCEPT:
(a) To confirm the diagnosis of angina
(b) To evaluate stable angina
(c) To assess outcome after coronary revascularization
(d) To evaluate the treatment efficacy of antianginal drugs
5.
Kussmaul?s sign is present in all of the following conditions, EXCEPT:

(a) Massive Pulmonary Embolism
(b) Restrictive Cardiomyopathy
(c) Hypertrophic Cardiomyopathy
(d) Right Ventricular Infarction

6.
Which one of the following is NOT a common cause of atrial fibrillation?

(a) Mitral regurgitation
(b) Hypothyroidism
(c) Hypertension
(d) Acute myocardial infarction

7.
In an ECG recording, P wave is produced by:

(a) Atrial depolarisation
(b) Atrial repolarisation
(c) Ventricular depolarisation
(d) Ventricular repolarisation
8.
Which one of the following modalities is NOT used in the treatment of ventricular
tachycardia?

(a) DC Cardioversion
(b) Injection Lignocaine
(c) Injection Amiodarone
(d) Injection Adenosine
9.
Which one of the following drugs is a Direct Renin Inhibitor?

(a) Benedipine
(b) Azilsartan
(c) Aliskiren
(d) Lisinopril
10.
All of the following are causes of systolic hypertension with wide pulse pressure EXCEPT:

(a) Aortic regurgitation
(b) Thyrotoxicosis
(c) Patent ductus arteriosus
(d) Aortic stenosis
11.
Signet-ring sign on CT-chest is suggestive of:
(a) Bronchiectasis
(b) Active alveolitis
(c) Aspergilloma
(d) Sarcoidosis
12.
Oral corticosteroids are best introduced in the treatment of chronic bronchial asthma when it
is:
(a) Mild persistent
(b) Moderate persistent
(c) Severe persistent
(d) Very severe persistent
13.
An 18-year old male has insulin dependent diabetes, with malabsorption syndrome and
bilateral upper lobe bronchiectasis. A diagnostic work-up will include all EXCEPT:
(a) 24 hour fecal fat-estimation
(b) CT-chest
(c) Lung biopsy
(d) DNA sequencing study
14.
Which one of the following antigens is commonly associated with causation of Farmer?s
lung?
(a) Penicillium
(b) Actinomycetes
(c) Aspergillus
(d) Candida
15.
The Gene X-pert test used for MTB detection has the additional advantage of detection of
which of the following?

(a) INH resistance
(b) Rifampicin resistance
(c) Multi drug resistance
(d) Ethambutol resistance
16.
The most common cause of chronic type II respiratory failure is:
(a) Severe pneumonia
(b) Severe COPD
(c) Severe bronchial asthma
(d) Severe pulmonary thromboembolism
17.
The most frequent symptom in respiratory diseases is:
(a) Breathlessness
(b) Chest pain
(c) Hemoptysis
(d) Cough
18.
A 26-year old young lady attends medical emergency and is labelled as acute severe asthma
by the emergency physician. Which one of the following is unlikely to be a part of the
prescription?

(a) Albuterol
(b) Anti-leukotrienes
(c) Aminophylline
(d) Magnesium sulfate
19.
Consider the following statements with regard to respiratory examination:

1. Change in note, when patient phonates ?EEE? (Egophony) is characteristic of
interstitial fibrosis
2. Whispered pectoriloquy is characteristic of lung consolidation
3. Monophonic wheeze is characteristic of asthma
4. Hyper-resonant note on percussion is characteristic of pnemothorax

Which of the above statements are correct?
(a) 1 and 3
(b) 2 and 4 only
(c) 1, 2 and 4
(d) 2, 3 and 4
20.
All of the following are the causes of exudative Pleural Effusion, EXCEPT:
(a) SVC obstruction
(b) Fungal infection
(c) SLE
(d) Meig syndrome
21.
Which one of the following is NOT true about Ghrelin?
(a) It stimulates appetite
(b) It decreases gastric emptying
(c) It increases acid secretion
(d) Fasting increases its secretion
22.
Fecal elastase test is used for diagnosing which one of the following conditions?
(a) Lactose intolerance
(b) Mucosal inflammation
(c) Bile acid secretory defects
(d) Pancreatic dysfunction
23.
The recent classification system for listing a patient as a candidate for liver transplantation is:
(a) Child Pugh score
(b) APACHE score
(c) MELD score
(d) Metavir score
24.
The blood supply of liver consists of:
(a) 50 % hepatic artery and 50 % portal vein
(b) 80 % portal vein and 20 % hepatic artery
(c) 80 % hepatic artery and 20 % portal vein
(d) 70 % hepatic artery and 30 % portal vein
25.
All of the following conditions are associated with Glomerulonephritis with low complement
level, EXCEPT:
(a) Subacute bacterial endocarditis
(b) IgA nephropathy
(c) Systemic lupus erythematosus
(d) Cryoglobulinaemia













26.
Consider the following statements with regard to oral aphthous ulcers:

1. They are superficial and painful
2. They may occur in women just prior to menstruation
3. Oral glucocorticoids may be needed for the treatment of severe, recurrent cases
4. They are pre-malignant and progress over few years to squamous cell carcinoma of
the oral cavity

Which of the above statements are correct?

(a) 1 and 4 only
(b) 2 and 3 only
(c) 1, 2 and 3 only
(d) 1, 2, 3 and 4
27.
Which one of the following statements about Barrett?s Oesophagus is NOT correct?

(a) It is a pre-malignant condition
(b) Normal squamous cells lining the lower oesophagus are replaced by columnar
cells
(c) Normal columnar cells lining the lower oesophagus are replaced by squamous
cells
(d) It is an adaptive response to chronic gastro-oesophageal reflux

28.
The most common cause of acute hepatitis outbreaks in India is:

(a) Hepatitis E
(b) Hepatitis C
(c) Hepatitis B
(d) Hepatitis A

29.
Spontaneous oesophageal perforation after a bout of forceful vomiting or retching is
characteristically seen in which one of the following conditions?

(a) Menetrier?s disease
(b) Boerhaave?s syndrome
(c) Achalasia of the oesophagus
(d) Barrett?s oesophagus
30.
Eradication of helicobacter pylori infection may prove beneficial in the following extra-
gastric disorders EXCEPT:

(a) Unexplained vitamin B12 deficiency
(b) Idiopathic thrombocytopenic purpura
(c) Iron deficiency anaemia without gastro-intestinal bleeding
(d) Acute glomerulonephritis

31.
What is the target blood Hemoglobin level when treating anaemia in Stage-4 chronic kidney
disease patient?
(a) 8 to 10 gm/dL
(b) 10 to 12 gm/dL
(c) 12 to 13 gm/dL
(d) 13 to 14 gm/dL
32.
Which one of the following complications of chronic kidney disease is observed in patients
with low parathyroid hormone levels?
(a) Adynamic bone disease
(b) Tumoral calcinosis
(c) Osteitis fibrosa cystic
(d) Calciphylaxis
33.
All of the following statements about adult polycystic kidney disease are true, EXCEPT:
(a) It is inherited as autosomal dominant trait
(b) It is usually associated with marked proteinuria
(c) It is usually associated with systematic hypertension, from young age
(d) It is associated with Berry aneurysm
34.
Consider the following statements in relation to an adult patient:

1. Oliguria is defined as passage of urine less than 300 ml per day
2. Anuria is said to exist when less than 50 ml urine is passed per day
3. Polyuria is defined as urine volume in excess of 3 litre per day

Which of the statements given above are correct?
(a) 1 and 3 only
(b) 2 and 3 only
(c) 1 and 2 only
(d) 1, 2 and 3

35.
White cell casts in urine examination are strongly suggestive of:
(a) Nephritis
(b) Pyelonephritis
(c) Renal stone disease
(d) Papillitis
36.
A 13-year old boy presents with hematuria, oliguria, edema and hypertension. He has history
of sore throat two weeks prior to presentation. Laboratory investigations are remarkable for
low C
3
and increased titres of ASO and antiDNase. Which one of the following statements is
NOT correct about management for this condition?

(a) Renal biopsy is rarely required for making diagnosis
(b) Antibiotic treatment is given for streptococcal infection
(c) Treatment is largely supportive
(d) Immunosuppressants are to be used for crescentic glomerulonephritis
37.
Which one of the following statements is NOT correct regarding the risk factors for
nephrolithiasis?

(a) Higher dietary calcium increases risk
(b) Higher animal protein intake increases risk
(c) Higher dietary potassium intake lowers risk
(d) Lower urine citrate increases the risk
38.
A 60-year old patient is admitted in emergency with seizure, aphasia and altered sensorium.
He has history of fever and headache for the preceding 3 days. The CSF examination is
unremarkable. What would be the probable diagnosis?
(a) Multiple sclerosis
(b) Tubercular meningitis
(c) Pyogenic (bacterial) meningitis
(d) Viral encephalitis
39.
Drug of choice for the treatment of Trigeminal Neuralgia is:
(a) Carbamazepine
(b) Aceclofenac
(c) Thiamine
(d) Prednisdone
40.
Constructional skills is a function of which lobe?
(a) Dominant temporal lobe
(b) Non dominant parietal lobe
(c) Frontal lobe
(d) Non dominant temporal lobe

41.
Vertical gaze palsy with convergence retraction nystagmus is seen in:
(a) Weber syndrome
(b) Millard Gubler syndrome
(c) Claude syndrome
(d) Parinaud syndrome
42.
A patient on looking forward was found to have his right eye deviated downwards and
outwards with pupil dilated. He is suffering from:
(a) Left 3
rd
nerve palsy
(b) Left 6
th
nerve palsy
(c) Right 4
th
nerve palsy
(d) Right 3
rd
nerve palsy
43.
All of the following are features of Cauda Equina syndrome EXCEPT:

(a) Low back pain
(b) Lower limb areflexia
(c) Loss of bladder function
(d) Extensor plantar response

44.
A 68-year old male presented in OPD with complaints of progressive small handwriting. On
examination, he had resting tremor, bradykinesia, rigidity and postural instability. This
patient is suffering most likely from:

(a) Parkinson?s disease
(b) Vascular dementia
(c) Alzheimer?s disease
(d) Frunto temporal dementia
45.
An elderly patient presents with abnormal gait. He has a wide base freezing gait with
imbalance, comprising short strides, shuffling along the floor and difficulty with starts and
turns. Heal-Shin test is normal. Which one of the following is most likely disorder?
(a) Cerebellar ataxia
(b) Sensory ataxia
(c) Frontal gait disorder
(d) Parkinson?s disease
46.
Cushing reflex, seen in conditions of raised intra cranial pressure includes all of the following
EXCEPT:
(a) Bradycardia
(b) Hypertension
(c) Irregular respiration
(d) Hypothermia

47.
A 35 year old man presents with history of low grade fever and headache for last five weeks.
Clinical examination is remarkable for signs of meningismus. He undergoes lumbar puncture
(LP) and a day later reports worsening of headache. Which one of the following features is
NOT consistent with diagnosis of ?Post-LP? headache?

(a) Post-LP headache usually begins within 48 hours
(b) Post-LP headache worsens in sitting position
(c) Post-LP headache is most severe upon waking up
(d) Post-LP headache may improve with caffeine intake
48.
Which one of the following investigations is NOT appropriate in a case of recurrent
thrombosis?
(a) Antiphospholipid antibodies
(b) Bcr-Abl assay
(c) Protein C and S assays
(d) Antithrombin level
49.
All of the following are causes of reactive thrombocytosis, EXCEPT:
(a) Hemolytic anemia
(b) Megaloblastic anemia
(c) Post splenectomy
(d) Chronic inflammatory disorders


50.
Cryoprecipitate cannot be used for treatment of which one of the following conditions?
(a) Von Willebrand disease
(b) Hypofibrinogenemia
(c) Hemophilia?B
(d) Hemphilia?A
51.
Pyruvate kinase deficiency results in deficiency of ATP production and a chronic haemolytic
anaemia. The disorder is inherited as an:

(a) Autosomal recessive trait
(b) X-linked recessive trait
(c) Autosomal dominant trait
(d) X-linked dominant trait
52.
Sickle cell disease may be associated with any of the following EXCEPT:

(a) Acute chest syndrome
(b) Aplastic crisis
(c) Splenic sequestration crisis
(d) Conn?s syndrome

53.
Fever with splenomegaly and lymphadenopathy can be seen in
(a) Infectious mononucleosis
(b) Chronic leukaemia
(c) Both infectious mononucleosis and chronic leukaemia
(d) Neither infectious mononucleosis nor chronic leukaemia
54.
Every individual carries four alpha gene alleles. Deletion of three alleles leads to
development of:
(a) Haemoglobin X disease
(b) Haemoglobin H disease
(c) Haemoglobin F disease
(d) Hydrops fetalis
55.
Which one of the following is NOT true regarding Chronic Myeloid Leukaemia (CML)?
(a) It is a clonal malignancy of haematopoetic stem
(b) Risk of developing CML is increased in monozygotic turns
(c) CML is defined by presence of BCR-ABL 1 fusion gene
(d) 10 year survival with TKI therapy is 85%









FirstRanker.com - FirstRanker's Choice
Combined Medical Services Examination-2020
Paper-I
1.
A chronic alcoholic develops a paroxysm of palpitations after alcohol binge. Which of the
following Arrhythmia is most likely?
(a) Ventricular fibrillation
(b) Ventricular premature complex
(c) Atrial flutter
(d) Atrial fibrillation
2.
An elderly-man with history of Diabetes mellitus and Coronary Artery Disease comes for
follow-up, with complaints of muscle pains. Which one of the following drugs could be the
most likely cause?
(a) Aspirin
(b) Glimepiride
(c) Enalapril
(d) Atorvastatin
3.
A 40-year old lady comes to outdoor clinic with complaints of sudden onset chest pain. The
chest X-ray shows bilateral Pneumothorax. Examination reveals abnormalities of body
habitus-including long arms, legs and finger (arachnodactyly), scoliosis, high arched palate,
joint hypermobility, and a pansystolic murmur at cardiac apex. Which one of the following
diseases is most likely?
(a) Takayasu?s arteritis
(b) Raynaud?s syndrome
(c) Marfan?s syndrome
(d) Rheumatic heart disease-Mitral regurgitation
4.
All of the following are indications for treadmill testing/exercise-testing EXCEPT:
(a) To confirm the diagnosis of angina
(b) To evaluate stable angina
(c) To assess outcome after coronary revascularization
(d) To evaluate the treatment efficacy of antianginal drugs
5.
Kussmaul?s sign is present in all of the following conditions, EXCEPT:

(a) Massive Pulmonary Embolism
(b) Restrictive Cardiomyopathy
(c) Hypertrophic Cardiomyopathy
(d) Right Ventricular Infarction

6.
Which one of the following is NOT a common cause of atrial fibrillation?

(a) Mitral regurgitation
(b) Hypothyroidism
(c) Hypertension
(d) Acute myocardial infarction

7.
In an ECG recording, P wave is produced by:

(a) Atrial depolarisation
(b) Atrial repolarisation
(c) Ventricular depolarisation
(d) Ventricular repolarisation
8.
Which one of the following modalities is NOT used in the treatment of ventricular
tachycardia?

(a) DC Cardioversion
(b) Injection Lignocaine
(c) Injection Amiodarone
(d) Injection Adenosine
9.
Which one of the following drugs is a Direct Renin Inhibitor?

(a) Benedipine
(b) Azilsartan
(c) Aliskiren
(d) Lisinopril
10.
All of the following are causes of systolic hypertension with wide pulse pressure EXCEPT:

(a) Aortic regurgitation
(b) Thyrotoxicosis
(c) Patent ductus arteriosus
(d) Aortic stenosis
11.
Signet-ring sign on CT-chest is suggestive of:
(a) Bronchiectasis
(b) Active alveolitis
(c) Aspergilloma
(d) Sarcoidosis
12.
Oral corticosteroids are best introduced in the treatment of chronic bronchial asthma when it
is:
(a) Mild persistent
(b) Moderate persistent
(c) Severe persistent
(d) Very severe persistent
13.
An 18-year old male has insulin dependent diabetes, with malabsorption syndrome and
bilateral upper lobe bronchiectasis. A diagnostic work-up will include all EXCEPT:
(a) 24 hour fecal fat-estimation
(b) CT-chest
(c) Lung biopsy
(d) DNA sequencing study
14.
Which one of the following antigens is commonly associated with causation of Farmer?s
lung?
(a) Penicillium
(b) Actinomycetes
(c) Aspergillus
(d) Candida
15.
The Gene X-pert test used for MTB detection has the additional advantage of detection of
which of the following?

(a) INH resistance
(b) Rifampicin resistance
(c) Multi drug resistance
(d) Ethambutol resistance
16.
The most common cause of chronic type II respiratory failure is:
(a) Severe pneumonia
(b) Severe COPD
(c) Severe bronchial asthma
(d) Severe pulmonary thromboembolism
17.
The most frequent symptom in respiratory diseases is:
(a) Breathlessness
(b) Chest pain
(c) Hemoptysis
(d) Cough
18.
A 26-year old young lady attends medical emergency and is labelled as acute severe asthma
by the emergency physician. Which one of the following is unlikely to be a part of the
prescription?

(a) Albuterol
(b) Anti-leukotrienes
(c) Aminophylline
(d) Magnesium sulfate
19.
Consider the following statements with regard to respiratory examination:

1. Change in note, when patient phonates ?EEE? (Egophony) is characteristic of
interstitial fibrosis
2. Whispered pectoriloquy is characteristic of lung consolidation
3. Monophonic wheeze is characteristic of asthma
4. Hyper-resonant note on percussion is characteristic of pnemothorax

Which of the above statements are correct?
(a) 1 and 3
(b) 2 and 4 only
(c) 1, 2 and 4
(d) 2, 3 and 4
20.
All of the following are the causes of exudative Pleural Effusion, EXCEPT:
(a) SVC obstruction
(b) Fungal infection
(c) SLE
(d) Meig syndrome
21.
Which one of the following is NOT true about Ghrelin?
(a) It stimulates appetite
(b) It decreases gastric emptying
(c) It increases acid secretion
(d) Fasting increases its secretion
22.
Fecal elastase test is used for diagnosing which one of the following conditions?
(a) Lactose intolerance
(b) Mucosal inflammation
(c) Bile acid secretory defects
(d) Pancreatic dysfunction
23.
The recent classification system for listing a patient as a candidate for liver transplantation is:
(a) Child Pugh score
(b) APACHE score
(c) MELD score
(d) Metavir score
24.
The blood supply of liver consists of:
(a) 50 % hepatic artery and 50 % portal vein
(b) 80 % portal vein and 20 % hepatic artery
(c) 80 % hepatic artery and 20 % portal vein
(d) 70 % hepatic artery and 30 % portal vein
25.
All of the following conditions are associated with Glomerulonephritis with low complement
level, EXCEPT:
(a) Subacute bacterial endocarditis
(b) IgA nephropathy
(c) Systemic lupus erythematosus
(d) Cryoglobulinaemia













26.
Consider the following statements with regard to oral aphthous ulcers:

1. They are superficial and painful
2. They may occur in women just prior to menstruation
3. Oral glucocorticoids may be needed for the treatment of severe, recurrent cases
4. They are pre-malignant and progress over few years to squamous cell carcinoma of
the oral cavity

Which of the above statements are correct?

(a) 1 and 4 only
(b) 2 and 3 only
(c) 1, 2 and 3 only
(d) 1, 2, 3 and 4
27.
Which one of the following statements about Barrett?s Oesophagus is NOT correct?

(a) It is a pre-malignant condition
(b) Normal squamous cells lining the lower oesophagus are replaced by columnar
cells
(c) Normal columnar cells lining the lower oesophagus are replaced by squamous
cells
(d) It is an adaptive response to chronic gastro-oesophageal reflux

28.
The most common cause of acute hepatitis outbreaks in India is:

(a) Hepatitis E
(b) Hepatitis C
(c) Hepatitis B
(d) Hepatitis A

29.
Spontaneous oesophageal perforation after a bout of forceful vomiting or retching is
characteristically seen in which one of the following conditions?

(a) Menetrier?s disease
(b) Boerhaave?s syndrome
(c) Achalasia of the oesophagus
(d) Barrett?s oesophagus
30.
Eradication of helicobacter pylori infection may prove beneficial in the following extra-
gastric disorders EXCEPT:

(a) Unexplained vitamin B12 deficiency
(b) Idiopathic thrombocytopenic purpura
(c) Iron deficiency anaemia without gastro-intestinal bleeding
(d) Acute glomerulonephritis

31.
What is the target blood Hemoglobin level when treating anaemia in Stage-4 chronic kidney
disease patient?
(a) 8 to 10 gm/dL
(b) 10 to 12 gm/dL
(c) 12 to 13 gm/dL
(d) 13 to 14 gm/dL
32.
Which one of the following complications of chronic kidney disease is observed in patients
with low parathyroid hormone levels?
(a) Adynamic bone disease
(b) Tumoral calcinosis
(c) Osteitis fibrosa cystic
(d) Calciphylaxis
33.
All of the following statements about adult polycystic kidney disease are true, EXCEPT:
(a) It is inherited as autosomal dominant trait
(b) It is usually associated with marked proteinuria
(c) It is usually associated with systematic hypertension, from young age
(d) It is associated with Berry aneurysm
34.
Consider the following statements in relation to an adult patient:

1. Oliguria is defined as passage of urine less than 300 ml per day
2. Anuria is said to exist when less than 50 ml urine is passed per day
3. Polyuria is defined as urine volume in excess of 3 litre per day

Which of the statements given above are correct?
(a) 1 and 3 only
(b) 2 and 3 only
(c) 1 and 2 only
(d) 1, 2 and 3

35.
White cell casts in urine examination are strongly suggestive of:
(a) Nephritis
(b) Pyelonephritis
(c) Renal stone disease
(d) Papillitis
36.
A 13-year old boy presents with hematuria, oliguria, edema and hypertension. He has history
of sore throat two weeks prior to presentation. Laboratory investigations are remarkable for
low C
3
and increased titres of ASO and antiDNase. Which one of the following statements is
NOT correct about management for this condition?

(a) Renal biopsy is rarely required for making diagnosis
(b) Antibiotic treatment is given for streptococcal infection
(c) Treatment is largely supportive
(d) Immunosuppressants are to be used for crescentic glomerulonephritis
37.
Which one of the following statements is NOT correct regarding the risk factors for
nephrolithiasis?

(a) Higher dietary calcium increases risk
(b) Higher animal protein intake increases risk
(c) Higher dietary potassium intake lowers risk
(d) Lower urine citrate increases the risk
38.
A 60-year old patient is admitted in emergency with seizure, aphasia and altered sensorium.
He has history of fever and headache for the preceding 3 days. The CSF examination is
unremarkable. What would be the probable diagnosis?
(a) Multiple sclerosis
(b) Tubercular meningitis
(c) Pyogenic (bacterial) meningitis
(d) Viral encephalitis
39.
Drug of choice for the treatment of Trigeminal Neuralgia is:
(a) Carbamazepine
(b) Aceclofenac
(c) Thiamine
(d) Prednisdone
40.
Constructional skills is a function of which lobe?
(a) Dominant temporal lobe
(b) Non dominant parietal lobe
(c) Frontal lobe
(d) Non dominant temporal lobe

41.
Vertical gaze palsy with convergence retraction nystagmus is seen in:
(a) Weber syndrome
(b) Millard Gubler syndrome
(c) Claude syndrome
(d) Parinaud syndrome
42.
A patient on looking forward was found to have his right eye deviated downwards and
outwards with pupil dilated. He is suffering from:
(a) Left 3
rd
nerve palsy
(b) Left 6
th
nerve palsy
(c) Right 4
th
nerve palsy
(d) Right 3
rd
nerve palsy
43.
All of the following are features of Cauda Equina syndrome EXCEPT:

(a) Low back pain
(b) Lower limb areflexia
(c) Loss of bladder function
(d) Extensor plantar response

44.
A 68-year old male presented in OPD with complaints of progressive small handwriting. On
examination, he had resting tremor, bradykinesia, rigidity and postural instability. This
patient is suffering most likely from:

(a) Parkinson?s disease
(b) Vascular dementia
(c) Alzheimer?s disease
(d) Frunto temporal dementia
45.
An elderly patient presents with abnormal gait. He has a wide base freezing gait with
imbalance, comprising short strides, shuffling along the floor and difficulty with starts and
turns. Heal-Shin test is normal. Which one of the following is most likely disorder?
(a) Cerebellar ataxia
(b) Sensory ataxia
(c) Frontal gait disorder
(d) Parkinson?s disease
46.
Cushing reflex, seen in conditions of raised intra cranial pressure includes all of the following
EXCEPT:
(a) Bradycardia
(b) Hypertension
(c) Irregular respiration
(d) Hypothermia

47.
A 35 year old man presents with history of low grade fever and headache for last five weeks.
Clinical examination is remarkable for signs of meningismus. He undergoes lumbar puncture
(LP) and a day later reports worsening of headache. Which one of the following features is
NOT consistent with diagnosis of ?Post-LP? headache?

(a) Post-LP headache usually begins within 48 hours
(b) Post-LP headache worsens in sitting position
(c) Post-LP headache is most severe upon waking up
(d) Post-LP headache may improve with caffeine intake
48.
Which one of the following investigations is NOT appropriate in a case of recurrent
thrombosis?
(a) Antiphospholipid antibodies
(b) Bcr-Abl assay
(c) Protein C and S assays
(d) Antithrombin level
49.
All of the following are causes of reactive thrombocytosis, EXCEPT:
(a) Hemolytic anemia
(b) Megaloblastic anemia
(c) Post splenectomy
(d) Chronic inflammatory disorders


50.
Cryoprecipitate cannot be used for treatment of which one of the following conditions?
(a) Von Willebrand disease
(b) Hypofibrinogenemia
(c) Hemophilia?B
(d) Hemphilia?A
51.
Pyruvate kinase deficiency results in deficiency of ATP production and a chronic haemolytic
anaemia. The disorder is inherited as an:

(a) Autosomal recessive trait
(b) X-linked recessive trait
(c) Autosomal dominant trait
(d) X-linked dominant trait
52.
Sickle cell disease may be associated with any of the following EXCEPT:

(a) Acute chest syndrome
(b) Aplastic crisis
(c) Splenic sequestration crisis
(d) Conn?s syndrome

53.
Fever with splenomegaly and lymphadenopathy can be seen in
(a) Infectious mononucleosis
(b) Chronic leukaemia
(c) Both infectious mononucleosis and chronic leukaemia
(d) Neither infectious mononucleosis nor chronic leukaemia
54.
Every individual carries four alpha gene alleles. Deletion of three alleles leads to
development of:
(a) Haemoglobin X disease
(b) Haemoglobin H disease
(c) Haemoglobin F disease
(d) Hydrops fetalis
55.
Which one of the following is NOT true regarding Chronic Myeloid Leukaemia (CML)?
(a) It is a clonal malignancy of haematopoetic stem
(b) Risk of developing CML is increased in monozygotic turns
(c) CML is defined by presence of BCR-ABL 1 fusion gene
(d) 10 year survival with TKI therapy is 85%









56.
Diagnostic criteria of multiple myeloma includes which of the following?

1. Increased malignant plasma cells in the bone marrow
2. Serum and/or urinary M protein
3. Skeletal lytic lesions
4. Skeletal blastic lesions

Select the correct answer using the code given below:
(a) 1, 2 and 3
(b) 1, 3 and 4
(c) 1, 2 and 4
(d) 2, 3 and 4
57.
The most predominant type of serum immunoglobulin involved in patients with plasma cell
disorder is:
(a) Ig G
(b) Ig A
(c) Ig D
(d) Light chains
58.
Which one of the following statements is NOT true regarding the thyroid function/disorder in
pregnancy?
(a) There is an increase in the metabolism of thyroxine by placenta
(b) Hyperemesis gravidarum may be associated with thyrotoxicosis in pregnancy
(c) Levothyroxine replacement therapy dose should be decreased by 30?50% early in
pregnancy
(d) Impaired cognitive impairment may be seen in the offspring of mother with
subclinical hypothyroidism

59.
Which one of the following is NOT the criterion for the diagnosis of Diabetes mellitus?
(a) Symptoms of diabetes and random plasma sugar concentration ? 200 mg/dL
(b) HbA1c ? 6.5 %
(c) 2-hours plasma glucose ? 200 mg/dL during an oral glucose tolerance test
(d) Fasting plasma glucose ? 110 mg/dL
60.
A young lady presents in outdoor clinic with complaints of menstrual irregularity, weight
gain, hair loss, tiredness and weakness. What will be the investigation of choice?
(a) FSH & LH level
(b) Estrogen level
(c) Free T
3
and free T
4
level
(d) Free T
4
and TSH level







FirstRanker.com - FirstRanker's Choice
Combined Medical Services Examination-2020
Paper-I
1.
A chronic alcoholic develops a paroxysm of palpitations after alcohol binge. Which of the
following Arrhythmia is most likely?
(a) Ventricular fibrillation
(b) Ventricular premature complex
(c) Atrial flutter
(d) Atrial fibrillation
2.
An elderly-man with history of Diabetes mellitus and Coronary Artery Disease comes for
follow-up, with complaints of muscle pains. Which one of the following drugs could be the
most likely cause?
(a) Aspirin
(b) Glimepiride
(c) Enalapril
(d) Atorvastatin
3.
A 40-year old lady comes to outdoor clinic with complaints of sudden onset chest pain. The
chest X-ray shows bilateral Pneumothorax. Examination reveals abnormalities of body
habitus-including long arms, legs and finger (arachnodactyly), scoliosis, high arched palate,
joint hypermobility, and a pansystolic murmur at cardiac apex. Which one of the following
diseases is most likely?
(a) Takayasu?s arteritis
(b) Raynaud?s syndrome
(c) Marfan?s syndrome
(d) Rheumatic heart disease-Mitral regurgitation
4.
All of the following are indications for treadmill testing/exercise-testing EXCEPT:
(a) To confirm the diagnosis of angina
(b) To evaluate stable angina
(c) To assess outcome after coronary revascularization
(d) To evaluate the treatment efficacy of antianginal drugs
5.
Kussmaul?s sign is present in all of the following conditions, EXCEPT:

(a) Massive Pulmonary Embolism
(b) Restrictive Cardiomyopathy
(c) Hypertrophic Cardiomyopathy
(d) Right Ventricular Infarction

6.
Which one of the following is NOT a common cause of atrial fibrillation?

(a) Mitral regurgitation
(b) Hypothyroidism
(c) Hypertension
(d) Acute myocardial infarction

7.
In an ECG recording, P wave is produced by:

(a) Atrial depolarisation
(b) Atrial repolarisation
(c) Ventricular depolarisation
(d) Ventricular repolarisation
8.
Which one of the following modalities is NOT used in the treatment of ventricular
tachycardia?

(a) DC Cardioversion
(b) Injection Lignocaine
(c) Injection Amiodarone
(d) Injection Adenosine
9.
Which one of the following drugs is a Direct Renin Inhibitor?

(a) Benedipine
(b) Azilsartan
(c) Aliskiren
(d) Lisinopril
10.
All of the following are causes of systolic hypertension with wide pulse pressure EXCEPT:

(a) Aortic regurgitation
(b) Thyrotoxicosis
(c) Patent ductus arteriosus
(d) Aortic stenosis
11.
Signet-ring sign on CT-chest is suggestive of:
(a) Bronchiectasis
(b) Active alveolitis
(c) Aspergilloma
(d) Sarcoidosis
12.
Oral corticosteroids are best introduced in the treatment of chronic bronchial asthma when it
is:
(a) Mild persistent
(b) Moderate persistent
(c) Severe persistent
(d) Very severe persistent
13.
An 18-year old male has insulin dependent diabetes, with malabsorption syndrome and
bilateral upper lobe bronchiectasis. A diagnostic work-up will include all EXCEPT:
(a) 24 hour fecal fat-estimation
(b) CT-chest
(c) Lung biopsy
(d) DNA sequencing study
14.
Which one of the following antigens is commonly associated with causation of Farmer?s
lung?
(a) Penicillium
(b) Actinomycetes
(c) Aspergillus
(d) Candida
15.
The Gene X-pert test used for MTB detection has the additional advantage of detection of
which of the following?

(a) INH resistance
(b) Rifampicin resistance
(c) Multi drug resistance
(d) Ethambutol resistance
16.
The most common cause of chronic type II respiratory failure is:
(a) Severe pneumonia
(b) Severe COPD
(c) Severe bronchial asthma
(d) Severe pulmonary thromboembolism
17.
The most frequent symptom in respiratory diseases is:
(a) Breathlessness
(b) Chest pain
(c) Hemoptysis
(d) Cough
18.
A 26-year old young lady attends medical emergency and is labelled as acute severe asthma
by the emergency physician. Which one of the following is unlikely to be a part of the
prescription?

(a) Albuterol
(b) Anti-leukotrienes
(c) Aminophylline
(d) Magnesium sulfate
19.
Consider the following statements with regard to respiratory examination:

1. Change in note, when patient phonates ?EEE? (Egophony) is characteristic of
interstitial fibrosis
2. Whispered pectoriloquy is characteristic of lung consolidation
3. Monophonic wheeze is characteristic of asthma
4. Hyper-resonant note on percussion is characteristic of pnemothorax

Which of the above statements are correct?
(a) 1 and 3
(b) 2 and 4 only
(c) 1, 2 and 4
(d) 2, 3 and 4
20.
All of the following are the causes of exudative Pleural Effusion, EXCEPT:
(a) SVC obstruction
(b) Fungal infection
(c) SLE
(d) Meig syndrome
21.
Which one of the following is NOT true about Ghrelin?
(a) It stimulates appetite
(b) It decreases gastric emptying
(c) It increases acid secretion
(d) Fasting increases its secretion
22.
Fecal elastase test is used for diagnosing which one of the following conditions?
(a) Lactose intolerance
(b) Mucosal inflammation
(c) Bile acid secretory defects
(d) Pancreatic dysfunction
23.
The recent classification system for listing a patient as a candidate for liver transplantation is:
(a) Child Pugh score
(b) APACHE score
(c) MELD score
(d) Metavir score
24.
The blood supply of liver consists of:
(a) 50 % hepatic artery and 50 % portal vein
(b) 80 % portal vein and 20 % hepatic artery
(c) 80 % hepatic artery and 20 % portal vein
(d) 70 % hepatic artery and 30 % portal vein
25.
All of the following conditions are associated with Glomerulonephritis with low complement
level, EXCEPT:
(a) Subacute bacterial endocarditis
(b) IgA nephropathy
(c) Systemic lupus erythematosus
(d) Cryoglobulinaemia













26.
Consider the following statements with regard to oral aphthous ulcers:

1. They are superficial and painful
2. They may occur in women just prior to menstruation
3. Oral glucocorticoids may be needed for the treatment of severe, recurrent cases
4. They are pre-malignant and progress over few years to squamous cell carcinoma of
the oral cavity

Which of the above statements are correct?

(a) 1 and 4 only
(b) 2 and 3 only
(c) 1, 2 and 3 only
(d) 1, 2, 3 and 4
27.
Which one of the following statements about Barrett?s Oesophagus is NOT correct?

(a) It is a pre-malignant condition
(b) Normal squamous cells lining the lower oesophagus are replaced by columnar
cells
(c) Normal columnar cells lining the lower oesophagus are replaced by squamous
cells
(d) It is an adaptive response to chronic gastro-oesophageal reflux

28.
The most common cause of acute hepatitis outbreaks in India is:

(a) Hepatitis E
(b) Hepatitis C
(c) Hepatitis B
(d) Hepatitis A

29.
Spontaneous oesophageal perforation after a bout of forceful vomiting or retching is
characteristically seen in which one of the following conditions?

(a) Menetrier?s disease
(b) Boerhaave?s syndrome
(c) Achalasia of the oesophagus
(d) Barrett?s oesophagus
30.
Eradication of helicobacter pylori infection may prove beneficial in the following extra-
gastric disorders EXCEPT:

(a) Unexplained vitamin B12 deficiency
(b) Idiopathic thrombocytopenic purpura
(c) Iron deficiency anaemia without gastro-intestinal bleeding
(d) Acute glomerulonephritis

31.
What is the target blood Hemoglobin level when treating anaemia in Stage-4 chronic kidney
disease patient?
(a) 8 to 10 gm/dL
(b) 10 to 12 gm/dL
(c) 12 to 13 gm/dL
(d) 13 to 14 gm/dL
32.
Which one of the following complications of chronic kidney disease is observed in patients
with low parathyroid hormone levels?
(a) Adynamic bone disease
(b) Tumoral calcinosis
(c) Osteitis fibrosa cystic
(d) Calciphylaxis
33.
All of the following statements about adult polycystic kidney disease are true, EXCEPT:
(a) It is inherited as autosomal dominant trait
(b) It is usually associated with marked proteinuria
(c) It is usually associated with systematic hypertension, from young age
(d) It is associated with Berry aneurysm
34.
Consider the following statements in relation to an adult patient:

1. Oliguria is defined as passage of urine less than 300 ml per day
2. Anuria is said to exist when less than 50 ml urine is passed per day
3. Polyuria is defined as urine volume in excess of 3 litre per day

Which of the statements given above are correct?
(a) 1 and 3 only
(b) 2 and 3 only
(c) 1 and 2 only
(d) 1, 2 and 3

35.
White cell casts in urine examination are strongly suggestive of:
(a) Nephritis
(b) Pyelonephritis
(c) Renal stone disease
(d) Papillitis
36.
A 13-year old boy presents with hematuria, oliguria, edema and hypertension. He has history
of sore throat two weeks prior to presentation. Laboratory investigations are remarkable for
low C
3
and increased titres of ASO and antiDNase. Which one of the following statements is
NOT correct about management for this condition?

(a) Renal biopsy is rarely required for making diagnosis
(b) Antibiotic treatment is given for streptococcal infection
(c) Treatment is largely supportive
(d) Immunosuppressants are to be used for crescentic glomerulonephritis
37.
Which one of the following statements is NOT correct regarding the risk factors for
nephrolithiasis?

(a) Higher dietary calcium increases risk
(b) Higher animal protein intake increases risk
(c) Higher dietary potassium intake lowers risk
(d) Lower urine citrate increases the risk
38.
A 60-year old patient is admitted in emergency with seizure, aphasia and altered sensorium.
He has history of fever and headache for the preceding 3 days. The CSF examination is
unremarkable. What would be the probable diagnosis?
(a) Multiple sclerosis
(b) Tubercular meningitis
(c) Pyogenic (bacterial) meningitis
(d) Viral encephalitis
39.
Drug of choice for the treatment of Trigeminal Neuralgia is:
(a) Carbamazepine
(b) Aceclofenac
(c) Thiamine
(d) Prednisdone
40.
Constructional skills is a function of which lobe?
(a) Dominant temporal lobe
(b) Non dominant parietal lobe
(c) Frontal lobe
(d) Non dominant temporal lobe

41.
Vertical gaze palsy with convergence retraction nystagmus is seen in:
(a) Weber syndrome
(b) Millard Gubler syndrome
(c) Claude syndrome
(d) Parinaud syndrome
42.
A patient on looking forward was found to have his right eye deviated downwards and
outwards with pupil dilated. He is suffering from:
(a) Left 3
rd
nerve palsy
(b) Left 6
th
nerve palsy
(c) Right 4
th
nerve palsy
(d) Right 3
rd
nerve palsy
43.
All of the following are features of Cauda Equina syndrome EXCEPT:

(a) Low back pain
(b) Lower limb areflexia
(c) Loss of bladder function
(d) Extensor plantar response

44.
A 68-year old male presented in OPD with complaints of progressive small handwriting. On
examination, he had resting tremor, bradykinesia, rigidity and postural instability. This
patient is suffering most likely from:

(a) Parkinson?s disease
(b) Vascular dementia
(c) Alzheimer?s disease
(d) Frunto temporal dementia
45.
An elderly patient presents with abnormal gait. He has a wide base freezing gait with
imbalance, comprising short strides, shuffling along the floor and difficulty with starts and
turns. Heal-Shin test is normal. Which one of the following is most likely disorder?
(a) Cerebellar ataxia
(b) Sensory ataxia
(c) Frontal gait disorder
(d) Parkinson?s disease
46.
Cushing reflex, seen in conditions of raised intra cranial pressure includes all of the following
EXCEPT:
(a) Bradycardia
(b) Hypertension
(c) Irregular respiration
(d) Hypothermia

47.
A 35 year old man presents with history of low grade fever and headache for last five weeks.
Clinical examination is remarkable for signs of meningismus. He undergoes lumbar puncture
(LP) and a day later reports worsening of headache. Which one of the following features is
NOT consistent with diagnosis of ?Post-LP? headache?

(a) Post-LP headache usually begins within 48 hours
(b) Post-LP headache worsens in sitting position
(c) Post-LP headache is most severe upon waking up
(d) Post-LP headache may improve with caffeine intake
48.
Which one of the following investigations is NOT appropriate in a case of recurrent
thrombosis?
(a) Antiphospholipid antibodies
(b) Bcr-Abl assay
(c) Protein C and S assays
(d) Antithrombin level
49.
All of the following are causes of reactive thrombocytosis, EXCEPT:
(a) Hemolytic anemia
(b) Megaloblastic anemia
(c) Post splenectomy
(d) Chronic inflammatory disorders


50.
Cryoprecipitate cannot be used for treatment of which one of the following conditions?
(a) Von Willebrand disease
(b) Hypofibrinogenemia
(c) Hemophilia?B
(d) Hemphilia?A
51.
Pyruvate kinase deficiency results in deficiency of ATP production and a chronic haemolytic
anaemia. The disorder is inherited as an:

(a) Autosomal recessive trait
(b) X-linked recessive trait
(c) Autosomal dominant trait
(d) X-linked dominant trait
52.
Sickle cell disease may be associated with any of the following EXCEPT:

(a) Acute chest syndrome
(b) Aplastic crisis
(c) Splenic sequestration crisis
(d) Conn?s syndrome

53.
Fever with splenomegaly and lymphadenopathy can be seen in
(a) Infectious mononucleosis
(b) Chronic leukaemia
(c) Both infectious mononucleosis and chronic leukaemia
(d) Neither infectious mononucleosis nor chronic leukaemia
54.
Every individual carries four alpha gene alleles. Deletion of three alleles leads to
development of:
(a) Haemoglobin X disease
(b) Haemoglobin H disease
(c) Haemoglobin F disease
(d) Hydrops fetalis
55.
Which one of the following is NOT true regarding Chronic Myeloid Leukaemia (CML)?
(a) It is a clonal malignancy of haematopoetic stem
(b) Risk of developing CML is increased in monozygotic turns
(c) CML is defined by presence of BCR-ABL 1 fusion gene
(d) 10 year survival with TKI therapy is 85%









56.
Diagnostic criteria of multiple myeloma includes which of the following?

1. Increased malignant plasma cells in the bone marrow
2. Serum and/or urinary M protein
3. Skeletal lytic lesions
4. Skeletal blastic lesions

Select the correct answer using the code given below:
(a) 1, 2 and 3
(b) 1, 3 and 4
(c) 1, 2 and 4
(d) 2, 3 and 4
57.
The most predominant type of serum immunoglobulin involved in patients with plasma cell
disorder is:
(a) Ig G
(b) Ig A
(c) Ig D
(d) Light chains
58.
Which one of the following statements is NOT true regarding the thyroid function/disorder in
pregnancy?
(a) There is an increase in the metabolism of thyroxine by placenta
(b) Hyperemesis gravidarum may be associated with thyrotoxicosis in pregnancy
(c) Levothyroxine replacement therapy dose should be decreased by 30?50% early in
pregnancy
(d) Impaired cognitive impairment may be seen in the offspring of mother with
subclinical hypothyroidism

59.
Which one of the following is NOT the criterion for the diagnosis of Diabetes mellitus?
(a) Symptoms of diabetes and random plasma sugar concentration ? 200 mg/dL
(b) HbA1c ? 6.5 %
(c) 2-hours plasma glucose ? 200 mg/dL during an oral glucose tolerance test
(d) Fasting plasma glucose ? 110 mg/dL
60.
A young lady presents in outdoor clinic with complaints of menstrual irregularity, weight
gain, hair loss, tiredness and weakness. What will be the investigation of choice?
(a) FSH & LH level
(b) Estrogen level
(c) Free T
3
and free T
4
level
(d) Free T
4
and TSH level







61.
Consider the following statements about erectile dysfunction in diabetic males:

1. It affects 60 % of males
2. Its common cause is an underlying neuro-vascular pathology
3. It may be aggravated by beta-adrenergic agonist drugs
4. Endocrine disorders like hyperprolactinemia may cause it

Which of the statements given above are true?
(a) 1 and 2
(b) 2 and 3
(c) 2 and 4
(d) 3 and 4
62.
All of the following can cause Hyperprolactinemia, EXCEPT:
(a) Craniopharyngioma
(b) Hyperthyroidism
(c) Stress
(d) Chronic renal failure
63.
All of the following can be seen in a case of thyrotoxicosis, EXCEPT:
(a) Atrial fibrillation
(b) Periodic paralysis
(c) Deafness
(d) Osteoporosis
64.
Which one of the following hormones acts via the receptor tyrosine kinase?
(a) FSH
(b) TSH
(c) IGF-1
(d) PTH
65.
Consider the following disorders as constituent of Multiple Endocrine Neoplasia Type 2a
(MEN 2a):

1. Primary Hyperparathyroidism
2. Pituitary tumours
3. Medullary carcinoma of thyroid
4. Pheochromocytoma

Which of the above are correct?:

(a) 1, 2 and 4
(b) 1, 2 and 3
(c) 1, 3 and 4
(d) 2, 3 and 4




FirstRanker.com - FirstRanker's Choice
Combined Medical Services Examination-2020
Paper-I
1.
A chronic alcoholic develops a paroxysm of palpitations after alcohol binge. Which of the
following Arrhythmia is most likely?
(a) Ventricular fibrillation
(b) Ventricular premature complex
(c) Atrial flutter
(d) Atrial fibrillation
2.
An elderly-man with history of Diabetes mellitus and Coronary Artery Disease comes for
follow-up, with complaints of muscle pains. Which one of the following drugs could be the
most likely cause?
(a) Aspirin
(b) Glimepiride
(c) Enalapril
(d) Atorvastatin
3.
A 40-year old lady comes to outdoor clinic with complaints of sudden onset chest pain. The
chest X-ray shows bilateral Pneumothorax. Examination reveals abnormalities of body
habitus-including long arms, legs and finger (arachnodactyly), scoliosis, high arched palate,
joint hypermobility, and a pansystolic murmur at cardiac apex. Which one of the following
diseases is most likely?
(a) Takayasu?s arteritis
(b) Raynaud?s syndrome
(c) Marfan?s syndrome
(d) Rheumatic heart disease-Mitral regurgitation
4.
All of the following are indications for treadmill testing/exercise-testing EXCEPT:
(a) To confirm the diagnosis of angina
(b) To evaluate stable angina
(c) To assess outcome after coronary revascularization
(d) To evaluate the treatment efficacy of antianginal drugs
5.
Kussmaul?s sign is present in all of the following conditions, EXCEPT:

(a) Massive Pulmonary Embolism
(b) Restrictive Cardiomyopathy
(c) Hypertrophic Cardiomyopathy
(d) Right Ventricular Infarction

6.
Which one of the following is NOT a common cause of atrial fibrillation?

(a) Mitral regurgitation
(b) Hypothyroidism
(c) Hypertension
(d) Acute myocardial infarction

7.
In an ECG recording, P wave is produced by:

(a) Atrial depolarisation
(b) Atrial repolarisation
(c) Ventricular depolarisation
(d) Ventricular repolarisation
8.
Which one of the following modalities is NOT used in the treatment of ventricular
tachycardia?

(a) DC Cardioversion
(b) Injection Lignocaine
(c) Injection Amiodarone
(d) Injection Adenosine
9.
Which one of the following drugs is a Direct Renin Inhibitor?

(a) Benedipine
(b) Azilsartan
(c) Aliskiren
(d) Lisinopril
10.
All of the following are causes of systolic hypertension with wide pulse pressure EXCEPT:

(a) Aortic regurgitation
(b) Thyrotoxicosis
(c) Patent ductus arteriosus
(d) Aortic stenosis
11.
Signet-ring sign on CT-chest is suggestive of:
(a) Bronchiectasis
(b) Active alveolitis
(c) Aspergilloma
(d) Sarcoidosis
12.
Oral corticosteroids are best introduced in the treatment of chronic bronchial asthma when it
is:
(a) Mild persistent
(b) Moderate persistent
(c) Severe persistent
(d) Very severe persistent
13.
An 18-year old male has insulin dependent diabetes, with malabsorption syndrome and
bilateral upper lobe bronchiectasis. A diagnostic work-up will include all EXCEPT:
(a) 24 hour fecal fat-estimation
(b) CT-chest
(c) Lung biopsy
(d) DNA sequencing study
14.
Which one of the following antigens is commonly associated with causation of Farmer?s
lung?
(a) Penicillium
(b) Actinomycetes
(c) Aspergillus
(d) Candida
15.
The Gene X-pert test used for MTB detection has the additional advantage of detection of
which of the following?

(a) INH resistance
(b) Rifampicin resistance
(c) Multi drug resistance
(d) Ethambutol resistance
16.
The most common cause of chronic type II respiratory failure is:
(a) Severe pneumonia
(b) Severe COPD
(c) Severe bronchial asthma
(d) Severe pulmonary thromboembolism
17.
The most frequent symptom in respiratory diseases is:
(a) Breathlessness
(b) Chest pain
(c) Hemoptysis
(d) Cough
18.
A 26-year old young lady attends medical emergency and is labelled as acute severe asthma
by the emergency physician. Which one of the following is unlikely to be a part of the
prescription?

(a) Albuterol
(b) Anti-leukotrienes
(c) Aminophylline
(d) Magnesium sulfate
19.
Consider the following statements with regard to respiratory examination:

1. Change in note, when patient phonates ?EEE? (Egophony) is characteristic of
interstitial fibrosis
2. Whispered pectoriloquy is characteristic of lung consolidation
3. Monophonic wheeze is characteristic of asthma
4. Hyper-resonant note on percussion is characteristic of pnemothorax

Which of the above statements are correct?
(a) 1 and 3
(b) 2 and 4 only
(c) 1, 2 and 4
(d) 2, 3 and 4
20.
All of the following are the causes of exudative Pleural Effusion, EXCEPT:
(a) SVC obstruction
(b) Fungal infection
(c) SLE
(d) Meig syndrome
21.
Which one of the following is NOT true about Ghrelin?
(a) It stimulates appetite
(b) It decreases gastric emptying
(c) It increases acid secretion
(d) Fasting increases its secretion
22.
Fecal elastase test is used for diagnosing which one of the following conditions?
(a) Lactose intolerance
(b) Mucosal inflammation
(c) Bile acid secretory defects
(d) Pancreatic dysfunction
23.
The recent classification system for listing a patient as a candidate for liver transplantation is:
(a) Child Pugh score
(b) APACHE score
(c) MELD score
(d) Metavir score
24.
The blood supply of liver consists of:
(a) 50 % hepatic artery and 50 % portal vein
(b) 80 % portal vein and 20 % hepatic artery
(c) 80 % hepatic artery and 20 % portal vein
(d) 70 % hepatic artery and 30 % portal vein
25.
All of the following conditions are associated with Glomerulonephritis with low complement
level, EXCEPT:
(a) Subacute bacterial endocarditis
(b) IgA nephropathy
(c) Systemic lupus erythematosus
(d) Cryoglobulinaemia













26.
Consider the following statements with regard to oral aphthous ulcers:

1. They are superficial and painful
2. They may occur in women just prior to menstruation
3. Oral glucocorticoids may be needed for the treatment of severe, recurrent cases
4. They are pre-malignant and progress over few years to squamous cell carcinoma of
the oral cavity

Which of the above statements are correct?

(a) 1 and 4 only
(b) 2 and 3 only
(c) 1, 2 and 3 only
(d) 1, 2, 3 and 4
27.
Which one of the following statements about Barrett?s Oesophagus is NOT correct?

(a) It is a pre-malignant condition
(b) Normal squamous cells lining the lower oesophagus are replaced by columnar
cells
(c) Normal columnar cells lining the lower oesophagus are replaced by squamous
cells
(d) It is an adaptive response to chronic gastro-oesophageal reflux

28.
The most common cause of acute hepatitis outbreaks in India is:

(a) Hepatitis E
(b) Hepatitis C
(c) Hepatitis B
(d) Hepatitis A

29.
Spontaneous oesophageal perforation after a bout of forceful vomiting or retching is
characteristically seen in which one of the following conditions?

(a) Menetrier?s disease
(b) Boerhaave?s syndrome
(c) Achalasia of the oesophagus
(d) Barrett?s oesophagus
30.
Eradication of helicobacter pylori infection may prove beneficial in the following extra-
gastric disorders EXCEPT:

(a) Unexplained vitamin B12 deficiency
(b) Idiopathic thrombocytopenic purpura
(c) Iron deficiency anaemia without gastro-intestinal bleeding
(d) Acute glomerulonephritis

31.
What is the target blood Hemoglobin level when treating anaemia in Stage-4 chronic kidney
disease patient?
(a) 8 to 10 gm/dL
(b) 10 to 12 gm/dL
(c) 12 to 13 gm/dL
(d) 13 to 14 gm/dL
32.
Which one of the following complications of chronic kidney disease is observed in patients
with low parathyroid hormone levels?
(a) Adynamic bone disease
(b) Tumoral calcinosis
(c) Osteitis fibrosa cystic
(d) Calciphylaxis
33.
All of the following statements about adult polycystic kidney disease are true, EXCEPT:
(a) It is inherited as autosomal dominant trait
(b) It is usually associated with marked proteinuria
(c) It is usually associated with systematic hypertension, from young age
(d) It is associated with Berry aneurysm
34.
Consider the following statements in relation to an adult patient:

1. Oliguria is defined as passage of urine less than 300 ml per day
2. Anuria is said to exist when less than 50 ml urine is passed per day
3. Polyuria is defined as urine volume in excess of 3 litre per day

Which of the statements given above are correct?
(a) 1 and 3 only
(b) 2 and 3 only
(c) 1 and 2 only
(d) 1, 2 and 3

35.
White cell casts in urine examination are strongly suggestive of:
(a) Nephritis
(b) Pyelonephritis
(c) Renal stone disease
(d) Papillitis
36.
A 13-year old boy presents with hematuria, oliguria, edema and hypertension. He has history
of sore throat two weeks prior to presentation. Laboratory investigations are remarkable for
low C
3
and increased titres of ASO and antiDNase. Which one of the following statements is
NOT correct about management for this condition?

(a) Renal biopsy is rarely required for making diagnosis
(b) Antibiotic treatment is given for streptococcal infection
(c) Treatment is largely supportive
(d) Immunosuppressants are to be used for crescentic glomerulonephritis
37.
Which one of the following statements is NOT correct regarding the risk factors for
nephrolithiasis?

(a) Higher dietary calcium increases risk
(b) Higher animal protein intake increases risk
(c) Higher dietary potassium intake lowers risk
(d) Lower urine citrate increases the risk
38.
A 60-year old patient is admitted in emergency with seizure, aphasia and altered sensorium.
He has history of fever and headache for the preceding 3 days. The CSF examination is
unremarkable. What would be the probable diagnosis?
(a) Multiple sclerosis
(b) Tubercular meningitis
(c) Pyogenic (bacterial) meningitis
(d) Viral encephalitis
39.
Drug of choice for the treatment of Trigeminal Neuralgia is:
(a) Carbamazepine
(b) Aceclofenac
(c) Thiamine
(d) Prednisdone
40.
Constructional skills is a function of which lobe?
(a) Dominant temporal lobe
(b) Non dominant parietal lobe
(c) Frontal lobe
(d) Non dominant temporal lobe

41.
Vertical gaze palsy with convergence retraction nystagmus is seen in:
(a) Weber syndrome
(b) Millard Gubler syndrome
(c) Claude syndrome
(d) Parinaud syndrome
42.
A patient on looking forward was found to have his right eye deviated downwards and
outwards with pupil dilated. He is suffering from:
(a) Left 3
rd
nerve palsy
(b) Left 6
th
nerve palsy
(c) Right 4
th
nerve palsy
(d) Right 3
rd
nerve palsy
43.
All of the following are features of Cauda Equina syndrome EXCEPT:

(a) Low back pain
(b) Lower limb areflexia
(c) Loss of bladder function
(d) Extensor plantar response

44.
A 68-year old male presented in OPD with complaints of progressive small handwriting. On
examination, he had resting tremor, bradykinesia, rigidity and postural instability. This
patient is suffering most likely from:

(a) Parkinson?s disease
(b) Vascular dementia
(c) Alzheimer?s disease
(d) Frunto temporal dementia
45.
An elderly patient presents with abnormal gait. He has a wide base freezing gait with
imbalance, comprising short strides, shuffling along the floor and difficulty with starts and
turns. Heal-Shin test is normal. Which one of the following is most likely disorder?
(a) Cerebellar ataxia
(b) Sensory ataxia
(c) Frontal gait disorder
(d) Parkinson?s disease
46.
Cushing reflex, seen in conditions of raised intra cranial pressure includes all of the following
EXCEPT:
(a) Bradycardia
(b) Hypertension
(c) Irregular respiration
(d) Hypothermia

47.
A 35 year old man presents with history of low grade fever and headache for last five weeks.
Clinical examination is remarkable for signs of meningismus. He undergoes lumbar puncture
(LP) and a day later reports worsening of headache. Which one of the following features is
NOT consistent with diagnosis of ?Post-LP? headache?

(a) Post-LP headache usually begins within 48 hours
(b) Post-LP headache worsens in sitting position
(c) Post-LP headache is most severe upon waking up
(d) Post-LP headache may improve with caffeine intake
48.
Which one of the following investigations is NOT appropriate in a case of recurrent
thrombosis?
(a) Antiphospholipid antibodies
(b) Bcr-Abl assay
(c) Protein C and S assays
(d) Antithrombin level
49.
All of the following are causes of reactive thrombocytosis, EXCEPT:
(a) Hemolytic anemia
(b) Megaloblastic anemia
(c) Post splenectomy
(d) Chronic inflammatory disorders


50.
Cryoprecipitate cannot be used for treatment of which one of the following conditions?
(a) Von Willebrand disease
(b) Hypofibrinogenemia
(c) Hemophilia?B
(d) Hemphilia?A
51.
Pyruvate kinase deficiency results in deficiency of ATP production and a chronic haemolytic
anaemia. The disorder is inherited as an:

(a) Autosomal recessive trait
(b) X-linked recessive trait
(c) Autosomal dominant trait
(d) X-linked dominant trait
52.
Sickle cell disease may be associated with any of the following EXCEPT:

(a) Acute chest syndrome
(b) Aplastic crisis
(c) Splenic sequestration crisis
(d) Conn?s syndrome

53.
Fever with splenomegaly and lymphadenopathy can be seen in
(a) Infectious mononucleosis
(b) Chronic leukaemia
(c) Both infectious mononucleosis and chronic leukaemia
(d) Neither infectious mononucleosis nor chronic leukaemia
54.
Every individual carries four alpha gene alleles. Deletion of three alleles leads to
development of:
(a) Haemoglobin X disease
(b) Haemoglobin H disease
(c) Haemoglobin F disease
(d) Hydrops fetalis
55.
Which one of the following is NOT true regarding Chronic Myeloid Leukaemia (CML)?
(a) It is a clonal malignancy of haematopoetic stem
(b) Risk of developing CML is increased in monozygotic turns
(c) CML is defined by presence of BCR-ABL 1 fusion gene
(d) 10 year survival with TKI therapy is 85%









56.
Diagnostic criteria of multiple myeloma includes which of the following?

1. Increased malignant plasma cells in the bone marrow
2. Serum and/or urinary M protein
3. Skeletal lytic lesions
4. Skeletal blastic lesions

Select the correct answer using the code given below:
(a) 1, 2 and 3
(b) 1, 3 and 4
(c) 1, 2 and 4
(d) 2, 3 and 4
57.
The most predominant type of serum immunoglobulin involved in patients with plasma cell
disorder is:
(a) Ig G
(b) Ig A
(c) Ig D
(d) Light chains
58.
Which one of the following statements is NOT true regarding the thyroid function/disorder in
pregnancy?
(a) There is an increase in the metabolism of thyroxine by placenta
(b) Hyperemesis gravidarum may be associated with thyrotoxicosis in pregnancy
(c) Levothyroxine replacement therapy dose should be decreased by 30?50% early in
pregnancy
(d) Impaired cognitive impairment may be seen in the offspring of mother with
subclinical hypothyroidism

59.
Which one of the following is NOT the criterion for the diagnosis of Diabetes mellitus?
(a) Symptoms of diabetes and random plasma sugar concentration ? 200 mg/dL
(b) HbA1c ? 6.5 %
(c) 2-hours plasma glucose ? 200 mg/dL during an oral glucose tolerance test
(d) Fasting plasma glucose ? 110 mg/dL
60.
A young lady presents in outdoor clinic with complaints of menstrual irregularity, weight
gain, hair loss, tiredness and weakness. What will be the investigation of choice?
(a) FSH & LH level
(b) Estrogen level
(c) Free T
3
and free T
4
level
(d) Free T
4
and TSH level







61.
Consider the following statements about erectile dysfunction in diabetic males:

1. It affects 60 % of males
2. Its common cause is an underlying neuro-vascular pathology
3. It may be aggravated by beta-adrenergic agonist drugs
4. Endocrine disorders like hyperprolactinemia may cause it

Which of the statements given above are true?
(a) 1 and 2
(b) 2 and 3
(c) 2 and 4
(d) 3 and 4
62.
All of the following can cause Hyperprolactinemia, EXCEPT:
(a) Craniopharyngioma
(b) Hyperthyroidism
(c) Stress
(d) Chronic renal failure
63.
All of the following can be seen in a case of thyrotoxicosis, EXCEPT:
(a) Atrial fibrillation
(b) Periodic paralysis
(c) Deafness
(d) Osteoporosis
64.
Which one of the following hormones acts via the receptor tyrosine kinase?
(a) FSH
(b) TSH
(c) IGF-1
(d) PTH
65.
Consider the following disorders as constituent of Multiple Endocrine Neoplasia Type 2a
(MEN 2a):

1. Primary Hyperparathyroidism
2. Pituitary tumours
3. Medullary carcinoma of thyroid
4. Pheochromocytoma

Which of the above are correct?:

(a) 1, 2 and 4
(b) 1, 2 and 3
(c) 1, 3 and 4
(d) 2, 3 and 4




66.
Consider the following statements with regard to Graves? ophthalmopathy :

1. Proptosis is often asymmetric and can even appear to be unilateral
2. It is a clinical diagnosis
3. Worsening of symptoms upon glucocorticoid withdrawal is common
4. Radiation therapy is very effective in treatment

Which of the above statements are correct?

(a) 1 and 2 only
(b) 1, 2 and 3
(c) 2, 3 and 4
(d) 1 and 4 only
67.
All of the following are common aetiologies of delirium EXCEPT:
(a) Hyperthyroidism
(b) Hypothyroidism
(c) Hyperparathyroidism
(d) Hypoparathyroidism
68.
Key anthropometric measurements important for evaluating the degree of obesity are:

(a) Weight, height and waist circumference
(b) Weight, height and hip circumference
(c) Weight, height and mid-thigh circumference
(d) Weight, height and chest circumference
69.
What is the minimum BMI (Body Mass Index) recommended for considering adjuvant
pharmacologic treatment in obesity with comorbidity?
(a) 23 kg/m
2

(b) 25 kg/m
2

(c) 27 kg/m
2

(d) 30 kg/m
2

70.
A young medical student while working in HIV ward got pricked accidently on the finger by
a syringe needle which was used to draw blood from a patient. Patient?s infection status is not
known. Which one of the following therapeutic interventions will be the best interest of this
student?
(a) To start Dolultegravir immediately
(b) To wash wound with soap and water and start Emtricitabine
(c) To give Dolutegravir + Tenofovir + Emtricitabine
(d) To give Dolutegravir + Tenofovir + Emtricitabine + HBV Immunoglobulin
71.
Cryptococcal infection is acquired through:
(a) Inhalation route
(b) Bite of Culex mosquito
(c) Direct skin contact
(d) Ingestion of spores

FirstRanker.com - FirstRanker's Choice
Combined Medical Services Examination-2020
Paper-I
1.
A chronic alcoholic develops a paroxysm of palpitations after alcohol binge. Which of the
following Arrhythmia is most likely?
(a) Ventricular fibrillation
(b) Ventricular premature complex
(c) Atrial flutter
(d) Atrial fibrillation
2.
An elderly-man with history of Diabetes mellitus and Coronary Artery Disease comes for
follow-up, with complaints of muscle pains. Which one of the following drugs could be the
most likely cause?
(a) Aspirin
(b) Glimepiride
(c) Enalapril
(d) Atorvastatin
3.
A 40-year old lady comes to outdoor clinic with complaints of sudden onset chest pain. The
chest X-ray shows bilateral Pneumothorax. Examination reveals abnormalities of body
habitus-including long arms, legs and finger (arachnodactyly), scoliosis, high arched palate,
joint hypermobility, and a pansystolic murmur at cardiac apex. Which one of the following
diseases is most likely?
(a) Takayasu?s arteritis
(b) Raynaud?s syndrome
(c) Marfan?s syndrome
(d) Rheumatic heart disease-Mitral regurgitation
4.
All of the following are indications for treadmill testing/exercise-testing EXCEPT:
(a) To confirm the diagnosis of angina
(b) To evaluate stable angina
(c) To assess outcome after coronary revascularization
(d) To evaluate the treatment efficacy of antianginal drugs
5.
Kussmaul?s sign is present in all of the following conditions, EXCEPT:

(a) Massive Pulmonary Embolism
(b) Restrictive Cardiomyopathy
(c) Hypertrophic Cardiomyopathy
(d) Right Ventricular Infarction

6.
Which one of the following is NOT a common cause of atrial fibrillation?

(a) Mitral regurgitation
(b) Hypothyroidism
(c) Hypertension
(d) Acute myocardial infarction

7.
In an ECG recording, P wave is produced by:

(a) Atrial depolarisation
(b) Atrial repolarisation
(c) Ventricular depolarisation
(d) Ventricular repolarisation
8.
Which one of the following modalities is NOT used in the treatment of ventricular
tachycardia?

(a) DC Cardioversion
(b) Injection Lignocaine
(c) Injection Amiodarone
(d) Injection Adenosine
9.
Which one of the following drugs is a Direct Renin Inhibitor?

(a) Benedipine
(b) Azilsartan
(c) Aliskiren
(d) Lisinopril
10.
All of the following are causes of systolic hypertension with wide pulse pressure EXCEPT:

(a) Aortic regurgitation
(b) Thyrotoxicosis
(c) Patent ductus arteriosus
(d) Aortic stenosis
11.
Signet-ring sign on CT-chest is suggestive of:
(a) Bronchiectasis
(b) Active alveolitis
(c) Aspergilloma
(d) Sarcoidosis
12.
Oral corticosteroids are best introduced in the treatment of chronic bronchial asthma when it
is:
(a) Mild persistent
(b) Moderate persistent
(c) Severe persistent
(d) Very severe persistent
13.
An 18-year old male has insulin dependent diabetes, with malabsorption syndrome and
bilateral upper lobe bronchiectasis. A diagnostic work-up will include all EXCEPT:
(a) 24 hour fecal fat-estimation
(b) CT-chest
(c) Lung biopsy
(d) DNA sequencing study
14.
Which one of the following antigens is commonly associated with causation of Farmer?s
lung?
(a) Penicillium
(b) Actinomycetes
(c) Aspergillus
(d) Candida
15.
The Gene X-pert test used for MTB detection has the additional advantage of detection of
which of the following?

(a) INH resistance
(b) Rifampicin resistance
(c) Multi drug resistance
(d) Ethambutol resistance
16.
The most common cause of chronic type II respiratory failure is:
(a) Severe pneumonia
(b) Severe COPD
(c) Severe bronchial asthma
(d) Severe pulmonary thromboembolism
17.
The most frequent symptom in respiratory diseases is:
(a) Breathlessness
(b) Chest pain
(c) Hemoptysis
(d) Cough
18.
A 26-year old young lady attends medical emergency and is labelled as acute severe asthma
by the emergency physician. Which one of the following is unlikely to be a part of the
prescription?

(a) Albuterol
(b) Anti-leukotrienes
(c) Aminophylline
(d) Magnesium sulfate
19.
Consider the following statements with regard to respiratory examination:

1. Change in note, when patient phonates ?EEE? (Egophony) is characteristic of
interstitial fibrosis
2. Whispered pectoriloquy is characteristic of lung consolidation
3. Monophonic wheeze is characteristic of asthma
4. Hyper-resonant note on percussion is characteristic of pnemothorax

Which of the above statements are correct?
(a) 1 and 3
(b) 2 and 4 only
(c) 1, 2 and 4
(d) 2, 3 and 4
20.
All of the following are the causes of exudative Pleural Effusion, EXCEPT:
(a) SVC obstruction
(b) Fungal infection
(c) SLE
(d) Meig syndrome
21.
Which one of the following is NOT true about Ghrelin?
(a) It stimulates appetite
(b) It decreases gastric emptying
(c) It increases acid secretion
(d) Fasting increases its secretion
22.
Fecal elastase test is used for diagnosing which one of the following conditions?
(a) Lactose intolerance
(b) Mucosal inflammation
(c) Bile acid secretory defects
(d) Pancreatic dysfunction
23.
The recent classification system for listing a patient as a candidate for liver transplantation is:
(a) Child Pugh score
(b) APACHE score
(c) MELD score
(d) Metavir score
24.
The blood supply of liver consists of:
(a) 50 % hepatic artery and 50 % portal vein
(b) 80 % portal vein and 20 % hepatic artery
(c) 80 % hepatic artery and 20 % portal vein
(d) 70 % hepatic artery and 30 % portal vein
25.
All of the following conditions are associated with Glomerulonephritis with low complement
level, EXCEPT:
(a) Subacute bacterial endocarditis
(b) IgA nephropathy
(c) Systemic lupus erythematosus
(d) Cryoglobulinaemia













26.
Consider the following statements with regard to oral aphthous ulcers:

1. They are superficial and painful
2. They may occur in women just prior to menstruation
3. Oral glucocorticoids may be needed for the treatment of severe, recurrent cases
4. They are pre-malignant and progress over few years to squamous cell carcinoma of
the oral cavity

Which of the above statements are correct?

(a) 1 and 4 only
(b) 2 and 3 only
(c) 1, 2 and 3 only
(d) 1, 2, 3 and 4
27.
Which one of the following statements about Barrett?s Oesophagus is NOT correct?

(a) It is a pre-malignant condition
(b) Normal squamous cells lining the lower oesophagus are replaced by columnar
cells
(c) Normal columnar cells lining the lower oesophagus are replaced by squamous
cells
(d) It is an adaptive response to chronic gastro-oesophageal reflux

28.
The most common cause of acute hepatitis outbreaks in India is:

(a) Hepatitis E
(b) Hepatitis C
(c) Hepatitis B
(d) Hepatitis A

29.
Spontaneous oesophageal perforation after a bout of forceful vomiting or retching is
characteristically seen in which one of the following conditions?

(a) Menetrier?s disease
(b) Boerhaave?s syndrome
(c) Achalasia of the oesophagus
(d) Barrett?s oesophagus
30.
Eradication of helicobacter pylori infection may prove beneficial in the following extra-
gastric disorders EXCEPT:

(a) Unexplained vitamin B12 deficiency
(b) Idiopathic thrombocytopenic purpura
(c) Iron deficiency anaemia without gastro-intestinal bleeding
(d) Acute glomerulonephritis

31.
What is the target blood Hemoglobin level when treating anaemia in Stage-4 chronic kidney
disease patient?
(a) 8 to 10 gm/dL
(b) 10 to 12 gm/dL
(c) 12 to 13 gm/dL
(d) 13 to 14 gm/dL
32.
Which one of the following complications of chronic kidney disease is observed in patients
with low parathyroid hormone levels?
(a) Adynamic bone disease
(b) Tumoral calcinosis
(c) Osteitis fibrosa cystic
(d) Calciphylaxis
33.
All of the following statements about adult polycystic kidney disease are true, EXCEPT:
(a) It is inherited as autosomal dominant trait
(b) It is usually associated with marked proteinuria
(c) It is usually associated with systematic hypertension, from young age
(d) It is associated with Berry aneurysm
34.
Consider the following statements in relation to an adult patient:

1. Oliguria is defined as passage of urine less than 300 ml per day
2. Anuria is said to exist when less than 50 ml urine is passed per day
3. Polyuria is defined as urine volume in excess of 3 litre per day

Which of the statements given above are correct?
(a) 1 and 3 only
(b) 2 and 3 only
(c) 1 and 2 only
(d) 1, 2 and 3

35.
White cell casts in urine examination are strongly suggestive of:
(a) Nephritis
(b) Pyelonephritis
(c) Renal stone disease
(d) Papillitis
36.
A 13-year old boy presents with hematuria, oliguria, edema and hypertension. He has history
of sore throat two weeks prior to presentation. Laboratory investigations are remarkable for
low C
3
and increased titres of ASO and antiDNase. Which one of the following statements is
NOT correct about management for this condition?

(a) Renal biopsy is rarely required for making diagnosis
(b) Antibiotic treatment is given for streptococcal infection
(c) Treatment is largely supportive
(d) Immunosuppressants are to be used for crescentic glomerulonephritis
37.
Which one of the following statements is NOT correct regarding the risk factors for
nephrolithiasis?

(a) Higher dietary calcium increases risk
(b) Higher animal protein intake increases risk
(c) Higher dietary potassium intake lowers risk
(d) Lower urine citrate increases the risk
38.
A 60-year old patient is admitted in emergency with seizure, aphasia and altered sensorium.
He has history of fever and headache for the preceding 3 days. The CSF examination is
unremarkable. What would be the probable diagnosis?
(a) Multiple sclerosis
(b) Tubercular meningitis
(c) Pyogenic (bacterial) meningitis
(d) Viral encephalitis
39.
Drug of choice for the treatment of Trigeminal Neuralgia is:
(a) Carbamazepine
(b) Aceclofenac
(c) Thiamine
(d) Prednisdone
40.
Constructional skills is a function of which lobe?
(a) Dominant temporal lobe
(b) Non dominant parietal lobe
(c) Frontal lobe
(d) Non dominant temporal lobe

41.
Vertical gaze palsy with convergence retraction nystagmus is seen in:
(a) Weber syndrome
(b) Millard Gubler syndrome
(c) Claude syndrome
(d) Parinaud syndrome
42.
A patient on looking forward was found to have his right eye deviated downwards and
outwards with pupil dilated. He is suffering from:
(a) Left 3
rd
nerve palsy
(b) Left 6
th
nerve palsy
(c) Right 4
th
nerve palsy
(d) Right 3
rd
nerve palsy
43.
All of the following are features of Cauda Equina syndrome EXCEPT:

(a) Low back pain
(b) Lower limb areflexia
(c) Loss of bladder function
(d) Extensor plantar response

44.
A 68-year old male presented in OPD with complaints of progressive small handwriting. On
examination, he had resting tremor, bradykinesia, rigidity and postural instability. This
patient is suffering most likely from:

(a) Parkinson?s disease
(b) Vascular dementia
(c) Alzheimer?s disease
(d) Frunto temporal dementia
45.
An elderly patient presents with abnormal gait. He has a wide base freezing gait with
imbalance, comprising short strides, shuffling along the floor and difficulty with starts and
turns. Heal-Shin test is normal. Which one of the following is most likely disorder?
(a) Cerebellar ataxia
(b) Sensory ataxia
(c) Frontal gait disorder
(d) Parkinson?s disease
46.
Cushing reflex, seen in conditions of raised intra cranial pressure includes all of the following
EXCEPT:
(a) Bradycardia
(b) Hypertension
(c) Irregular respiration
(d) Hypothermia

47.
A 35 year old man presents with history of low grade fever and headache for last five weeks.
Clinical examination is remarkable for signs of meningismus. He undergoes lumbar puncture
(LP) and a day later reports worsening of headache. Which one of the following features is
NOT consistent with diagnosis of ?Post-LP? headache?

(a) Post-LP headache usually begins within 48 hours
(b) Post-LP headache worsens in sitting position
(c) Post-LP headache is most severe upon waking up
(d) Post-LP headache may improve with caffeine intake
48.
Which one of the following investigations is NOT appropriate in a case of recurrent
thrombosis?
(a) Antiphospholipid antibodies
(b) Bcr-Abl assay
(c) Protein C and S assays
(d) Antithrombin level
49.
All of the following are causes of reactive thrombocytosis, EXCEPT:
(a) Hemolytic anemia
(b) Megaloblastic anemia
(c) Post splenectomy
(d) Chronic inflammatory disorders


50.
Cryoprecipitate cannot be used for treatment of which one of the following conditions?
(a) Von Willebrand disease
(b) Hypofibrinogenemia
(c) Hemophilia?B
(d) Hemphilia?A
51.
Pyruvate kinase deficiency results in deficiency of ATP production and a chronic haemolytic
anaemia. The disorder is inherited as an:

(a) Autosomal recessive trait
(b) X-linked recessive trait
(c) Autosomal dominant trait
(d) X-linked dominant trait
52.
Sickle cell disease may be associated with any of the following EXCEPT:

(a) Acute chest syndrome
(b) Aplastic crisis
(c) Splenic sequestration crisis
(d) Conn?s syndrome

53.
Fever with splenomegaly and lymphadenopathy can be seen in
(a) Infectious mononucleosis
(b) Chronic leukaemia
(c) Both infectious mononucleosis and chronic leukaemia
(d) Neither infectious mononucleosis nor chronic leukaemia
54.
Every individual carries four alpha gene alleles. Deletion of three alleles leads to
development of:
(a) Haemoglobin X disease
(b) Haemoglobin H disease
(c) Haemoglobin F disease
(d) Hydrops fetalis
55.
Which one of the following is NOT true regarding Chronic Myeloid Leukaemia (CML)?
(a) It is a clonal malignancy of haematopoetic stem
(b) Risk of developing CML is increased in monozygotic turns
(c) CML is defined by presence of BCR-ABL 1 fusion gene
(d) 10 year survival with TKI therapy is 85%









56.
Diagnostic criteria of multiple myeloma includes which of the following?

1. Increased malignant plasma cells in the bone marrow
2. Serum and/or urinary M protein
3. Skeletal lytic lesions
4. Skeletal blastic lesions

Select the correct answer using the code given below:
(a) 1, 2 and 3
(b) 1, 3 and 4
(c) 1, 2 and 4
(d) 2, 3 and 4
57.
The most predominant type of serum immunoglobulin involved in patients with plasma cell
disorder is:
(a) Ig G
(b) Ig A
(c) Ig D
(d) Light chains
58.
Which one of the following statements is NOT true regarding the thyroid function/disorder in
pregnancy?
(a) There is an increase in the metabolism of thyroxine by placenta
(b) Hyperemesis gravidarum may be associated with thyrotoxicosis in pregnancy
(c) Levothyroxine replacement therapy dose should be decreased by 30?50% early in
pregnancy
(d) Impaired cognitive impairment may be seen in the offspring of mother with
subclinical hypothyroidism

59.
Which one of the following is NOT the criterion for the diagnosis of Diabetes mellitus?
(a) Symptoms of diabetes and random plasma sugar concentration ? 200 mg/dL
(b) HbA1c ? 6.5 %
(c) 2-hours plasma glucose ? 200 mg/dL during an oral glucose tolerance test
(d) Fasting plasma glucose ? 110 mg/dL
60.
A young lady presents in outdoor clinic with complaints of menstrual irregularity, weight
gain, hair loss, tiredness and weakness. What will be the investigation of choice?
(a) FSH & LH level
(b) Estrogen level
(c) Free T
3
and free T
4
level
(d) Free T
4
and TSH level







61.
Consider the following statements about erectile dysfunction in diabetic males:

1. It affects 60 % of males
2. Its common cause is an underlying neuro-vascular pathology
3. It may be aggravated by beta-adrenergic agonist drugs
4. Endocrine disorders like hyperprolactinemia may cause it

Which of the statements given above are true?
(a) 1 and 2
(b) 2 and 3
(c) 2 and 4
(d) 3 and 4
62.
All of the following can cause Hyperprolactinemia, EXCEPT:
(a) Craniopharyngioma
(b) Hyperthyroidism
(c) Stress
(d) Chronic renal failure
63.
All of the following can be seen in a case of thyrotoxicosis, EXCEPT:
(a) Atrial fibrillation
(b) Periodic paralysis
(c) Deafness
(d) Osteoporosis
64.
Which one of the following hormones acts via the receptor tyrosine kinase?
(a) FSH
(b) TSH
(c) IGF-1
(d) PTH
65.
Consider the following disorders as constituent of Multiple Endocrine Neoplasia Type 2a
(MEN 2a):

1. Primary Hyperparathyroidism
2. Pituitary tumours
3. Medullary carcinoma of thyroid
4. Pheochromocytoma

Which of the above are correct?:

(a) 1, 2 and 4
(b) 1, 2 and 3
(c) 1, 3 and 4
(d) 2, 3 and 4




66.
Consider the following statements with regard to Graves? ophthalmopathy :

1. Proptosis is often asymmetric and can even appear to be unilateral
2. It is a clinical diagnosis
3. Worsening of symptoms upon glucocorticoid withdrawal is common
4. Radiation therapy is very effective in treatment

Which of the above statements are correct?

(a) 1 and 2 only
(b) 1, 2 and 3
(c) 2, 3 and 4
(d) 1 and 4 only
67.
All of the following are common aetiologies of delirium EXCEPT:
(a) Hyperthyroidism
(b) Hypothyroidism
(c) Hyperparathyroidism
(d) Hypoparathyroidism
68.
Key anthropometric measurements important for evaluating the degree of obesity are:

(a) Weight, height and waist circumference
(b) Weight, height and hip circumference
(c) Weight, height and mid-thigh circumference
(d) Weight, height and chest circumference
69.
What is the minimum BMI (Body Mass Index) recommended for considering adjuvant
pharmacologic treatment in obesity with comorbidity?
(a) 23 kg/m
2

(b) 25 kg/m
2

(c) 27 kg/m
2

(d) 30 kg/m
2

70.
A young medical student while working in HIV ward got pricked accidently on the finger by
a syringe needle which was used to draw blood from a patient. Patient?s infection status is not
known. Which one of the following therapeutic interventions will be the best interest of this
student?
(a) To start Dolultegravir immediately
(b) To wash wound with soap and water and start Emtricitabine
(c) To give Dolutegravir + Tenofovir + Emtricitabine
(d) To give Dolutegravir + Tenofovir + Emtricitabine + HBV Immunoglobulin
71.
Cryptococcal infection is acquired through:
(a) Inhalation route
(b) Bite of Culex mosquito
(c) Direct skin contact
(d) Ingestion of spores

72.
Which one of the following is NOT appropriate treatment for Echinococcus granulosus and
hydatid disease?
(a) Albendazole 400 mg twice in a day for 3 months
(b) Diethylcarbamazine 2 mg/kg thrice in a day for 12 days
(c) PAIR (percutaneous puncture, aspiration, injection of scolicide, reaspiration)
(d) Praziquantel (20 mg/kg twice daily for 14 days)
73.
Which one of the following infections is NOT amenable to post-exposure prophylaxis with
specific immunoglobulins?
(a) Hepatitis B
(b) HIV
(c) Tetanus
(d) Rabies
74.
Echinocandins are a class of:
(a) Antiviral agents
(b) Antiparasitic agents
(c) Antitubercular agents
(d) Antifungal agents
75.
Recommended first time drug for initial treatment of tuberculosis include all EXCEPT:
(a) Isoniazid
(b) Rifampin
(c) Ethambutol
(d) Ethionamide
76.
The most frequent adverse reaction of significance among people treated for drug-susceptible
tuberculosis is:
(a) Colitis
(b) Hepatitis
(c) Cystitis
(d) Enteritis
77.
All adult patients in whom anti-tubercular treatment is being started should undergo baseline
assessment of:
(a) Complete blood count
(b) Liver function test
(c) Lipid profile
(d) Pulmonary function test
78.
A 30 year old male presents with fever, headache, anorexia, nausea, vomiting and diarrhoea.
On examination tongue is coated with abdominal tenderness, soft splenomegaly, relative
bradycardia and rose spots rash. The most likely diagnosis is:

(a) Malaria fever
(b) Enteric fever
(c) Leptospirosis
(d) Dengue fever

FirstRanker.com - FirstRanker's Choice
Combined Medical Services Examination-2020
Paper-I
1.
A chronic alcoholic develops a paroxysm of palpitations after alcohol binge. Which of the
following Arrhythmia is most likely?
(a) Ventricular fibrillation
(b) Ventricular premature complex
(c) Atrial flutter
(d) Atrial fibrillation
2.
An elderly-man with history of Diabetes mellitus and Coronary Artery Disease comes for
follow-up, with complaints of muscle pains. Which one of the following drugs could be the
most likely cause?
(a) Aspirin
(b) Glimepiride
(c) Enalapril
(d) Atorvastatin
3.
A 40-year old lady comes to outdoor clinic with complaints of sudden onset chest pain. The
chest X-ray shows bilateral Pneumothorax. Examination reveals abnormalities of body
habitus-including long arms, legs and finger (arachnodactyly), scoliosis, high arched palate,
joint hypermobility, and a pansystolic murmur at cardiac apex. Which one of the following
diseases is most likely?
(a) Takayasu?s arteritis
(b) Raynaud?s syndrome
(c) Marfan?s syndrome
(d) Rheumatic heart disease-Mitral regurgitation
4.
All of the following are indications for treadmill testing/exercise-testing EXCEPT:
(a) To confirm the diagnosis of angina
(b) To evaluate stable angina
(c) To assess outcome after coronary revascularization
(d) To evaluate the treatment efficacy of antianginal drugs
5.
Kussmaul?s sign is present in all of the following conditions, EXCEPT:

(a) Massive Pulmonary Embolism
(b) Restrictive Cardiomyopathy
(c) Hypertrophic Cardiomyopathy
(d) Right Ventricular Infarction

6.
Which one of the following is NOT a common cause of atrial fibrillation?

(a) Mitral regurgitation
(b) Hypothyroidism
(c) Hypertension
(d) Acute myocardial infarction

7.
In an ECG recording, P wave is produced by:

(a) Atrial depolarisation
(b) Atrial repolarisation
(c) Ventricular depolarisation
(d) Ventricular repolarisation
8.
Which one of the following modalities is NOT used in the treatment of ventricular
tachycardia?

(a) DC Cardioversion
(b) Injection Lignocaine
(c) Injection Amiodarone
(d) Injection Adenosine
9.
Which one of the following drugs is a Direct Renin Inhibitor?

(a) Benedipine
(b) Azilsartan
(c) Aliskiren
(d) Lisinopril
10.
All of the following are causes of systolic hypertension with wide pulse pressure EXCEPT:

(a) Aortic regurgitation
(b) Thyrotoxicosis
(c) Patent ductus arteriosus
(d) Aortic stenosis
11.
Signet-ring sign on CT-chest is suggestive of:
(a) Bronchiectasis
(b) Active alveolitis
(c) Aspergilloma
(d) Sarcoidosis
12.
Oral corticosteroids are best introduced in the treatment of chronic bronchial asthma when it
is:
(a) Mild persistent
(b) Moderate persistent
(c) Severe persistent
(d) Very severe persistent
13.
An 18-year old male has insulin dependent diabetes, with malabsorption syndrome and
bilateral upper lobe bronchiectasis. A diagnostic work-up will include all EXCEPT:
(a) 24 hour fecal fat-estimation
(b) CT-chest
(c) Lung biopsy
(d) DNA sequencing study
14.
Which one of the following antigens is commonly associated with causation of Farmer?s
lung?
(a) Penicillium
(b) Actinomycetes
(c) Aspergillus
(d) Candida
15.
The Gene X-pert test used for MTB detection has the additional advantage of detection of
which of the following?

(a) INH resistance
(b) Rifampicin resistance
(c) Multi drug resistance
(d) Ethambutol resistance
16.
The most common cause of chronic type II respiratory failure is:
(a) Severe pneumonia
(b) Severe COPD
(c) Severe bronchial asthma
(d) Severe pulmonary thromboembolism
17.
The most frequent symptom in respiratory diseases is:
(a) Breathlessness
(b) Chest pain
(c) Hemoptysis
(d) Cough
18.
A 26-year old young lady attends medical emergency and is labelled as acute severe asthma
by the emergency physician. Which one of the following is unlikely to be a part of the
prescription?

(a) Albuterol
(b) Anti-leukotrienes
(c) Aminophylline
(d) Magnesium sulfate
19.
Consider the following statements with regard to respiratory examination:

1. Change in note, when patient phonates ?EEE? (Egophony) is characteristic of
interstitial fibrosis
2. Whispered pectoriloquy is characteristic of lung consolidation
3. Monophonic wheeze is characteristic of asthma
4. Hyper-resonant note on percussion is characteristic of pnemothorax

Which of the above statements are correct?
(a) 1 and 3
(b) 2 and 4 only
(c) 1, 2 and 4
(d) 2, 3 and 4
20.
All of the following are the causes of exudative Pleural Effusion, EXCEPT:
(a) SVC obstruction
(b) Fungal infection
(c) SLE
(d) Meig syndrome
21.
Which one of the following is NOT true about Ghrelin?
(a) It stimulates appetite
(b) It decreases gastric emptying
(c) It increases acid secretion
(d) Fasting increases its secretion
22.
Fecal elastase test is used for diagnosing which one of the following conditions?
(a) Lactose intolerance
(b) Mucosal inflammation
(c) Bile acid secretory defects
(d) Pancreatic dysfunction
23.
The recent classification system for listing a patient as a candidate for liver transplantation is:
(a) Child Pugh score
(b) APACHE score
(c) MELD score
(d) Metavir score
24.
The blood supply of liver consists of:
(a) 50 % hepatic artery and 50 % portal vein
(b) 80 % portal vein and 20 % hepatic artery
(c) 80 % hepatic artery and 20 % portal vein
(d) 70 % hepatic artery and 30 % portal vein
25.
All of the following conditions are associated with Glomerulonephritis with low complement
level, EXCEPT:
(a) Subacute bacterial endocarditis
(b) IgA nephropathy
(c) Systemic lupus erythematosus
(d) Cryoglobulinaemia













26.
Consider the following statements with regard to oral aphthous ulcers:

1. They are superficial and painful
2. They may occur in women just prior to menstruation
3. Oral glucocorticoids may be needed for the treatment of severe, recurrent cases
4. They are pre-malignant and progress over few years to squamous cell carcinoma of
the oral cavity

Which of the above statements are correct?

(a) 1 and 4 only
(b) 2 and 3 only
(c) 1, 2 and 3 only
(d) 1, 2, 3 and 4
27.
Which one of the following statements about Barrett?s Oesophagus is NOT correct?

(a) It is a pre-malignant condition
(b) Normal squamous cells lining the lower oesophagus are replaced by columnar
cells
(c) Normal columnar cells lining the lower oesophagus are replaced by squamous
cells
(d) It is an adaptive response to chronic gastro-oesophageal reflux

28.
The most common cause of acute hepatitis outbreaks in India is:

(a) Hepatitis E
(b) Hepatitis C
(c) Hepatitis B
(d) Hepatitis A

29.
Spontaneous oesophageal perforation after a bout of forceful vomiting or retching is
characteristically seen in which one of the following conditions?

(a) Menetrier?s disease
(b) Boerhaave?s syndrome
(c) Achalasia of the oesophagus
(d) Barrett?s oesophagus
30.
Eradication of helicobacter pylori infection may prove beneficial in the following extra-
gastric disorders EXCEPT:

(a) Unexplained vitamin B12 deficiency
(b) Idiopathic thrombocytopenic purpura
(c) Iron deficiency anaemia without gastro-intestinal bleeding
(d) Acute glomerulonephritis

31.
What is the target blood Hemoglobin level when treating anaemia in Stage-4 chronic kidney
disease patient?
(a) 8 to 10 gm/dL
(b) 10 to 12 gm/dL
(c) 12 to 13 gm/dL
(d) 13 to 14 gm/dL
32.
Which one of the following complications of chronic kidney disease is observed in patients
with low parathyroid hormone levels?
(a) Adynamic bone disease
(b) Tumoral calcinosis
(c) Osteitis fibrosa cystic
(d) Calciphylaxis
33.
All of the following statements about adult polycystic kidney disease are true, EXCEPT:
(a) It is inherited as autosomal dominant trait
(b) It is usually associated with marked proteinuria
(c) It is usually associated with systematic hypertension, from young age
(d) It is associated with Berry aneurysm
34.
Consider the following statements in relation to an adult patient:

1. Oliguria is defined as passage of urine less than 300 ml per day
2. Anuria is said to exist when less than 50 ml urine is passed per day
3. Polyuria is defined as urine volume in excess of 3 litre per day

Which of the statements given above are correct?
(a) 1 and 3 only
(b) 2 and 3 only
(c) 1 and 2 only
(d) 1, 2 and 3

35.
White cell casts in urine examination are strongly suggestive of:
(a) Nephritis
(b) Pyelonephritis
(c) Renal stone disease
(d) Papillitis
36.
A 13-year old boy presents with hematuria, oliguria, edema and hypertension. He has history
of sore throat two weeks prior to presentation. Laboratory investigations are remarkable for
low C
3
and increased titres of ASO and antiDNase. Which one of the following statements is
NOT correct about management for this condition?

(a) Renal biopsy is rarely required for making diagnosis
(b) Antibiotic treatment is given for streptococcal infection
(c) Treatment is largely supportive
(d) Immunosuppressants are to be used for crescentic glomerulonephritis
37.
Which one of the following statements is NOT correct regarding the risk factors for
nephrolithiasis?

(a) Higher dietary calcium increases risk
(b) Higher animal protein intake increases risk
(c) Higher dietary potassium intake lowers risk
(d) Lower urine citrate increases the risk
38.
A 60-year old patient is admitted in emergency with seizure, aphasia and altered sensorium.
He has history of fever and headache for the preceding 3 days. The CSF examination is
unremarkable. What would be the probable diagnosis?
(a) Multiple sclerosis
(b) Tubercular meningitis
(c) Pyogenic (bacterial) meningitis
(d) Viral encephalitis
39.
Drug of choice for the treatment of Trigeminal Neuralgia is:
(a) Carbamazepine
(b) Aceclofenac
(c) Thiamine
(d) Prednisdone
40.
Constructional skills is a function of which lobe?
(a) Dominant temporal lobe
(b) Non dominant parietal lobe
(c) Frontal lobe
(d) Non dominant temporal lobe

41.
Vertical gaze palsy with convergence retraction nystagmus is seen in:
(a) Weber syndrome
(b) Millard Gubler syndrome
(c) Claude syndrome
(d) Parinaud syndrome
42.
A patient on looking forward was found to have his right eye deviated downwards and
outwards with pupil dilated. He is suffering from:
(a) Left 3
rd
nerve palsy
(b) Left 6
th
nerve palsy
(c) Right 4
th
nerve palsy
(d) Right 3
rd
nerve palsy
43.
All of the following are features of Cauda Equina syndrome EXCEPT:

(a) Low back pain
(b) Lower limb areflexia
(c) Loss of bladder function
(d) Extensor plantar response

44.
A 68-year old male presented in OPD with complaints of progressive small handwriting. On
examination, he had resting tremor, bradykinesia, rigidity and postural instability. This
patient is suffering most likely from:

(a) Parkinson?s disease
(b) Vascular dementia
(c) Alzheimer?s disease
(d) Frunto temporal dementia
45.
An elderly patient presents with abnormal gait. He has a wide base freezing gait with
imbalance, comprising short strides, shuffling along the floor and difficulty with starts and
turns. Heal-Shin test is normal. Which one of the following is most likely disorder?
(a) Cerebellar ataxia
(b) Sensory ataxia
(c) Frontal gait disorder
(d) Parkinson?s disease
46.
Cushing reflex, seen in conditions of raised intra cranial pressure includes all of the following
EXCEPT:
(a) Bradycardia
(b) Hypertension
(c) Irregular respiration
(d) Hypothermia

47.
A 35 year old man presents with history of low grade fever and headache for last five weeks.
Clinical examination is remarkable for signs of meningismus. He undergoes lumbar puncture
(LP) and a day later reports worsening of headache. Which one of the following features is
NOT consistent with diagnosis of ?Post-LP? headache?

(a) Post-LP headache usually begins within 48 hours
(b) Post-LP headache worsens in sitting position
(c) Post-LP headache is most severe upon waking up
(d) Post-LP headache may improve with caffeine intake
48.
Which one of the following investigations is NOT appropriate in a case of recurrent
thrombosis?
(a) Antiphospholipid antibodies
(b) Bcr-Abl assay
(c) Protein C and S assays
(d) Antithrombin level
49.
All of the following are causes of reactive thrombocytosis, EXCEPT:
(a) Hemolytic anemia
(b) Megaloblastic anemia
(c) Post splenectomy
(d) Chronic inflammatory disorders


50.
Cryoprecipitate cannot be used for treatment of which one of the following conditions?
(a) Von Willebrand disease
(b) Hypofibrinogenemia
(c) Hemophilia?B
(d) Hemphilia?A
51.
Pyruvate kinase deficiency results in deficiency of ATP production and a chronic haemolytic
anaemia. The disorder is inherited as an:

(a) Autosomal recessive trait
(b) X-linked recessive trait
(c) Autosomal dominant trait
(d) X-linked dominant trait
52.
Sickle cell disease may be associated with any of the following EXCEPT:

(a) Acute chest syndrome
(b) Aplastic crisis
(c) Splenic sequestration crisis
(d) Conn?s syndrome

53.
Fever with splenomegaly and lymphadenopathy can be seen in
(a) Infectious mononucleosis
(b) Chronic leukaemia
(c) Both infectious mononucleosis and chronic leukaemia
(d) Neither infectious mononucleosis nor chronic leukaemia
54.
Every individual carries four alpha gene alleles. Deletion of three alleles leads to
development of:
(a) Haemoglobin X disease
(b) Haemoglobin H disease
(c) Haemoglobin F disease
(d) Hydrops fetalis
55.
Which one of the following is NOT true regarding Chronic Myeloid Leukaemia (CML)?
(a) It is a clonal malignancy of haematopoetic stem
(b) Risk of developing CML is increased in monozygotic turns
(c) CML is defined by presence of BCR-ABL 1 fusion gene
(d) 10 year survival with TKI therapy is 85%









56.
Diagnostic criteria of multiple myeloma includes which of the following?

1. Increased malignant plasma cells in the bone marrow
2. Serum and/or urinary M protein
3. Skeletal lytic lesions
4. Skeletal blastic lesions

Select the correct answer using the code given below:
(a) 1, 2 and 3
(b) 1, 3 and 4
(c) 1, 2 and 4
(d) 2, 3 and 4
57.
The most predominant type of serum immunoglobulin involved in patients with plasma cell
disorder is:
(a) Ig G
(b) Ig A
(c) Ig D
(d) Light chains
58.
Which one of the following statements is NOT true regarding the thyroid function/disorder in
pregnancy?
(a) There is an increase in the metabolism of thyroxine by placenta
(b) Hyperemesis gravidarum may be associated with thyrotoxicosis in pregnancy
(c) Levothyroxine replacement therapy dose should be decreased by 30?50% early in
pregnancy
(d) Impaired cognitive impairment may be seen in the offspring of mother with
subclinical hypothyroidism

59.
Which one of the following is NOT the criterion for the diagnosis of Diabetes mellitus?
(a) Symptoms of diabetes and random plasma sugar concentration ? 200 mg/dL
(b) HbA1c ? 6.5 %
(c) 2-hours plasma glucose ? 200 mg/dL during an oral glucose tolerance test
(d) Fasting plasma glucose ? 110 mg/dL
60.
A young lady presents in outdoor clinic with complaints of menstrual irregularity, weight
gain, hair loss, tiredness and weakness. What will be the investigation of choice?
(a) FSH & LH level
(b) Estrogen level
(c) Free T
3
and free T
4
level
(d) Free T
4
and TSH level







61.
Consider the following statements about erectile dysfunction in diabetic males:

1. It affects 60 % of males
2. Its common cause is an underlying neuro-vascular pathology
3. It may be aggravated by beta-adrenergic agonist drugs
4. Endocrine disorders like hyperprolactinemia may cause it

Which of the statements given above are true?
(a) 1 and 2
(b) 2 and 3
(c) 2 and 4
(d) 3 and 4
62.
All of the following can cause Hyperprolactinemia, EXCEPT:
(a) Craniopharyngioma
(b) Hyperthyroidism
(c) Stress
(d) Chronic renal failure
63.
All of the following can be seen in a case of thyrotoxicosis, EXCEPT:
(a) Atrial fibrillation
(b) Periodic paralysis
(c) Deafness
(d) Osteoporosis
64.
Which one of the following hormones acts via the receptor tyrosine kinase?
(a) FSH
(b) TSH
(c) IGF-1
(d) PTH
65.
Consider the following disorders as constituent of Multiple Endocrine Neoplasia Type 2a
(MEN 2a):

1. Primary Hyperparathyroidism
2. Pituitary tumours
3. Medullary carcinoma of thyroid
4. Pheochromocytoma

Which of the above are correct?:

(a) 1, 2 and 4
(b) 1, 2 and 3
(c) 1, 3 and 4
(d) 2, 3 and 4




66.
Consider the following statements with regard to Graves? ophthalmopathy :

1. Proptosis is often asymmetric and can even appear to be unilateral
2. It is a clinical diagnosis
3. Worsening of symptoms upon glucocorticoid withdrawal is common
4. Radiation therapy is very effective in treatment

Which of the above statements are correct?

(a) 1 and 2 only
(b) 1, 2 and 3
(c) 2, 3 and 4
(d) 1 and 4 only
67.
All of the following are common aetiologies of delirium EXCEPT:
(a) Hyperthyroidism
(b) Hypothyroidism
(c) Hyperparathyroidism
(d) Hypoparathyroidism
68.
Key anthropometric measurements important for evaluating the degree of obesity are:

(a) Weight, height and waist circumference
(b) Weight, height and hip circumference
(c) Weight, height and mid-thigh circumference
(d) Weight, height and chest circumference
69.
What is the minimum BMI (Body Mass Index) recommended for considering adjuvant
pharmacologic treatment in obesity with comorbidity?
(a) 23 kg/m
2

(b) 25 kg/m
2

(c) 27 kg/m
2

(d) 30 kg/m
2

70.
A young medical student while working in HIV ward got pricked accidently on the finger by
a syringe needle which was used to draw blood from a patient. Patient?s infection status is not
known. Which one of the following therapeutic interventions will be the best interest of this
student?
(a) To start Dolultegravir immediately
(b) To wash wound with soap and water and start Emtricitabine
(c) To give Dolutegravir + Tenofovir + Emtricitabine
(d) To give Dolutegravir + Tenofovir + Emtricitabine + HBV Immunoglobulin
71.
Cryptococcal infection is acquired through:
(a) Inhalation route
(b) Bite of Culex mosquito
(c) Direct skin contact
(d) Ingestion of spores

72.
Which one of the following is NOT appropriate treatment for Echinococcus granulosus and
hydatid disease?
(a) Albendazole 400 mg twice in a day for 3 months
(b) Diethylcarbamazine 2 mg/kg thrice in a day for 12 days
(c) PAIR (percutaneous puncture, aspiration, injection of scolicide, reaspiration)
(d) Praziquantel (20 mg/kg twice daily for 14 days)
73.
Which one of the following infections is NOT amenable to post-exposure prophylaxis with
specific immunoglobulins?
(a) Hepatitis B
(b) HIV
(c) Tetanus
(d) Rabies
74.
Echinocandins are a class of:
(a) Antiviral agents
(b) Antiparasitic agents
(c) Antitubercular agents
(d) Antifungal agents
75.
Recommended first time drug for initial treatment of tuberculosis include all EXCEPT:
(a) Isoniazid
(b) Rifampin
(c) Ethambutol
(d) Ethionamide
76.
The most frequent adverse reaction of significance among people treated for drug-susceptible
tuberculosis is:
(a) Colitis
(b) Hepatitis
(c) Cystitis
(d) Enteritis
77.
All adult patients in whom anti-tubercular treatment is being started should undergo baseline
assessment of:
(a) Complete blood count
(b) Liver function test
(c) Lipid profile
(d) Pulmonary function test
78.
A 30 year old male presents with fever, headache, anorexia, nausea, vomiting and diarrhoea.
On examination tongue is coated with abdominal tenderness, soft splenomegaly, relative
bradycardia and rose spots rash. The most likely diagnosis is:

(a) Malaria fever
(b) Enteric fever
(c) Leptospirosis
(d) Dengue fever

79.
The extrapulmonary sites most commonly involved in tuberculosis are all EXCEPT:
(a) Lymph node TB
(b) Genitourinary TB
(c) Skeletal TB
(d) Skull TB
80.
Which of the following is NOT a clinical feature of Botulism?
(a) Impaired alertness
(b) Dysphagia
(c) Diplopia
(d) Ptosis
81.
Post exposure prophylaxis (PEP) for HIV should contain:
(a) 2 antiretroviral drugs administered for 4 weeks
(b) 2 antiretroviral drugs administered for 6 weeks
(c) 3 antiretroviral drugs administered for 4 weeks
(d) 3 antiretroviral drugs administered for 6 weeks
82.
Paralytic rabies is characterized by:
(a) Maculopapular rash
(b) Flaccid muscle weakness
(c) Spastic muscle weakness
(d) Hyperexcitability and facial weakness
83.
All of the following are seen in Marasmus EXCEPT:

(a) Reduced triceps skinfold
(b) Reduced mid arm circumference
(c) Protein wasting
(d) Decreased body mass index
84.
A 20 year young lady presents with high grade fever for five days and palpable purpura over
extremities. She is found to be confused, with presence of neck stiffness. Which of the
following is the most likely diagnosis?
(a) Disseminated intravascular coagulation
(b) Acute meningococcemia
(c) Antiphospholipid antibody syndrome
(d) Thrombotic thrombocytopenic purpura
85.
Fibroblast Growth Factor 23 (FGF 23) is increased in which one of the following conditions?
(a) Osteomalacia
(b) Osteoporosis
(c) Paget?s disease
(d) Renal Osteodystrophy





FirstRanker.com - FirstRanker's Choice
Combined Medical Services Examination-2020
Paper-I
1.
A chronic alcoholic develops a paroxysm of palpitations after alcohol binge. Which of the
following Arrhythmia is most likely?
(a) Ventricular fibrillation
(b) Ventricular premature complex
(c) Atrial flutter
(d) Atrial fibrillation
2.
An elderly-man with history of Diabetes mellitus and Coronary Artery Disease comes for
follow-up, with complaints of muscle pains. Which one of the following drugs could be the
most likely cause?
(a) Aspirin
(b) Glimepiride
(c) Enalapril
(d) Atorvastatin
3.
A 40-year old lady comes to outdoor clinic with complaints of sudden onset chest pain. The
chest X-ray shows bilateral Pneumothorax. Examination reveals abnormalities of body
habitus-including long arms, legs and finger (arachnodactyly), scoliosis, high arched palate,
joint hypermobility, and a pansystolic murmur at cardiac apex. Which one of the following
diseases is most likely?
(a) Takayasu?s arteritis
(b) Raynaud?s syndrome
(c) Marfan?s syndrome
(d) Rheumatic heart disease-Mitral regurgitation
4.
All of the following are indications for treadmill testing/exercise-testing EXCEPT:
(a) To confirm the diagnosis of angina
(b) To evaluate stable angina
(c) To assess outcome after coronary revascularization
(d) To evaluate the treatment efficacy of antianginal drugs
5.
Kussmaul?s sign is present in all of the following conditions, EXCEPT:

(a) Massive Pulmonary Embolism
(b) Restrictive Cardiomyopathy
(c) Hypertrophic Cardiomyopathy
(d) Right Ventricular Infarction

6.
Which one of the following is NOT a common cause of atrial fibrillation?

(a) Mitral regurgitation
(b) Hypothyroidism
(c) Hypertension
(d) Acute myocardial infarction

7.
In an ECG recording, P wave is produced by:

(a) Atrial depolarisation
(b) Atrial repolarisation
(c) Ventricular depolarisation
(d) Ventricular repolarisation
8.
Which one of the following modalities is NOT used in the treatment of ventricular
tachycardia?

(a) DC Cardioversion
(b) Injection Lignocaine
(c) Injection Amiodarone
(d) Injection Adenosine
9.
Which one of the following drugs is a Direct Renin Inhibitor?

(a) Benedipine
(b) Azilsartan
(c) Aliskiren
(d) Lisinopril
10.
All of the following are causes of systolic hypertension with wide pulse pressure EXCEPT:

(a) Aortic regurgitation
(b) Thyrotoxicosis
(c) Patent ductus arteriosus
(d) Aortic stenosis
11.
Signet-ring sign on CT-chest is suggestive of:
(a) Bronchiectasis
(b) Active alveolitis
(c) Aspergilloma
(d) Sarcoidosis
12.
Oral corticosteroids are best introduced in the treatment of chronic bronchial asthma when it
is:
(a) Mild persistent
(b) Moderate persistent
(c) Severe persistent
(d) Very severe persistent
13.
An 18-year old male has insulin dependent diabetes, with malabsorption syndrome and
bilateral upper lobe bronchiectasis. A diagnostic work-up will include all EXCEPT:
(a) 24 hour fecal fat-estimation
(b) CT-chest
(c) Lung biopsy
(d) DNA sequencing study
14.
Which one of the following antigens is commonly associated with causation of Farmer?s
lung?
(a) Penicillium
(b) Actinomycetes
(c) Aspergillus
(d) Candida
15.
The Gene X-pert test used for MTB detection has the additional advantage of detection of
which of the following?

(a) INH resistance
(b) Rifampicin resistance
(c) Multi drug resistance
(d) Ethambutol resistance
16.
The most common cause of chronic type II respiratory failure is:
(a) Severe pneumonia
(b) Severe COPD
(c) Severe bronchial asthma
(d) Severe pulmonary thromboembolism
17.
The most frequent symptom in respiratory diseases is:
(a) Breathlessness
(b) Chest pain
(c) Hemoptysis
(d) Cough
18.
A 26-year old young lady attends medical emergency and is labelled as acute severe asthma
by the emergency physician. Which one of the following is unlikely to be a part of the
prescription?

(a) Albuterol
(b) Anti-leukotrienes
(c) Aminophylline
(d) Magnesium sulfate
19.
Consider the following statements with regard to respiratory examination:

1. Change in note, when patient phonates ?EEE? (Egophony) is characteristic of
interstitial fibrosis
2. Whispered pectoriloquy is characteristic of lung consolidation
3. Monophonic wheeze is characteristic of asthma
4. Hyper-resonant note on percussion is characteristic of pnemothorax

Which of the above statements are correct?
(a) 1 and 3
(b) 2 and 4 only
(c) 1, 2 and 4
(d) 2, 3 and 4
20.
All of the following are the causes of exudative Pleural Effusion, EXCEPT:
(a) SVC obstruction
(b) Fungal infection
(c) SLE
(d) Meig syndrome
21.
Which one of the following is NOT true about Ghrelin?
(a) It stimulates appetite
(b) It decreases gastric emptying
(c) It increases acid secretion
(d) Fasting increases its secretion
22.
Fecal elastase test is used for diagnosing which one of the following conditions?
(a) Lactose intolerance
(b) Mucosal inflammation
(c) Bile acid secretory defects
(d) Pancreatic dysfunction
23.
The recent classification system for listing a patient as a candidate for liver transplantation is:
(a) Child Pugh score
(b) APACHE score
(c) MELD score
(d) Metavir score
24.
The blood supply of liver consists of:
(a) 50 % hepatic artery and 50 % portal vein
(b) 80 % portal vein and 20 % hepatic artery
(c) 80 % hepatic artery and 20 % portal vein
(d) 70 % hepatic artery and 30 % portal vein
25.
All of the following conditions are associated with Glomerulonephritis with low complement
level, EXCEPT:
(a) Subacute bacterial endocarditis
(b) IgA nephropathy
(c) Systemic lupus erythematosus
(d) Cryoglobulinaemia













26.
Consider the following statements with regard to oral aphthous ulcers:

1. They are superficial and painful
2. They may occur in women just prior to menstruation
3. Oral glucocorticoids may be needed for the treatment of severe, recurrent cases
4. They are pre-malignant and progress over few years to squamous cell carcinoma of
the oral cavity

Which of the above statements are correct?

(a) 1 and 4 only
(b) 2 and 3 only
(c) 1, 2 and 3 only
(d) 1, 2, 3 and 4
27.
Which one of the following statements about Barrett?s Oesophagus is NOT correct?

(a) It is a pre-malignant condition
(b) Normal squamous cells lining the lower oesophagus are replaced by columnar
cells
(c) Normal columnar cells lining the lower oesophagus are replaced by squamous
cells
(d) It is an adaptive response to chronic gastro-oesophageal reflux

28.
The most common cause of acute hepatitis outbreaks in India is:

(a) Hepatitis E
(b) Hepatitis C
(c) Hepatitis B
(d) Hepatitis A

29.
Spontaneous oesophageal perforation after a bout of forceful vomiting or retching is
characteristically seen in which one of the following conditions?

(a) Menetrier?s disease
(b) Boerhaave?s syndrome
(c) Achalasia of the oesophagus
(d) Barrett?s oesophagus
30.
Eradication of helicobacter pylori infection may prove beneficial in the following extra-
gastric disorders EXCEPT:

(a) Unexplained vitamin B12 deficiency
(b) Idiopathic thrombocytopenic purpura
(c) Iron deficiency anaemia without gastro-intestinal bleeding
(d) Acute glomerulonephritis

31.
What is the target blood Hemoglobin level when treating anaemia in Stage-4 chronic kidney
disease patient?
(a) 8 to 10 gm/dL
(b) 10 to 12 gm/dL
(c) 12 to 13 gm/dL
(d) 13 to 14 gm/dL
32.
Which one of the following complications of chronic kidney disease is observed in patients
with low parathyroid hormone levels?
(a) Adynamic bone disease
(b) Tumoral calcinosis
(c) Osteitis fibrosa cystic
(d) Calciphylaxis
33.
All of the following statements about adult polycystic kidney disease are true, EXCEPT:
(a) It is inherited as autosomal dominant trait
(b) It is usually associated with marked proteinuria
(c) It is usually associated with systematic hypertension, from young age
(d) It is associated with Berry aneurysm
34.
Consider the following statements in relation to an adult patient:

1. Oliguria is defined as passage of urine less than 300 ml per day
2. Anuria is said to exist when less than 50 ml urine is passed per day
3. Polyuria is defined as urine volume in excess of 3 litre per day

Which of the statements given above are correct?
(a) 1 and 3 only
(b) 2 and 3 only
(c) 1 and 2 only
(d) 1, 2 and 3

35.
White cell casts in urine examination are strongly suggestive of:
(a) Nephritis
(b) Pyelonephritis
(c) Renal stone disease
(d) Papillitis
36.
A 13-year old boy presents with hematuria, oliguria, edema and hypertension. He has history
of sore throat two weeks prior to presentation. Laboratory investigations are remarkable for
low C
3
and increased titres of ASO and antiDNase. Which one of the following statements is
NOT correct about management for this condition?

(a) Renal biopsy is rarely required for making diagnosis
(b) Antibiotic treatment is given for streptococcal infection
(c) Treatment is largely supportive
(d) Immunosuppressants are to be used for crescentic glomerulonephritis
37.
Which one of the following statements is NOT correct regarding the risk factors for
nephrolithiasis?

(a) Higher dietary calcium increases risk
(b) Higher animal protein intake increases risk
(c) Higher dietary potassium intake lowers risk
(d) Lower urine citrate increases the risk
38.
A 60-year old patient is admitted in emergency with seizure, aphasia and altered sensorium.
He has history of fever and headache for the preceding 3 days. The CSF examination is
unremarkable. What would be the probable diagnosis?
(a) Multiple sclerosis
(b) Tubercular meningitis
(c) Pyogenic (bacterial) meningitis
(d) Viral encephalitis
39.
Drug of choice for the treatment of Trigeminal Neuralgia is:
(a) Carbamazepine
(b) Aceclofenac
(c) Thiamine
(d) Prednisdone
40.
Constructional skills is a function of which lobe?
(a) Dominant temporal lobe
(b) Non dominant parietal lobe
(c) Frontal lobe
(d) Non dominant temporal lobe

41.
Vertical gaze palsy with convergence retraction nystagmus is seen in:
(a) Weber syndrome
(b) Millard Gubler syndrome
(c) Claude syndrome
(d) Parinaud syndrome
42.
A patient on looking forward was found to have his right eye deviated downwards and
outwards with pupil dilated. He is suffering from:
(a) Left 3
rd
nerve palsy
(b) Left 6
th
nerve palsy
(c) Right 4
th
nerve palsy
(d) Right 3
rd
nerve palsy
43.
All of the following are features of Cauda Equina syndrome EXCEPT:

(a) Low back pain
(b) Lower limb areflexia
(c) Loss of bladder function
(d) Extensor plantar response

44.
A 68-year old male presented in OPD with complaints of progressive small handwriting. On
examination, he had resting tremor, bradykinesia, rigidity and postural instability. This
patient is suffering most likely from:

(a) Parkinson?s disease
(b) Vascular dementia
(c) Alzheimer?s disease
(d) Frunto temporal dementia
45.
An elderly patient presents with abnormal gait. He has a wide base freezing gait with
imbalance, comprising short strides, shuffling along the floor and difficulty with starts and
turns. Heal-Shin test is normal. Which one of the following is most likely disorder?
(a) Cerebellar ataxia
(b) Sensory ataxia
(c) Frontal gait disorder
(d) Parkinson?s disease
46.
Cushing reflex, seen in conditions of raised intra cranial pressure includes all of the following
EXCEPT:
(a) Bradycardia
(b) Hypertension
(c) Irregular respiration
(d) Hypothermia

47.
A 35 year old man presents with history of low grade fever and headache for last five weeks.
Clinical examination is remarkable for signs of meningismus. He undergoes lumbar puncture
(LP) and a day later reports worsening of headache. Which one of the following features is
NOT consistent with diagnosis of ?Post-LP? headache?

(a) Post-LP headache usually begins within 48 hours
(b) Post-LP headache worsens in sitting position
(c) Post-LP headache is most severe upon waking up
(d) Post-LP headache may improve with caffeine intake
48.
Which one of the following investigations is NOT appropriate in a case of recurrent
thrombosis?
(a) Antiphospholipid antibodies
(b) Bcr-Abl assay
(c) Protein C and S assays
(d) Antithrombin level
49.
All of the following are causes of reactive thrombocytosis, EXCEPT:
(a) Hemolytic anemia
(b) Megaloblastic anemia
(c) Post splenectomy
(d) Chronic inflammatory disorders


50.
Cryoprecipitate cannot be used for treatment of which one of the following conditions?
(a) Von Willebrand disease
(b) Hypofibrinogenemia
(c) Hemophilia?B
(d) Hemphilia?A
51.
Pyruvate kinase deficiency results in deficiency of ATP production and a chronic haemolytic
anaemia. The disorder is inherited as an:

(a) Autosomal recessive trait
(b) X-linked recessive trait
(c) Autosomal dominant trait
(d) X-linked dominant trait
52.
Sickle cell disease may be associated with any of the following EXCEPT:

(a) Acute chest syndrome
(b) Aplastic crisis
(c) Splenic sequestration crisis
(d) Conn?s syndrome

53.
Fever with splenomegaly and lymphadenopathy can be seen in
(a) Infectious mononucleosis
(b) Chronic leukaemia
(c) Both infectious mononucleosis and chronic leukaemia
(d) Neither infectious mononucleosis nor chronic leukaemia
54.
Every individual carries four alpha gene alleles. Deletion of three alleles leads to
development of:
(a) Haemoglobin X disease
(b) Haemoglobin H disease
(c) Haemoglobin F disease
(d) Hydrops fetalis
55.
Which one of the following is NOT true regarding Chronic Myeloid Leukaemia (CML)?
(a) It is a clonal malignancy of haematopoetic stem
(b) Risk of developing CML is increased in monozygotic turns
(c) CML is defined by presence of BCR-ABL 1 fusion gene
(d) 10 year survival with TKI therapy is 85%









56.
Diagnostic criteria of multiple myeloma includes which of the following?

1. Increased malignant plasma cells in the bone marrow
2. Serum and/or urinary M protein
3. Skeletal lytic lesions
4. Skeletal blastic lesions

Select the correct answer using the code given below:
(a) 1, 2 and 3
(b) 1, 3 and 4
(c) 1, 2 and 4
(d) 2, 3 and 4
57.
The most predominant type of serum immunoglobulin involved in patients with plasma cell
disorder is:
(a) Ig G
(b) Ig A
(c) Ig D
(d) Light chains
58.
Which one of the following statements is NOT true regarding the thyroid function/disorder in
pregnancy?
(a) There is an increase in the metabolism of thyroxine by placenta
(b) Hyperemesis gravidarum may be associated with thyrotoxicosis in pregnancy
(c) Levothyroxine replacement therapy dose should be decreased by 30?50% early in
pregnancy
(d) Impaired cognitive impairment may be seen in the offspring of mother with
subclinical hypothyroidism

59.
Which one of the following is NOT the criterion for the diagnosis of Diabetes mellitus?
(a) Symptoms of diabetes and random plasma sugar concentration ? 200 mg/dL
(b) HbA1c ? 6.5 %
(c) 2-hours plasma glucose ? 200 mg/dL during an oral glucose tolerance test
(d) Fasting plasma glucose ? 110 mg/dL
60.
A young lady presents in outdoor clinic with complaints of menstrual irregularity, weight
gain, hair loss, tiredness and weakness. What will be the investigation of choice?
(a) FSH & LH level
(b) Estrogen level
(c) Free T
3
and free T
4
level
(d) Free T
4
and TSH level







61.
Consider the following statements about erectile dysfunction in diabetic males:

1. It affects 60 % of males
2. Its common cause is an underlying neuro-vascular pathology
3. It may be aggravated by beta-adrenergic agonist drugs
4. Endocrine disorders like hyperprolactinemia may cause it

Which of the statements given above are true?
(a) 1 and 2
(b) 2 and 3
(c) 2 and 4
(d) 3 and 4
62.
All of the following can cause Hyperprolactinemia, EXCEPT:
(a) Craniopharyngioma
(b) Hyperthyroidism
(c) Stress
(d) Chronic renal failure
63.
All of the following can be seen in a case of thyrotoxicosis, EXCEPT:
(a) Atrial fibrillation
(b) Periodic paralysis
(c) Deafness
(d) Osteoporosis
64.
Which one of the following hormones acts via the receptor tyrosine kinase?
(a) FSH
(b) TSH
(c) IGF-1
(d) PTH
65.
Consider the following disorders as constituent of Multiple Endocrine Neoplasia Type 2a
(MEN 2a):

1. Primary Hyperparathyroidism
2. Pituitary tumours
3. Medullary carcinoma of thyroid
4. Pheochromocytoma

Which of the above are correct?:

(a) 1, 2 and 4
(b) 1, 2 and 3
(c) 1, 3 and 4
(d) 2, 3 and 4




66.
Consider the following statements with regard to Graves? ophthalmopathy :

1. Proptosis is often asymmetric and can even appear to be unilateral
2. It is a clinical diagnosis
3. Worsening of symptoms upon glucocorticoid withdrawal is common
4. Radiation therapy is very effective in treatment

Which of the above statements are correct?

(a) 1 and 2 only
(b) 1, 2 and 3
(c) 2, 3 and 4
(d) 1 and 4 only
67.
All of the following are common aetiologies of delirium EXCEPT:
(a) Hyperthyroidism
(b) Hypothyroidism
(c) Hyperparathyroidism
(d) Hypoparathyroidism
68.
Key anthropometric measurements important for evaluating the degree of obesity are:

(a) Weight, height and waist circumference
(b) Weight, height and hip circumference
(c) Weight, height and mid-thigh circumference
(d) Weight, height and chest circumference
69.
What is the minimum BMI (Body Mass Index) recommended for considering adjuvant
pharmacologic treatment in obesity with comorbidity?
(a) 23 kg/m
2

(b) 25 kg/m
2

(c) 27 kg/m
2

(d) 30 kg/m
2

70.
A young medical student while working in HIV ward got pricked accidently on the finger by
a syringe needle which was used to draw blood from a patient. Patient?s infection status is not
known. Which one of the following therapeutic interventions will be the best interest of this
student?
(a) To start Dolultegravir immediately
(b) To wash wound with soap and water and start Emtricitabine
(c) To give Dolutegravir + Tenofovir + Emtricitabine
(d) To give Dolutegravir + Tenofovir + Emtricitabine + HBV Immunoglobulin
71.
Cryptococcal infection is acquired through:
(a) Inhalation route
(b) Bite of Culex mosquito
(c) Direct skin contact
(d) Ingestion of spores

72.
Which one of the following is NOT appropriate treatment for Echinococcus granulosus and
hydatid disease?
(a) Albendazole 400 mg twice in a day for 3 months
(b) Diethylcarbamazine 2 mg/kg thrice in a day for 12 days
(c) PAIR (percutaneous puncture, aspiration, injection of scolicide, reaspiration)
(d) Praziquantel (20 mg/kg twice daily for 14 days)
73.
Which one of the following infections is NOT amenable to post-exposure prophylaxis with
specific immunoglobulins?
(a) Hepatitis B
(b) HIV
(c) Tetanus
(d) Rabies
74.
Echinocandins are a class of:
(a) Antiviral agents
(b) Antiparasitic agents
(c) Antitubercular agents
(d) Antifungal agents
75.
Recommended first time drug for initial treatment of tuberculosis include all EXCEPT:
(a) Isoniazid
(b) Rifampin
(c) Ethambutol
(d) Ethionamide
76.
The most frequent adverse reaction of significance among people treated for drug-susceptible
tuberculosis is:
(a) Colitis
(b) Hepatitis
(c) Cystitis
(d) Enteritis
77.
All adult patients in whom anti-tubercular treatment is being started should undergo baseline
assessment of:
(a) Complete blood count
(b) Liver function test
(c) Lipid profile
(d) Pulmonary function test
78.
A 30 year old male presents with fever, headache, anorexia, nausea, vomiting and diarrhoea.
On examination tongue is coated with abdominal tenderness, soft splenomegaly, relative
bradycardia and rose spots rash. The most likely diagnosis is:

(a) Malaria fever
(b) Enteric fever
(c) Leptospirosis
(d) Dengue fever

79.
The extrapulmonary sites most commonly involved in tuberculosis are all EXCEPT:
(a) Lymph node TB
(b) Genitourinary TB
(c) Skeletal TB
(d) Skull TB
80.
Which of the following is NOT a clinical feature of Botulism?
(a) Impaired alertness
(b) Dysphagia
(c) Diplopia
(d) Ptosis
81.
Post exposure prophylaxis (PEP) for HIV should contain:
(a) 2 antiretroviral drugs administered for 4 weeks
(b) 2 antiretroviral drugs administered for 6 weeks
(c) 3 antiretroviral drugs administered for 4 weeks
(d) 3 antiretroviral drugs administered for 6 weeks
82.
Paralytic rabies is characterized by:
(a) Maculopapular rash
(b) Flaccid muscle weakness
(c) Spastic muscle weakness
(d) Hyperexcitability and facial weakness
83.
All of the following are seen in Marasmus EXCEPT:

(a) Reduced triceps skinfold
(b) Reduced mid arm circumference
(c) Protein wasting
(d) Decreased body mass index
84.
A 20 year young lady presents with high grade fever for five days and palpable purpura over
extremities. She is found to be confused, with presence of neck stiffness. Which of the
following is the most likely diagnosis?
(a) Disseminated intravascular coagulation
(b) Acute meningococcemia
(c) Antiphospholipid antibody syndrome
(d) Thrombotic thrombocytopenic purpura
85.
Fibroblast Growth Factor 23 (FGF 23) is increased in which one of the following conditions?
(a) Osteomalacia
(b) Osteoporosis
(c) Paget?s disease
(d) Renal Osteodystrophy





86.
Which of the following crystals are deposited in a case of pseudogout?
(a) Monosodium urate
(b) Calcium phosphate
(c) Cholestrol crystals
(d) Calcium pyrophosphate dehydrate (CPPD)
87.
Which of the following features are associated with poor prognosis in Diffuse Cutaneous
Systemic Sclerosis?

1. Older age
2. A high gas transfer factor for carbon monoxide (TLCO)
3. Proteinuria
4. Diffuse skin disease

Select the correct answer using the code given below:
(a) 1, 2 and 4
(b) 3 and 4 only
(c) 1, 3 and 4
(d) 1, 2 and 3

88.
Which of the following statements regarding pegloticase are correct?

1. It is enzyme conjugate
2. It is useful in chronic Gout management
3. The main adverse effects are infusion reactions and development of antibodies

Select the correct answer using the code given below:
(a) 1 and 2 only
(b) 2 and 3 only
(c) 1 and 3 only
(d) 1, 2 and 3
89.
The most common cause of sensorineural hearing loss in adults is:
(a) Otosclerosis
(b) Meniere?s disease
(c) Presbycusis
(d) Meningitis
90.
A 50-year old male presents with dizziness. Which one of the following clinical findings does
NOT favour a central cause?
(a) Poor saccades
(b) Gaze evoked nystagmus
(c) Inhibition by visual fixation
(d) Presence of diplopia, dysarthria




FirstRanker.com - FirstRanker's Choice
Combined Medical Services Examination-2020
Paper-I
1.
A chronic alcoholic develops a paroxysm of palpitations after alcohol binge. Which of the
following Arrhythmia is most likely?
(a) Ventricular fibrillation
(b) Ventricular premature complex
(c) Atrial flutter
(d) Atrial fibrillation
2.
An elderly-man with history of Diabetes mellitus and Coronary Artery Disease comes for
follow-up, with complaints of muscle pains. Which one of the following drugs could be the
most likely cause?
(a) Aspirin
(b) Glimepiride
(c) Enalapril
(d) Atorvastatin
3.
A 40-year old lady comes to outdoor clinic with complaints of sudden onset chest pain. The
chest X-ray shows bilateral Pneumothorax. Examination reveals abnormalities of body
habitus-including long arms, legs and finger (arachnodactyly), scoliosis, high arched palate,
joint hypermobility, and a pansystolic murmur at cardiac apex. Which one of the following
diseases is most likely?
(a) Takayasu?s arteritis
(b) Raynaud?s syndrome
(c) Marfan?s syndrome
(d) Rheumatic heart disease-Mitral regurgitation
4.
All of the following are indications for treadmill testing/exercise-testing EXCEPT:
(a) To confirm the diagnosis of angina
(b) To evaluate stable angina
(c) To assess outcome after coronary revascularization
(d) To evaluate the treatment efficacy of antianginal drugs
5.
Kussmaul?s sign is present in all of the following conditions, EXCEPT:

(a) Massive Pulmonary Embolism
(b) Restrictive Cardiomyopathy
(c) Hypertrophic Cardiomyopathy
(d) Right Ventricular Infarction

6.
Which one of the following is NOT a common cause of atrial fibrillation?

(a) Mitral regurgitation
(b) Hypothyroidism
(c) Hypertension
(d) Acute myocardial infarction

7.
In an ECG recording, P wave is produced by:

(a) Atrial depolarisation
(b) Atrial repolarisation
(c) Ventricular depolarisation
(d) Ventricular repolarisation
8.
Which one of the following modalities is NOT used in the treatment of ventricular
tachycardia?

(a) DC Cardioversion
(b) Injection Lignocaine
(c) Injection Amiodarone
(d) Injection Adenosine
9.
Which one of the following drugs is a Direct Renin Inhibitor?

(a) Benedipine
(b) Azilsartan
(c) Aliskiren
(d) Lisinopril
10.
All of the following are causes of systolic hypertension with wide pulse pressure EXCEPT:

(a) Aortic regurgitation
(b) Thyrotoxicosis
(c) Patent ductus arteriosus
(d) Aortic stenosis
11.
Signet-ring sign on CT-chest is suggestive of:
(a) Bronchiectasis
(b) Active alveolitis
(c) Aspergilloma
(d) Sarcoidosis
12.
Oral corticosteroids are best introduced in the treatment of chronic bronchial asthma when it
is:
(a) Mild persistent
(b) Moderate persistent
(c) Severe persistent
(d) Very severe persistent
13.
An 18-year old male has insulin dependent diabetes, with malabsorption syndrome and
bilateral upper lobe bronchiectasis. A diagnostic work-up will include all EXCEPT:
(a) 24 hour fecal fat-estimation
(b) CT-chest
(c) Lung biopsy
(d) DNA sequencing study
14.
Which one of the following antigens is commonly associated with causation of Farmer?s
lung?
(a) Penicillium
(b) Actinomycetes
(c) Aspergillus
(d) Candida
15.
The Gene X-pert test used for MTB detection has the additional advantage of detection of
which of the following?

(a) INH resistance
(b) Rifampicin resistance
(c) Multi drug resistance
(d) Ethambutol resistance
16.
The most common cause of chronic type II respiratory failure is:
(a) Severe pneumonia
(b) Severe COPD
(c) Severe bronchial asthma
(d) Severe pulmonary thromboembolism
17.
The most frequent symptom in respiratory diseases is:
(a) Breathlessness
(b) Chest pain
(c) Hemoptysis
(d) Cough
18.
A 26-year old young lady attends medical emergency and is labelled as acute severe asthma
by the emergency physician. Which one of the following is unlikely to be a part of the
prescription?

(a) Albuterol
(b) Anti-leukotrienes
(c) Aminophylline
(d) Magnesium sulfate
19.
Consider the following statements with regard to respiratory examination:

1. Change in note, when patient phonates ?EEE? (Egophony) is characteristic of
interstitial fibrosis
2. Whispered pectoriloquy is characteristic of lung consolidation
3. Monophonic wheeze is characteristic of asthma
4. Hyper-resonant note on percussion is characteristic of pnemothorax

Which of the above statements are correct?
(a) 1 and 3
(b) 2 and 4 only
(c) 1, 2 and 4
(d) 2, 3 and 4
20.
All of the following are the causes of exudative Pleural Effusion, EXCEPT:
(a) SVC obstruction
(b) Fungal infection
(c) SLE
(d) Meig syndrome
21.
Which one of the following is NOT true about Ghrelin?
(a) It stimulates appetite
(b) It decreases gastric emptying
(c) It increases acid secretion
(d) Fasting increases its secretion
22.
Fecal elastase test is used for diagnosing which one of the following conditions?
(a) Lactose intolerance
(b) Mucosal inflammation
(c) Bile acid secretory defects
(d) Pancreatic dysfunction
23.
The recent classification system for listing a patient as a candidate for liver transplantation is:
(a) Child Pugh score
(b) APACHE score
(c) MELD score
(d) Metavir score
24.
The blood supply of liver consists of:
(a) 50 % hepatic artery and 50 % portal vein
(b) 80 % portal vein and 20 % hepatic artery
(c) 80 % hepatic artery and 20 % portal vein
(d) 70 % hepatic artery and 30 % portal vein
25.
All of the following conditions are associated with Glomerulonephritis with low complement
level, EXCEPT:
(a) Subacute bacterial endocarditis
(b) IgA nephropathy
(c) Systemic lupus erythematosus
(d) Cryoglobulinaemia













26.
Consider the following statements with regard to oral aphthous ulcers:

1. They are superficial and painful
2. They may occur in women just prior to menstruation
3. Oral glucocorticoids may be needed for the treatment of severe, recurrent cases
4. They are pre-malignant and progress over few years to squamous cell carcinoma of
the oral cavity

Which of the above statements are correct?

(a) 1 and 4 only
(b) 2 and 3 only
(c) 1, 2 and 3 only
(d) 1, 2, 3 and 4
27.
Which one of the following statements about Barrett?s Oesophagus is NOT correct?

(a) It is a pre-malignant condition
(b) Normal squamous cells lining the lower oesophagus are replaced by columnar
cells
(c) Normal columnar cells lining the lower oesophagus are replaced by squamous
cells
(d) It is an adaptive response to chronic gastro-oesophageal reflux

28.
The most common cause of acute hepatitis outbreaks in India is:

(a) Hepatitis E
(b) Hepatitis C
(c) Hepatitis B
(d) Hepatitis A

29.
Spontaneous oesophageal perforation after a bout of forceful vomiting or retching is
characteristically seen in which one of the following conditions?

(a) Menetrier?s disease
(b) Boerhaave?s syndrome
(c) Achalasia of the oesophagus
(d) Barrett?s oesophagus
30.
Eradication of helicobacter pylori infection may prove beneficial in the following extra-
gastric disorders EXCEPT:

(a) Unexplained vitamin B12 deficiency
(b) Idiopathic thrombocytopenic purpura
(c) Iron deficiency anaemia without gastro-intestinal bleeding
(d) Acute glomerulonephritis

31.
What is the target blood Hemoglobin level when treating anaemia in Stage-4 chronic kidney
disease patient?
(a) 8 to 10 gm/dL
(b) 10 to 12 gm/dL
(c) 12 to 13 gm/dL
(d) 13 to 14 gm/dL
32.
Which one of the following complications of chronic kidney disease is observed in patients
with low parathyroid hormone levels?
(a) Adynamic bone disease
(b) Tumoral calcinosis
(c) Osteitis fibrosa cystic
(d) Calciphylaxis
33.
All of the following statements about adult polycystic kidney disease are true, EXCEPT:
(a) It is inherited as autosomal dominant trait
(b) It is usually associated with marked proteinuria
(c) It is usually associated with systematic hypertension, from young age
(d) It is associated with Berry aneurysm
34.
Consider the following statements in relation to an adult patient:

1. Oliguria is defined as passage of urine less than 300 ml per day
2. Anuria is said to exist when less than 50 ml urine is passed per day
3. Polyuria is defined as urine volume in excess of 3 litre per day

Which of the statements given above are correct?
(a) 1 and 3 only
(b) 2 and 3 only
(c) 1 and 2 only
(d) 1, 2 and 3

35.
White cell casts in urine examination are strongly suggestive of:
(a) Nephritis
(b) Pyelonephritis
(c) Renal stone disease
(d) Papillitis
36.
A 13-year old boy presents with hematuria, oliguria, edema and hypertension. He has history
of sore throat two weeks prior to presentation. Laboratory investigations are remarkable for
low C
3
and increased titres of ASO and antiDNase. Which one of the following statements is
NOT correct about management for this condition?

(a) Renal biopsy is rarely required for making diagnosis
(b) Antibiotic treatment is given for streptococcal infection
(c) Treatment is largely supportive
(d) Immunosuppressants are to be used for crescentic glomerulonephritis
37.
Which one of the following statements is NOT correct regarding the risk factors for
nephrolithiasis?

(a) Higher dietary calcium increases risk
(b) Higher animal protein intake increases risk
(c) Higher dietary potassium intake lowers risk
(d) Lower urine citrate increases the risk
38.
A 60-year old patient is admitted in emergency with seizure, aphasia and altered sensorium.
He has history of fever and headache for the preceding 3 days. The CSF examination is
unremarkable. What would be the probable diagnosis?
(a) Multiple sclerosis
(b) Tubercular meningitis
(c) Pyogenic (bacterial) meningitis
(d) Viral encephalitis
39.
Drug of choice for the treatment of Trigeminal Neuralgia is:
(a) Carbamazepine
(b) Aceclofenac
(c) Thiamine
(d) Prednisdone
40.
Constructional skills is a function of which lobe?
(a) Dominant temporal lobe
(b) Non dominant parietal lobe
(c) Frontal lobe
(d) Non dominant temporal lobe

41.
Vertical gaze palsy with convergence retraction nystagmus is seen in:
(a) Weber syndrome
(b) Millard Gubler syndrome
(c) Claude syndrome
(d) Parinaud syndrome
42.
A patient on looking forward was found to have his right eye deviated downwards and
outwards with pupil dilated. He is suffering from:
(a) Left 3
rd
nerve palsy
(b) Left 6
th
nerve palsy
(c) Right 4
th
nerve palsy
(d) Right 3
rd
nerve palsy
43.
All of the following are features of Cauda Equina syndrome EXCEPT:

(a) Low back pain
(b) Lower limb areflexia
(c) Loss of bladder function
(d) Extensor plantar response

44.
A 68-year old male presented in OPD with complaints of progressive small handwriting. On
examination, he had resting tremor, bradykinesia, rigidity and postural instability. This
patient is suffering most likely from:

(a) Parkinson?s disease
(b) Vascular dementia
(c) Alzheimer?s disease
(d) Frunto temporal dementia
45.
An elderly patient presents with abnormal gait. He has a wide base freezing gait with
imbalance, comprising short strides, shuffling along the floor and difficulty with starts and
turns. Heal-Shin test is normal. Which one of the following is most likely disorder?
(a) Cerebellar ataxia
(b) Sensory ataxia
(c) Frontal gait disorder
(d) Parkinson?s disease
46.
Cushing reflex, seen in conditions of raised intra cranial pressure includes all of the following
EXCEPT:
(a) Bradycardia
(b) Hypertension
(c) Irregular respiration
(d) Hypothermia

47.
A 35 year old man presents with history of low grade fever and headache for last five weeks.
Clinical examination is remarkable for signs of meningismus. He undergoes lumbar puncture
(LP) and a day later reports worsening of headache. Which one of the following features is
NOT consistent with diagnosis of ?Post-LP? headache?

(a) Post-LP headache usually begins within 48 hours
(b) Post-LP headache worsens in sitting position
(c) Post-LP headache is most severe upon waking up
(d) Post-LP headache may improve with caffeine intake
48.
Which one of the following investigations is NOT appropriate in a case of recurrent
thrombosis?
(a) Antiphospholipid antibodies
(b) Bcr-Abl assay
(c) Protein C and S assays
(d) Antithrombin level
49.
All of the following are causes of reactive thrombocytosis, EXCEPT:
(a) Hemolytic anemia
(b) Megaloblastic anemia
(c) Post splenectomy
(d) Chronic inflammatory disorders


50.
Cryoprecipitate cannot be used for treatment of which one of the following conditions?
(a) Von Willebrand disease
(b) Hypofibrinogenemia
(c) Hemophilia?B
(d) Hemphilia?A
51.
Pyruvate kinase deficiency results in deficiency of ATP production and a chronic haemolytic
anaemia. The disorder is inherited as an:

(a) Autosomal recessive trait
(b) X-linked recessive trait
(c) Autosomal dominant trait
(d) X-linked dominant trait
52.
Sickle cell disease may be associated with any of the following EXCEPT:

(a) Acute chest syndrome
(b) Aplastic crisis
(c) Splenic sequestration crisis
(d) Conn?s syndrome

53.
Fever with splenomegaly and lymphadenopathy can be seen in
(a) Infectious mononucleosis
(b) Chronic leukaemia
(c) Both infectious mononucleosis and chronic leukaemia
(d) Neither infectious mononucleosis nor chronic leukaemia
54.
Every individual carries four alpha gene alleles. Deletion of three alleles leads to
development of:
(a) Haemoglobin X disease
(b) Haemoglobin H disease
(c) Haemoglobin F disease
(d) Hydrops fetalis
55.
Which one of the following is NOT true regarding Chronic Myeloid Leukaemia (CML)?
(a) It is a clonal malignancy of haematopoetic stem
(b) Risk of developing CML is increased in monozygotic turns
(c) CML is defined by presence of BCR-ABL 1 fusion gene
(d) 10 year survival with TKI therapy is 85%









56.
Diagnostic criteria of multiple myeloma includes which of the following?

1. Increased malignant plasma cells in the bone marrow
2. Serum and/or urinary M protein
3. Skeletal lytic lesions
4. Skeletal blastic lesions

Select the correct answer using the code given below:
(a) 1, 2 and 3
(b) 1, 3 and 4
(c) 1, 2 and 4
(d) 2, 3 and 4
57.
The most predominant type of serum immunoglobulin involved in patients with plasma cell
disorder is:
(a) Ig G
(b) Ig A
(c) Ig D
(d) Light chains
58.
Which one of the following statements is NOT true regarding the thyroid function/disorder in
pregnancy?
(a) There is an increase in the metabolism of thyroxine by placenta
(b) Hyperemesis gravidarum may be associated with thyrotoxicosis in pregnancy
(c) Levothyroxine replacement therapy dose should be decreased by 30?50% early in
pregnancy
(d) Impaired cognitive impairment may be seen in the offspring of mother with
subclinical hypothyroidism

59.
Which one of the following is NOT the criterion for the diagnosis of Diabetes mellitus?
(a) Symptoms of diabetes and random plasma sugar concentration ? 200 mg/dL
(b) HbA1c ? 6.5 %
(c) 2-hours plasma glucose ? 200 mg/dL during an oral glucose tolerance test
(d) Fasting plasma glucose ? 110 mg/dL
60.
A young lady presents in outdoor clinic with complaints of menstrual irregularity, weight
gain, hair loss, tiredness and weakness. What will be the investigation of choice?
(a) FSH & LH level
(b) Estrogen level
(c) Free T
3
and free T
4
level
(d) Free T
4
and TSH level







61.
Consider the following statements about erectile dysfunction in diabetic males:

1. It affects 60 % of males
2. Its common cause is an underlying neuro-vascular pathology
3. It may be aggravated by beta-adrenergic agonist drugs
4. Endocrine disorders like hyperprolactinemia may cause it

Which of the statements given above are true?
(a) 1 and 2
(b) 2 and 3
(c) 2 and 4
(d) 3 and 4
62.
All of the following can cause Hyperprolactinemia, EXCEPT:
(a) Craniopharyngioma
(b) Hyperthyroidism
(c) Stress
(d) Chronic renal failure
63.
All of the following can be seen in a case of thyrotoxicosis, EXCEPT:
(a) Atrial fibrillation
(b) Periodic paralysis
(c) Deafness
(d) Osteoporosis
64.
Which one of the following hormones acts via the receptor tyrosine kinase?
(a) FSH
(b) TSH
(c) IGF-1
(d) PTH
65.
Consider the following disorders as constituent of Multiple Endocrine Neoplasia Type 2a
(MEN 2a):

1. Primary Hyperparathyroidism
2. Pituitary tumours
3. Medullary carcinoma of thyroid
4. Pheochromocytoma

Which of the above are correct?:

(a) 1, 2 and 4
(b) 1, 2 and 3
(c) 1, 3 and 4
(d) 2, 3 and 4




66.
Consider the following statements with regard to Graves? ophthalmopathy :

1. Proptosis is often asymmetric and can even appear to be unilateral
2. It is a clinical diagnosis
3. Worsening of symptoms upon glucocorticoid withdrawal is common
4. Radiation therapy is very effective in treatment

Which of the above statements are correct?

(a) 1 and 2 only
(b) 1, 2 and 3
(c) 2, 3 and 4
(d) 1 and 4 only
67.
All of the following are common aetiologies of delirium EXCEPT:
(a) Hyperthyroidism
(b) Hypothyroidism
(c) Hyperparathyroidism
(d) Hypoparathyroidism
68.
Key anthropometric measurements important for evaluating the degree of obesity are:

(a) Weight, height and waist circumference
(b) Weight, height and hip circumference
(c) Weight, height and mid-thigh circumference
(d) Weight, height and chest circumference
69.
What is the minimum BMI (Body Mass Index) recommended for considering adjuvant
pharmacologic treatment in obesity with comorbidity?
(a) 23 kg/m
2

(b) 25 kg/m
2

(c) 27 kg/m
2

(d) 30 kg/m
2

70.
A young medical student while working in HIV ward got pricked accidently on the finger by
a syringe needle which was used to draw blood from a patient. Patient?s infection status is not
known. Which one of the following therapeutic interventions will be the best interest of this
student?
(a) To start Dolultegravir immediately
(b) To wash wound with soap and water and start Emtricitabine
(c) To give Dolutegravir + Tenofovir + Emtricitabine
(d) To give Dolutegravir + Tenofovir + Emtricitabine + HBV Immunoglobulin
71.
Cryptococcal infection is acquired through:
(a) Inhalation route
(b) Bite of Culex mosquito
(c) Direct skin contact
(d) Ingestion of spores

72.
Which one of the following is NOT appropriate treatment for Echinococcus granulosus and
hydatid disease?
(a) Albendazole 400 mg twice in a day for 3 months
(b) Diethylcarbamazine 2 mg/kg thrice in a day for 12 days
(c) PAIR (percutaneous puncture, aspiration, injection of scolicide, reaspiration)
(d) Praziquantel (20 mg/kg twice daily for 14 days)
73.
Which one of the following infections is NOT amenable to post-exposure prophylaxis with
specific immunoglobulins?
(a) Hepatitis B
(b) HIV
(c) Tetanus
(d) Rabies
74.
Echinocandins are a class of:
(a) Antiviral agents
(b) Antiparasitic agents
(c) Antitubercular agents
(d) Antifungal agents
75.
Recommended first time drug for initial treatment of tuberculosis include all EXCEPT:
(a) Isoniazid
(b) Rifampin
(c) Ethambutol
(d) Ethionamide
76.
The most frequent adverse reaction of significance among people treated for drug-susceptible
tuberculosis is:
(a) Colitis
(b) Hepatitis
(c) Cystitis
(d) Enteritis
77.
All adult patients in whom anti-tubercular treatment is being started should undergo baseline
assessment of:
(a) Complete blood count
(b) Liver function test
(c) Lipid profile
(d) Pulmonary function test
78.
A 30 year old male presents with fever, headache, anorexia, nausea, vomiting and diarrhoea.
On examination tongue is coated with abdominal tenderness, soft splenomegaly, relative
bradycardia and rose spots rash. The most likely diagnosis is:

(a) Malaria fever
(b) Enteric fever
(c) Leptospirosis
(d) Dengue fever

79.
The extrapulmonary sites most commonly involved in tuberculosis are all EXCEPT:
(a) Lymph node TB
(b) Genitourinary TB
(c) Skeletal TB
(d) Skull TB
80.
Which of the following is NOT a clinical feature of Botulism?
(a) Impaired alertness
(b) Dysphagia
(c) Diplopia
(d) Ptosis
81.
Post exposure prophylaxis (PEP) for HIV should contain:
(a) 2 antiretroviral drugs administered for 4 weeks
(b) 2 antiretroviral drugs administered for 6 weeks
(c) 3 antiretroviral drugs administered for 4 weeks
(d) 3 antiretroviral drugs administered for 6 weeks
82.
Paralytic rabies is characterized by:
(a) Maculopapular rash
(b) Flaccid muscle weakness
(c) Spastic muscle weakness
(d) Hyperexcitability and facial weakness
83.
All of the following are seen in Marasmus EXCEPT:

(a) Reduced triceps skinfold
(b) Reduced mid arm circumference
(c) Protein wasting
(d) Decreased body mass index
84.
A 20 year young lady presents with high grade fever for five days and palpable purpura over
extremities. She is found to be confused, with presence of neck stiffness. Which of the
following is the most likely diagnosis?
(a) Disseminated intravascular coagulation
(b) Acute meningococcemia
(c) Antiphospholipid antibody syndrome
(d) Thrombotic thrombocytopenic purpura
85.
Fibroblast Growth Factor 23 (FGF 23) is increased in which one of the following conditions?
(a) Osteomalacia
(b) Osteoporosis
(c) Paget?s disease
(d) Renal Osteodystrophy





86.
Which of the following crystals are deposited in a case of pseudogout?
(a) Monosodium urate
(b) Calcium phosphate
(c) Cholestrol crystals
(d) Calcium pyrophosphate dehydrate (CPPD)
87.
Which of the following features are associated with poor prognosis in Diffuse Cutaneous
Systemic Sclerosis?

1. Older age
2. A high gas transfer factor for carbon monoxide (TLCO)
3. Proteinuria
4. Diffuse skin disease

Select the correct answer using the code given below:
(a) 1, 2 and 4
(b) 3 and 4 only
(c) 1, 3 and 4
(d) 1, 2 and 3

88.
Which of the following statements regarding pegloticase are correct?

1. It is enzyme conjugate
2. It is useful in chronic Gout management
3. The main adverse effects are infusion reactions and development of antibodies

Select the correct answer using the code given below:
(a) 1 and 2 only
(b) 2 and 3 only
(c) 1 and 3 only
(d) 1, 2 and 3
89.
The most common cause of sensorineural hearing loss in adults is:
(a) Otosclerosis
(b) Meniere?s disease
(c) Presbycusis
(d) Meningitis
90.
A 50-year old male presents with dizziness. Which one of the following clinical findings does
NOT favour a central cause?
(a) Poor saccades
(b) Gaze evoked nystagmus
(c) Inhibition by visual fixation
(d) Presence of diplopia, dysarthria




91.
A young primigravida with 11 weeks pregnancy presents with sudden onset pain and
swelling of left lower limb. Homan?s sign is positive. Duplex ultrasonogram shows non-
compressibility of veins in the affected region, with absent flow. The drug of choice for
treatment is:
(a) Enoxaparin
(b) Amoxycillin?Clavulanic acid
(c) Warfarin
(d) Limb elevation only
92.
All of the following are clinical features of hypovolaemic shock EXCEPT:
(a) Cold clammy skin
(b) Bradycardia
(c) Hypotension
(d) Oliguria
93.
Which one of the following is most appropriate to describe classic heat stroke?
(a) Patients core body temperature rises above 44?C with headache, nausea and loss of
consciousness
(b) Patients core body temperature rises above 40 ?C with muscle tremors, confusion,
loss of consciousness and loss of sweating
(c) Patients core body temperature rises above 38 ?C with increased sweating,
tachypnoea, tachycardia and loss of consciousness
(d) Patients core body temperature is above 40 ?C with disorientation, increased
sweating, peripheral vasodilation, tachycardia and loss of consciousness
94.
Which of the following is NOT a priority in the initial management of shock from venomous
snake?

(a) Aggressive volume resuscitation
(b) Vasopressor support
(c) Anti-venom administration
(d) All of these are undertaken simultaneously

95.
Which one of the following is NOT an appropriate criterion for diagnosis of Systemic
Inflammatory Response Syndrome (SIRS)?
(a) Respiratory rate > 20/mm
(b) Heart rate > 90/min
(c) Total leucocyte count > 16 ? 10
9
per Litre
(d) Temperature > 38
0
C








FirstRanker.com - FirstRanker's Choice
Combined Medical Services Examination-2020
Paper-I
1.
A chronic alcoholic develops a paroxysm of palpitations after alcohol binge. Which of the
following Arrhythmia is most likely?
(a) Ventricular fibrillation
(b) Ventricular premature complex
(c) Atrial flutter
(d) Atrial fibrillation
2.
An elderly-man with history of Diabetes mellitus and Coronary Artery Disease comes for
follow-up, with complaints of muscle pains. Which one of the following drugs could be the
most likely cause?
(a) Aspirin
(b) Glimepiride
(c) Enalapril
(d) Atorvastatin
3.
A 40-year old lady comes to outdoor clinic with complaints of sudden onset chest pain. The
chest X-ray shows bilateral Pneumothorax. Examination reveals abnormalities of body
habitus-including long arms, legs and finger (arachnodactyly), scoliosis, high arched palate,
joint hypermobility, and a pansystolic murmur at cardiac apex. Which one of the following
diseases is most likely?
(a) Takayasu?s arteritis
(b) Raynaud?s syndrome
(c) Marfan?s syndrome
(d) Rheumatic heart disease-Mitral regurgitation
4.
All of the following are indications for treadmill testing/exercise-testing EXCEPT:
(a) To confirm the diagnosis of angina
(b) To evaluate stable angina
(c) To assess outcome after coronary revascularization
(d) To evaluate the treatment efficacy of antianginal drugs
5.
Kussmaul?s sign is present in all of the following conditions, EXCEPT:

(a) Massive Pulmonary Embolism
(b) Restrictive Cardiomyopathy
(c) Hypertrophic Cardiomyopathy
(d) Right Ventricular Infarction

6.
Which one of the following is NOT a common cause of atrial fibrillation?

(a) Mitral regurgitation
(b) Hypothyroidism
(c) Hypertension
(d) Acute myocardial infarction

7.
In an ECG recording, P wave is produced by:

(a) Atrial depolarisation
(b) Atrial repolarisation
(c) Ventricular depolarisation
(d) Ventricular repolarisation
8.
Which one of the following modalities is NOT used in the treatment of ventricular
tachycardia?

(a) DC Cardioversion
(b) Injection Lignocaine
(c) Injection Amiodarone
(d) Injection Adenosine
9.
Which one of the following drugs is a Direct Renin Inhibitor?

(a) Benedipine
(b) Azilsartan
(c) Aliskiren
(d) Lisinopril
10.
All of the following are causes of systolic hypertension with wide pulse pressure EXCEPT:

(a) Aortic regurgitation
(b) Thyrotoxicosis
(c) Patent ductus arteriosus
(d) Aortic stenosis
11.
Signet-ring sign on CT-chest is suggestive of:
(a) Bronchiectasis
(b) Active alveolitis
(c) Aspergilloma
(d) Sarcoidosis
12.
Oral corticosteroids are best introduced in the treatment of chronic bronchial asthma when it
is:
(a) Mild persistent
(b) Moderate persistent
(c) Severe persistent
(d) Very severe persistent
13.
An 18-year old male has insulin dependent diabetes, with malabsorption syndrome and
bilateral upper lobe bronchiectasis. A diagnostic work-up will include all EXCEPT:
(a) 24 hour fecal fat-estimation
(b) CT-chest
(c) Lung biopsy
(d) DNA sequencing study
14.
Which one of the following antigens is commonly associated with causation of Farmer?s
lung?
(a) Penicillium
(b) Actinomycetes
(c) Aspergillus
(d) Candida
15.
The Gene X-pert test used for MTB detection has the additional advantage of detection of
which of the following?

(a) INH resistance
(b) Rifampicin resistance
(c) Multi drug resistance
(d) Ethambutol resistance
16.
The most common cause of chronic type II respiratory failure is:
(a) Severe pneumonia
(b) Severe COPD
(c) Severe bronchial asthma
(d) Severe pulmonary thromboembolism
17.
The most frequent symptom in respiratory diseases is:
(a) Breathlessness
(b) Chest pain
(c) Hemoptysis
(d) Cough
18.
A 26-year old young lady attends medical emergency and is labelled as acute severe asthma
by the emergency physician. Which one of the following is unlikely to be a part of the
prescription?

(a) Albuterol
(b) Anti-leukotrienes
(c) Aminophylline
(d) Magnesium sulfate
19.
Consider the following statements with regard to respiratory examination:

1. Change in note, when patient phonates ?EEE? (Egophony) is characteristic of
interstitial fibrosis
2. Whispered pectoriloquy is characteristic of lung consolidation
3. Monophonic wheeze is characteristic of asthma
4. Hyper-resonant note on percussion is characteristic of pnemothorax

Which of the above statements are correct?
(a) 1 and 3
(b) 2 and 4 only
(c) 1, 2 and 4
(d) 2, 3 and 4
20.
All of the following are the causes of exudative Pleural Effusion, EXCEPT:
(a) SVC obstruction
(b) Fungal infection
(c) SLE
(d) Meig syndrome
21.
Which one of the following is NOT true about Ghrelin?
(a) It stimulates appetite
(b) It decreases gastric emptying
(c) It increases acid secretion
(d) Fasting increases its secretion
22.
Fecal elastase test is used for diagnosing which one of the following conditions?
(a) Lactose intolerance
(b) Mucosal inflammation
(c) Bile acid secretory defects
(d) Pancreatic dysfunction
23.
The recent classification system for listing a patient as a candidate for liver transplantation is:
(a) Child Pugh score
(b) APACHE score
(c) MELD score
(d) Metavir score
24.
The blood supply of liver consists of:
(a) 50 % hepatic artery and 50 % portal vein
(b) 80 % portal vein and 20 % hepatic artery
(c) 80 % hepatic artery and 20 % portal vein
(d) 70 % hepatic artery and 30 % portal vein
25.
All of the following conditions are associated with Glomerulonephritis with low complement
level, EXCEPT:
(a) Subacute bacterial endocarditis
(b) IgA nephropathy
(c) Systemic lupus erythematosus
(d) Cryoglobulinaemia













26.
Consider the following statements with regard to oral aphthous ulcers:

1. They are superficial and painful
2. They may occur in women just prior to menstruation
3. Oral glucocorticoids may be needed for the treatment of severe, recurrent cases
4. They are pre-malignant and progress over few years to squamous cell carcinoma of
the oral cavity

Which of the above statements are correct?

(a) 1 and 4 only
(b) 2 and 3 only
(c) 1, 2 and 3 only
(d) 1, 2, 3 and 4
27.
Which one of the following statements about Barrett?s Oesophagus is NOT correct?

(a) It is a pre-malignant condition
(b) Normal squamous cells lining the lower oesophagus are replaced by columnar
cells
(c) Normal columnar cells lining the lower oesophagus are replaced by squamous
cells
(d) It is an adaptive response to chronic gastro-oesophageal reflux

28.
The most common cause of acute hepatitis outbreaks in India is:

(a) Hepatitis E
(b) Hepatitis C
(c) Hepatitis B
(d) Hepatitis A

29.
Spontaneous oesophageal perforation after a bout of forceful vomiting or retching is
characteristically seen in which one of the following conditions?

(a) Menetrier?s disease
(b) Boerhaave?s syndrome
(c) Achalasia of the oesophagus
(d) Barrett?s oesophagus
30.
Eradication of helicobacter pylori infection may prove beneficial in the following extra-
gastric disorders EXCEPT:

(a) Unexplained vitamin B12 deficiency
(b) Idiopathic thrombocytopenic purpura
(c) Iron deficiency anaemia without gastro-intestinal bleeding
(d) Acute glomerulonephritis

31.
What is the target blood Hemoglobin level when treating anaemia in Stage-4 chronic kidney
disease patient?
(a) 8 to 10 gm/dL
(b) 10 to 12 gm/dL
(c) 12 to 13 gm/dL
(d) 13 to 14 gm/dL
32.
Which one of the following complications of chronic kidney disease is observed in patients
with low parathyroid hormone levels?
(a) Adynamic bone disease
(b) Tumoral calcinosis
(c) Osteitis fibrosa cystic
(d) Calciphylaxis
33.
All of the following statements about adult polycystic kidney disease are true, EXCEPT:
(a) It is inherited as autosomal dominant trait
(b) It is usually associated with marked proteinuria
(c) It is usually associated with systematic hypertension, from young age
(d) It is associated with Berry aneurysm
34.
Consider the following statements in relation to an adult patient:

1. Oliguria is defined as passage of urine less than 300 ml per day
2. Anuria is said to exist when less than 50 ml urine is passed per day
3. Polyuria is defined as urine volume in excess of 3 litre per day

Which of the statements given above are correct?
(a) 1 and 3 only
(b) 2 and 3 only
(c) 1 and 2 only
(d) 1, 2 and 3

35.
White cell casts in urine examination are strongly suggestive of:
(a) Nephritis
(b) Pyelonephritis
(c) Renal stone disease
(d) Papillitis
36.
A 13-year old boy presents with hematuria, oliguria, edema and hypertension. He has history
of sore throat two weeks prior to presentation. Laboratory investigations are remarkable for
low C
3
and increased titres of ASO and antiDNase. Which one of the following statements is
NOT correct about management for this condition?

(a) Renal biopsy is rarely required for making diagnosis
(b) Antibiotic treatment is given for streptococcal infection
(c) Treatment is largely supportive
(d) Immunosuppressants are to be used for crescentic glomerulonephritis
37.
Which one of the following statements is NOT correct regarding the risk factors for
nephrolithiasis?

(a) Higher dietary calcium increases risk
(b) Higher animal protein intake increases risk
(c) Higher dietary potassium intake lowers risk
(d) Lower urine citrate increases the risk
38.
A 60-year old patient is admitted in emergency with seizure, aphasia and altered sensorium.
He has history of fever and headache for the preceding 3 days. The CSF examination is
unremarkable. What would be the probable diagnosis?
(a) Multiple sclerosis
(b) Tubercular meningitis
(c) Pyogenic (bacterial) meningitis
(d) Viral encephalitis
39.
Drug of choice for the treatment of Trigeminal Neuralgia is:
(a) Carbamazepine
(b) Aceclofenac
(c) Thiamine
(d) Prednisdone
40.
Constructional skills is a function of which lobe?
(a) Dominant temporal lobe
(b) Non dominant parietal lobe
(c) Frontal lobe
(d) Non dominant temporal lobe

41.
Vertical gaze palsy with convergence retraction nystagmus is seen in:
(a) Weber syndrome
(b) Millard Gubler syndrome
(c) Claude syndrome
(d) Parinaud syndrome
42.
A patient on looking forward was found to have his right eye deviated downwards and
outwards with pupil dilated. He is suffering from:
(a) Left 3
rd
nerve palsy
(b) Left 6
th
nerve palsy
(c) Right 4
th
nerve palsy
(d) Right 3
rd
nerve palsy
43.
All of the following are features of Cauda Equina syndrome EXCEPT:

(a) Low back pain
(b) Lower limb areflexia
(c) Loss of bladder function
(d) Extensor plantar response

44.
A 68-year old male presented in OPD with complaints of progressive small handwriting. On
examination, he had resting tremor, bradykinesia, rigidity and postural instability. This
patient is suffering most likely from:

(a) Parkinson?s disease
(b) Vascular dementia
(c) Alzheimer?s disease
(d) Frunto temporal dementia
45.
An elderly patient presents with abnormal gait. He has a wide base freezing gait with
imbalance, comprising short strides, shuffling along the floor and difficulty with starts and
turns. Heal-Shin test is normal. Which one of the following is most likely disorder?
(a) Cerebellar ataxia
(b) Sensory ataxia
(c) Frontal gait disorder
(d) Parkinson?s disease
46.
Cushing reflex, seen in conditions of raised intra cranial pressure includes all of the following
EXCEPT:
(a) Bradycardia
(b) Hypertension
(c) Irregular respiration
(d) Hypothermia

47.
A 35 year old man presents with history of low grade fever and headache for last five weeks.
Clinical examination is remarkable for signs of meningismus. He undergoes lumbar puncture
(LP) and a day later reports worsening of headache. Which one of the following features is
NOT consistent with diagnosis of ?Post-LP? headache?

(a) Post-LP headache usually begins within 48 hours
(b) Post-LP headache worsens in sitting position
(c) Post-LP headache is most severe upon waking up
(d) Post-LP headache may improve with caffeine intake
48.
Which one of the following investigations is NOT appropriate in a case of recurrent
thrombosis?
(a) Antiphospholipid antibodies
(b) Bcr-Abl assay
(c) Protein C and S assays
(d) Antithrombin level
49.
All of the following are causes of reactive thrombocytosis, EXCEPT:
(a) Hemolytic anemia
(b) Megaloblastic anemia
(c) Post splenectomy
(d) Chronic inflammatory disorders


50.
Cryoprecipitate cannot be used for treatment of which one of the following conditions?
(a) Von Willebrand disease
(b) Hypofibrinogenemia
(c) Hemophilia?B
(d) Hemphilia?A
51.
Pyruvate kinase deficiency results in deficiency of ATP production and a chronic haemolytic
anaemia. The disorder is inherited as an:

(a) Autosomal recessive trait
(b) X-linked recessive trait
(c) Autosomal dominant trait
(d) X-linked dominant trait
52.
Sickle cell disease may be associated with any of the following EXCEPT:

(a) Acute chest syndrome
(b) Aplastic crisis
(c) Splenic sequestration crisis
(d) Conn?s syndrome

53.
Fever with splenomegaly and lymphadenopathy can be seen in
(a) Infectious mononucleosis
(b) Chronic leukaemia
(c) Both infectious mononucleosis and chronic leukaemia
(d) Neither infectious mononucleosis nor chronic leukaemia
54.
Every individual carries four alpha gene alleles. Deletion of three alleles leads to
development of:
(a) Haemoglobin X disease
(b) Haemoglobin H disease
(c) Haemoglobin F disease
(d) Hydrops fetalis
55.
Which one of the following is NOT true regarding Chronic Myeloid Leukaemia (CML)?
(a) It is a clonal malignancy of haematopoetic stem
(b) Risk of developing CML is increased in monozygotic turns
(c) CML is defined by presence of BCR-ABL 1 fusion gene
(d) 10 year survival with TKI therapy is 85%









56.
Diagnostic criteria of multiple myeloma includes which of the following?

1. Increased malignant plasma cells in the bone marrow
2. Serum and/or urinary M protein
3. Skeletal lytic lesions
4. Skeletal blastic lesions

Select the correct answer using the code given below:
(a) 1, 2 and 3
(b) 1, 3 and 4
(c) 1, 2 and 4
(d) 2, 3 and 4
57.
The most predominant type of serum immunoglobulin involved in patients with plasma cell
disorder is:
(a) Ig G
(b) Ig A
(c) Ig D
(d) Light chains
58.
Which one of the following statements is NOT true regarding the thyroid function/disorder in
pregnancy?
(a) There is an increase in the metabolism of thyroxine by placenta
(b) Hyperemesis gravidarum may be associated with thyrotoxicosis in pregnancy
(c) Levothyroxine replacement therapy dose should be decreased by 30?50% early in
pregnancy
(d) Impaired cognitive impairment may be seen in the offspring of mother with
subclinical hypothyroidism

59.
Which one of the following is NOT the criterion for the diagnosis of Diabetes mellitus?
(a) Symptoms of diabetes and random plasma sugar concentration ? 200 mg/dL
(b) HbA1c ? 6.5 %
(c) 2-hours plasma glucose ? 200 mg/dL during an oral glucose tolerance test
(d) Fasting plasma glucose ? 110 mg/dL
60.
A young lady presents in outdoor clinic with complaints of menstrual irregularity, weight
gain, hair loss, tiredness and weakness. What will be the investigation of choice?
(a) FSH & LH level
(b) Estrogen level
(c) Free T
3
and free T
4
level
(d) Free T
4
and TSH level







61.
Consider the following statements about erectile dysfunction in diabetic males:

1. It affects 60 % of males
2. Its common cause is an underlying neuro-vascular pathology
3. It may be aggravated by beta-adrenergic agonist drugs
4. Endocrine disorders like hyperprolactinemia may cause it

Which of the statements given above are true?
(a) 1 and 2
(b) 2 and 3
(c) 2 and 4
(d) 3 and 4
62.
All of the following can cause Hyperprolactinemia, EXCEPT:
(a) Craniopharyngioma
(b) Hyperthyroidism
(c) Stress
(d) Chronic renal failure
63.
All of the following can be seen in a case of thyrotoxicosis, EXCEPT:
(a) Atrial fibrillation
(b) Periodic paralysis
(c) Deafness
(d) Osteoporosis
64.
Which one of the following hormones acts via the receptor tyrosine kinase?
(a) FSH
(b) TSH
(c) IGF-1
(d) PTH
65.
Consider the following disorders as constituent of Multiple Endocrine Neoplasia Type 2a
(MEN 2a):

1. Primary Hyperparathyroidism
2. Pituitary tumours
3. Medullary carcinoma of thyroid
4. Pheochromocytoma

Which of the above are correct?:

(a) 1, 2 and 4
(b) 1, 2 and 3
(c) 1, 3 and 4
(d) 2, 3 and 4




66.
Consider the following statements with regard to Graves? ophthalmopathy :

1. Proptosis is often asymmetric and can even appear to be unilateral
2. It is a clinical diagnosis
3. Worsening of symptoms upon glucocorticoid withdrawal is common
4. Radiation therapy is very effective in treatment

Which of the above statements are correct?

(a) 1 and 2 only
(b) 1, 2 and 3
(c) 2, 3 and 4
(d) 1 and 4 only
67.
All of the following are common aetiologies of delirium EXCEPT:
(a) Hyperthyroidism
(b) Hypothyroidism
(c) Hyperparathyroidism
(d) Hypoparathyroidism
68.
Key anthropometric measurements important for evaluating the degree of obesity are:

(a) Weight, height and waist circumference
(b) Weight, height and hip circumference
(c) Weight, height and mid-thigh circumference
(d) Weight, height and chest circumference
69.
What is the minimum BMI (Body Mass Index) recommended for considering adjuvant
pharmacologic treatment in obesity with comorbidity?
(a) 23 kg/m
2

(b) 25 kg/m
2

(c) 27 kg/m
2

(d) 30 kg/m
2

70.
A young medical student while working in HIV ward got pricked accidently on the finger by
a syringe needle which was used to draw blood from a patient. Patient?s infection status is not
known. Which one of the following therapeutic interventions will be the best interest of this
student?
(a) To start Dolultegravir immediately
(b) To wash wound with soap and water and start Emtricitabine
(c) To give Dolutegravir + Tenofovir + Emtricitabine
(d) To give Dolutegravir + Tenofovir + Emtricitabine + HBV Immunoglobulin
71.
Cryptococcal infection is acquired through:
(a) Inhalation route
(b) Bite of Culex mosquito
(c) Direct skin contact
(d) Ingestion of spores

72.
Which one of the following is NOT appropriate treatment for Echinococcus granulosus and
hydatid disease?
(a) Albendazole 400 mg twice in a day for 3 months
(b) Diethylcarbamazine 2 mg/kg thrice in a day for 12 days
(c) PAIR (percutaneous puncture, aspiration, injection of scolicide, reaspiration)
(d) Praziquantel (20 mg/kg twice daily for 14 days)
73.
Which one of the following infections is NOT amenable to post-exposure prophylaxis with
specific immunoglobulins?
(a) Hepatitis B
(b) HIV
(c) Tetanus
(d) Rabies
74.
Echinocandins are a class of:
(a) Antiviral agents
(b) Antiparasitic agents
(c) Antitubercular agents
(d) Antifungal agents
75.
Recommended first time drug for initial treatment of tuberculosis include all EXCEPT:
(a) Isoniazid
(b) Rifampin
(c) Ethambutol
(d) Ethionamide
76.
The most frequent adverse reaction of significance among people treated for drug-susceptible
tuberculosis is:
(a) Colitis
(b) Hepatitis
(c) Cystitis
(d) Enteritis
77.
All adult patients in whom anti-tubercular treatment is being started should undergo baseline
assessment of:
(a) Complete blood count
(b) Liver function test
(c) Lipid profile
(d) Pulmonary function test
78.
A 30 year old male presents with fever, headache, anorexia, nausea, vomiting and diarrhoea.
On examination tongue is coated with abdominal tenderness, soft splenomegaly, relative
bradycardia and rose spots rash. The most likely diagnosis is:

(a) Malaria fever
(b) Enteric fever
(c) Leptospirosis
(d) Dengue fever

79.
The extrapulmonary sites most commonly involved in tuberculosis are all EXCEPT:
(a) Lymph node TB
(b) Genitourinary TB
(c) Skeletal TB
(d) Skull TB
80.
Which of the following is NOT a clinical feature of Botulism?
(a) Impaired alertness
(b) Dysphagia
(c) Diplopia
(d) Ptosis
81.
Post exposure prophylaxis (PEP) for HIV should contain:
(a) 2 antiretroviral drugs administered for 4 weeks
(b) 2 antiretroviral drugs administered for 6 weeks
(c) 3 antiretroviral drugs administered for 4 weeks
(d) 3 antiretroviral drugs administered for 6 weeks
82.
Paralytic rabies is characterized by:
(a) Maculopapular rash
(b) Flaccid muscle weakness
(c) Spastic muscle weakness
(d) Hyperexcitability and facial weakness
83.
All of the following are seen in Marasmus EXCEPT:

(a) Reduced triceps skinfold
(b) Reduced mid arm circumference
(c) Protein wasting
(d) Decreased body mass index
84.
A 20 year young lady presents with high grade fever for five days and palpable purpura over
extremities. She is found to be confused, with presence of neck stiffness. Which of the
following is the most likely diagnosis?
(a) Disseminated intravascular coagulation
(b) Acute meningococcemia
(c) Antiphospholipid antibody syndrome
(d) Thrombotic thrombocytopenic purpura
85.
Fibroblast Growth Factor 23 (FGF 23) is increased in which one of the following conditions?
(a) Osteomalacia
(b) Osteoporosis
(c) Paget?s disease
(d) Renal Osteodystrophy





86.
Which of the following crystals are deposited in a case of pseudogout?
(a) Monosodium urate
(b) Calcium phosphate
(c) Cholestrol crystals
(d) Calcium pyrophosphate dehydrate (CPPD)
87.
Which of the following features are associated with poor prognosis in Diffuse Cutaneous
Systemic Sclerosis?

1. Older age
2. A high gas transfer factor for carbon monoxide (TLCO)
3. Proteinuria
4. Diffuse skin disease

Select the correct answer using the code given below:
(a) 1, 2 and 4
(b) 3 and 4 only
(c) 1, 3 and 4
(d) 1, 2 and 3

88.
Which of the following statements regarding pegloticase are correct?

1. It is enzyme conjugate
2. It is useful in chronic Gout management
3. The main adverse effects are infusion reactions and development of antibodies

Select the correct answer using the code given below:
(a) 1 and 2 only
(b) 2 and 3 only
(c) 1 and 3 only
(d) 1, 2 and 3
89.
The most common cause of sensorineural hearing loss in adults is:
(a) Otosclerosis
(b) Meniere?s disease
(c) Presbycusis
(d) Meningitis
90.
A 50-year old male presents with dizziness. Which one of the following clinical findings does
NOT favour a central cause?
(a) Poor saccades
(b) Gaze evoked nystagmus
(c) Inhibition by visual fixation
(d) Presence of diplopia, dysarthria




91.
A young primigravida with 11 weeks pregnancy presents with sudden onset pain and
swelling of left lower limb. Homan?s sign is positive. Duplex ultrasonogram shows non-
compressibility of veins in the affected region, with absent flow. The drug of choice for
treatment is:
(a) Enoxaparin
(b) Amoxycillin?Clavulanic acid
(c) Warfarin
(d) Limb elevation only
92.
All of the following are clinical features of hypovolaemic shock EXCEPT:
(a) Cold clammy skin
(b) Bradycardia
(c) Hypotension
(d) Oliguria
93.
Which one of the following is most appropriate to describe classic heat stroke?
(a) Patients core body temperature rises above 44?C with headache, nausea and loss of
consciousness
(b) Patients core body temperature rises above 40 ?C with muscle tremors, confusion,
loss of consciousness and loss of sweating
(c) Patients core body temperature rises above 38 ?C with increased sweating,
tachypnoea, tachycardia and loss of consciousness
(d) Patients core body temperature is above 40 ?C with disorientation, increased
sweating, peripheral vasodilation, tachycardia and loss of consciousness
94.
Which of the following is NOT a priority in the initial management of shock from venomous
snake?

(a) Aggressive volume resuscitation
(b) Vasopressor support
(c) Anti-venom administration
(d) All of these are undertaken simultaneously

95.
Which one of the following is NOT an appropriate criterion for diagnosis of Systemic
Inflammatory Response Syndrome (SIRS)?
(a) Respiratory rate > 20/mm
(b) Heart rate > 90/min
(c) Total leucocyte count > 16 ? 10
9
per Litre
(d) Temperature > 38
0
C








96.
All of the following predispose to Acute Respiratory Distress Syndrome EXCEPT:
(a) Acute Pancreatitis
(b) Toxic gas inhalation
(c) Aspiration of gastric contents
(d) Severe Bronchial asthma
97.
An 8 year old child has consumed a few tablets of a drug that were being taken by his mother
on a regular basis. Following this the child develops gastric necrosis, acidosis, shock and
hepatic necrosis. Which is the most likely drug?
(a) Multivitamins
(b) Calcium carbonate
(c) Folic acid
(d) Iron
98.
A newborn weighing 1.5 kg at birth presents at 6 hours of age with lethargy, weak and high
pitched cry and difficulty in feeding. What is the next step in management?

(a) Estimation of blood sugar
(b) Estimation of serum calcium
(c) Supplementary oxygen
(d) Bedside electrocardiogram

99.
In a child with acute organophosphorus poisoning, which one of the following drugs will act
as anti-sialogogue and peripheral parasympatholytic agent?

(a) Carbamates
(b) Atropine
(c) Benzodiazepine
(d) Pralidoxime
100.
A 6-year old child needs to undergo a central Catheter placement for total parental nutrition.
The procedure is likely to evoke pain and anxiety. Which of the following is the most
appropriate drug for sedation and analgesia in this child?

(a) Chloral hydrate
(b) Midozolam
(c) Propofol
(d) Ketamine
101.
Which one of the following is the drug of choice in a child presenting with supraventricular
tachycardia?

(a) Atropine
(b) Lidocaine
(c) Adenosine
(d) Nalaxone


FirstRanker.com - FirstRanker's Choice
Combined Medical Services Examination-2020
Paper-I
1.
A chronic alcoholic develops a paroxysm of palpitations after alcohol binge. Which of the
following Arrhythmia is most likely?
(a) Ventricular fibrillation
(b) Ventricular premature complex
(c) Atrial flutter
(d) Atrial fibrillation
2.
An elderly-man with history of Diabetes mellitus and Coronary Artery Disease comes for
follow-up, with complaints of muscle pains. Which one of the following drugs could be the
most likely cause?
(a) Aspirin
(b) Glimepiride
(c) Enalapril
(d) Atorvastatin
3.
A 40-year old lady comes to outdoor clinic with complaints of sudden onset chest pain. The
chest X-ray shows bilateral Pneumothorax. Examination reveals abnormalities of body
habitus-including long arms, legs and finger (arachnodactyly), scoliosis, high arched palate,
joint hypermobility, and a pansystolic murmur at cardiac apex. Which one of the following
diseases is most likely?
(a) Takayasu?s arteritis
(b) Raynaud?s syndrome
(c) Marfan?s syndrome
(d) Rheumatic heart disease-Mitral regurgitation
4.
All of the following are indications for treadmill testing/exercise-testing EXCEPT:
(a) To confirm the diagnosis of angina
(b) To evaluate stable angina
(c) To assess outcome after coronary revascularization
(d) To evaluate the treatment efficacy of antianginal drugs
5.
Kussmaul?s sign is present in all of the following conditions, EXCEPT:

(a) Massive Pulmonary Embolism
(b) Restrictive Cardiomyopathy
(c) Hypertrophic Cardiomyopathy
(d) Right Ventricular Infarction

6.
Which one of the following is NOT a common cause of atrial fibrillation?

(a) Mitral regurgitation
(b) Hypothyroidism
(c) Hypertension
(d) Acute myocardial infarction

7.
In an ECG recording, P wave is produced by:

(a) Atrial depolarisation
(b) Atrial repolarisation
(c) Ventricular depolarisation
(d) Ventricular repolarisation
8.
Which one of the following modalities is NOT used in the treatment of ventricular
tachycardia?

(a) DC Cardioversion
(b) Injection Lignocaine
(c) Injection Amiodarone
(d) Injection Adenosine
9.
Which one of the following drugs is a Direct Renin Inhibitor?

(a) Benedipine
(b) Azilsartan
(c) Aliskiren
(d) Lisinopril
10.
All of the following are causes of systolic hypertension with wide pulse pressure EXCEPT:

(a) Aortic regurgitation
(b) Thyrotoxicosis
(c) Patent ductus arteriosus
(d) Aortic stenosis
11.
Signet-ring sign on CT-chest is suggestive of:
(a) Bronchiectasis
(b) Active alveolitis
(c) Aspergilloma
(d) Sarcoidosis
12.
Oral corticosteroids are best introduced in the treatment of chronic bronchial asthma when it
is:
(a) Mild persistent
(b) Moderate persistent
(c) Severe persistent
(d) Very severe persistent
13.
An 18-year old male has insulin dependent diabetes, with malabsorption syndrome and
bilateral upper lobe bronchiectasis. A diagnostic work-up will include all EXCEPT:
(a) 24 hour fecal fat-estimation
(b) CT-chest
(c) Lung biopsy
(d) DNA sequencing study
14.
Which one of the following antigens is commonly associated with causation of Farmer?s
lung?
(a) Penicillium
(b) Actinomycetes
(c) Aspergillus
(d) Candida
15.
The Gene X-pert test used for MTB detection has the additional advantage of detection of
which of the following?

(a) INH resistance
(b) Rifampicin resistance
(c) Multi drug resistance
(d) Ethambutol resistance
16.
The most common cause of chronic type II respiratory failure is:
(a) Severe pneumonia
(b) Severe COPD
(c) Severe bronchial asthma
(d) Severe pulmonary thromboembolism
17.
The most frequent symptom in respiratory diseases is:
(a) Breathlessness
(b) Chest pain
(c) Hemoptysis
(d) Cough
18.
A 26-year old young lady attends medical emergency and is labelled as acute severe asthma
by the emergency physician. Which one of the following is unlikely to be a part of the
prescription?

(a) Albuterol
(b) Anti-leukotrienes
(c) Aminophylline
(d) Magnesium sulfate
19.
Consider the following statements with regard to respiratory examination:

1. Change in note, when patient phonates ?EEE? (Egophony) is characteristic of
interstitial fibrosis
2. Whispered pectoriloquy is characteristic of lung consolidation
3. Monophonic wheeze is characteristic of asthma
4. Hyper-resonant note on percussion is characteristic of pnemothorax

Which of the above statements are correct?
(a) 1 and 3
(b) 2 and 4 only
(c) 1, 2 and 4
(d) 2, 3 and 4
20.
All of the following are the causes of exudative Pleural Effusion, EXCEPT:
(a) SVC obstruction
(b) Fungal infection
(c) SLE
(d) Meig syndrome
21.
Which one of the following is NOT true about Ghrelin?
(a) It stimulates appetite
(b) It decreases gastric emptying
(c) It increases acid secretion
(d) Fasting increases its secretion
22.
Fecal elastase test is used for diagnosing which one of the following conditions?
(a) Lactose intolerance
(b) Mucosal inflammation
(c) Bile acid secretory defects
(d) Pancreatic dysfunction
23.
The recent classification system for listing a patient as a candidate for liver transplantation is:
(a) Child Pugh score
(b) APACHE score
(c) MELD score
(d) Metavir score
24.
The blood supply of liver consists of:
(a) 50 % hepatic artery and 50 % portal vein
(b) 80 % portal vein and 20 % hepatic artery
(c) 80 % hepatic artery and 20 % portal vein
(d) 70 % hepatic artery and 30 % portal vein
25.
All of the following conditions are associated with Glomerulonephritis with low complement
level, EXCEPT:
(a) Subacute bacterial endocarditis
(b) IgA nephropathy
(c) Systemic lupus erythematosus
(d) Cryoglobulinaemia













26.
Consider the following statements with regard to oral aphthous ulcers:

1. They are superficial and painful
2. They may occur in women just prior to menstruation
3. Oral glucocorticoids may be needed for the treatment of severe, recurrent cases
4. They are pre-malignant and progress over few years to squamous cell carcinoma of
the oral cavity

Which of the above statements are correct?

(a) 1 and 4 only
(b) 2 and 3 only
(c) 1, 2 and 3 only
(d) 1, 2, 3 and 4
27.
Which one of the following statements about Barrett?s Oesophagus is NOT correct?

(a) It is a pre-malignant condition
(b) Normal squamous cells lining the lower oesophagus are replaced by columnar
cells
(c) Normal columnar cells lining the lower oesophagus are replaced by squamous
cells
(d) It is an adaptive response to chronic gastro-oesophageal reflux

28.
The most common cause of acute hepatitis outbreaks in India is:

(a) Hepatitis E
(b) Hepatitis C
(c) Hepatitis B
(d) Hepatitis A

29.
Spontaneous oesophageal perforation after a bout of forceful vomiting or retching is
characteristically seen in which one of the following conditions?

(a) Menetrier?s disease
(b) Boerhaave?s syndrome
(c) Achalasia of the oesophagus
(d) Barrett?s oesophagus
30.
Eradication of helicobacter pylori infection may prove beneficial in the following extra-
gastric disorders EXCEPT:

(a) Unexplained vitamin B12 deficiency
(b) Idiopathic thrombocytopenic purpura
(c) Iron deficiency anaemia without gastro-intestinal bleeding
(d) Acute glomerulonephritis

31.
What is the target blood Hemoglobin level when treating anaemia in Stage-4 chronic kidney
disease patient?
(a) 8 to 10 gm/dL
(b) 10 to 12 gm/dL
(c) 12 to 13 gm/dL
(d) 13 to 14 gm/dL
32.
Which one of the following complications of chronic kidney disease is observed in patients
with low parathyroid hormone levels?
(a) Adynamic bone disease
(b) Tumoral calcinosis
(c) Osteitis fibrosa cystic
(d) Calciphylaxis
33.
All of the following statements about adult polycystic kidney disease are true, EXCEPT:
(a) It is inherited as autosomal dominant trait
(b) It is usually associated with marked proteinuria
(c) It is usually associated with systematic hypertension, from young age
(d) It is associated with Berry aneurysm
34.
Consider the following statements in relation to an adult patient:

1. Oliguria is defined as passage of urine less than 300 ml per day
2. Anuria is said to exist when less than 50 ml urine is passed per day
3. Polyuria is defined as urine volume in excess of 3 litre per day

Which of the statements given above are correct?
(a) 1 and 3 only
(b) 2 and 3 only
(c) 1 and 2 only
(d) 1, 2 and 3

35.
White cell casts in urine examination are strongly suggestive of:
(a) Nephritis
(b) Pyelonephritis
(c) Renal stone disease
(d) Papillitis
36.
A 13-year old boy presents with hematuria, oliguria, edema and hypertension. He has history
of sore throat two weeks prior to presentation. Laboratory investigations are remarkable for
low C
3
and increased titres of ASO and antiDNase. Which one of the following statements is
NOT correct about management for this condition?

(a) Renal biopsy is rarely required for making diagnosis
(b) Antibiotic treatment is given for streptococcal infection
(c) Treatment is largely supportive
(d) Immunosuppressants are to be used for crescentic glomerulonephritis
37.
Which one of the following statements is NOT correct regarding the risk factors for
nephrolithiasis?

(a) Higher dietary calcium increases risk
(b) Higher animal protein intake increases risk
(c) Higher dietary potassium intake lowers risk
(d) Lower urine citrate increases the risk
38.
A 60-year old patient is admitted in emergency with seizure, aphasia and altered sensorium.
He has history of fever and headache for the preceding 3 days. The CSF examination is
unremarkable. What would be the probable diagnosis?
(a) Multiple sclerosis
(b) Tubercular meningitis
(c) Pyogenic (bacterial) meningitis
(d) Viral encephalitis
39.
Drug of choice for the treatment of Trigeminal Neuralgia is:
(a) Carbamazepine
(b) Aceclofenac
(c) Thiamine
(d) Prednisdone
40.
Constructional skills is a function of which lobe?
(a) Dominant temporal lobe
(b) Non dominant parietal lobe
(c) Frontal lobe
(d) Non dominant temporal lobe

41.
Vertical gaze palsy with convergence retraction nystagmus is seen in:
(a) Weber syndrome
(b) Millard Gubler syndrome
(c) Claude syndrome
(d) Parinaud syndrome
42.
A patient on looking forward was found to have his right eye deviated downwards and
outwards with pupil dilated. He is suffering from:
(a) Left 3
rd
nerve palsy
(b) Left 6
th
nerve palsy
(c) Right 4
th
nerve palsy
(d) Right 3
rd
nerve palsy
43.
All of the following are features of Cauda Equina syndrome EXCEPT:

(a) Low back pain
(b) Lower limb areflexia
(c) Loss of bladder function
(d) Extensor plantar response

44.
A 68-year old male presented in OPD with complaints of progressive small handwriting. On
examination, he had resting tremor, bradykinesia, rigidity and postural instability. This
patient is suffering most likely from:

(a) Parkinson?s disease
(b) Vascular dementia
(c) Alzheimer?s disease
(d) Frunto temporal dementia
45.
An elderly patient presents with abnormal gait. He has a wide base freezing gait with
imbalance, comprising short strides, shuffling along the floor and difficulty with starts and
turns. Heal-Shin test is normal. Which one of the following is most likely disorder?
(a) Cerebellar ataxia
(b) Sensory ataxia
(c) Frontal gait disorder
(d) Parkinson?s disease
46.
Cushing reflex, seen in conditions of raised intra cranial pressure includes all of the following
EXCEPT:
(a) Bradycardia
(b) Hypertension
(c) Irregular respiration
(d) Hypothermia

47.
A 35 year old man presents with history of low grade fever and headache for last five weeks.
Clinical examination is remarkable for signs of meningismus. He undergoes lumbar puncture
(LP) and a day later reports worsening of headache. Which one of the following features is
NOT consistent with diagnosis of ?Post-LP? headache?

(a) Post-LP headache usually begins within 48 hours
(b) Post-LP headache worsens in sitting position
(c) Post-LP headache is most severe upon waking up
(d) Post-LP headache may improve with caffeine intake
48.
Which one of the following investigations is NOT appropriate in a case of recurrent
thrombosis?
(a) Antiphospholipid antibodies
(b) Bcr-Abl assay
(c) Protein C and S assays
(d) Antithrombin level
49.
All of the following are causes of reactive thrombocytosis, EXCEPT:
(a) Hemolytic anemia
(b) Megaloblastic anemia
(c) Post splenectomy
(d) Chronic inflammatory disorders


50.
Cryoprecipitate cannot be used for treatment of which one of the following conditions?
(a) Von Willebrand disease
(b) Hypofibrinogenemia
(c) Hemophilia?B
(d) Hemphilia?A
51.
Pyruvate kinase deficiency results in deficiency of ATP production and a chronic haemolytic
anaemia. The disorder is inherited as an:

(a) Autosomal recessive trait
(b) X-linked recessive trait
(c) Autosomal dominant trait
(d) X-linked dominant trait
52.
Sickle cell disease may be associated with any of the following EXCEPT:

(a) Acute chest syndrome
(b) Aplastic crisis
(c) Splenic sequestration crisis
(d) Conn?s syndrome

53.
Fever with splenomegaly and lymphadenopathy can be seen in
(a) Infectious mononucleosis
(b) Chronic leukaemia
(c) Both infectious mononucleosis and chronic leukaemia
(d) Neither infectious mononucleosis nor chronic leukaemia
54.
Every individual carries four alpha gene alleles. Deletion of three alleles leads to
development of:
(a) Haemoglobin X disease
(b) Haemoglobin H disease
(c) Haemoglobin F disease
(d) Hydrops fetalis
55.
Which one of the following is NOT true regarding Chronic Myeloid Leukaemia (CML)?
(a) It is a clonal malignancy of haematopoetic stem
(b) Risk of developing CML is increased in monozygotic turns
(c) CML is defined by presence of BCR-ABL 1 fusion gene
(d) 10 year survival with TKI therapy is 85%









56.
Diagnostic criteria of multiple myeloma includes which of the following?

1. Increased malignant plasma cells in the bone marrow
2. Serum and/or urinary M protein
3. Skeletal lytic lesions
4. Skeletal blastic lesions

Select the correct answer using the code given below:
(a) 1, 2 and 3
(b) 1, 3 and 4
(c) 1, 2 and 4
(d) 2, 3 and 4
57.
The most predominant type of serum immunoglobulin involved in patients with plasma cell
disorder is:
(a) Ig G
(b) Ig A
(c) Ig D
(d) Light chains
58.
Which one of the following statements is NOT true regarding the thyroid function/disorder in
pregnancy?
(a) There is an increase in the metabolism of thyroxine by placenta
(b) Hyperemesis gravidarum may be associated with thyrotoxicosis in pregnancy
(c) Levothyroxine replacement therapy dose should be decreased by 30?50% early in
pregnancy
(d) Impaired cognitive impairment may be seen in the offspring of mother with
subclinical hypothyroidism

59.
Which one of the following is NOT the criterion for the diagnosis of Diabetes mellitus?
(a) Symptoms of diabetes and random plasma sugar concentration ? 200 mg/dL
(b) HbA1c ? 6.5 %
(c) 2-hours plasma glucose ? 200 mg/dL during an oral glucose tolerance test
(d) Fasting plasma glucose ? 110 mg/dL
60.
A young lady presents in outdoor clinic with complaints of menstrual irregularity, weight
gain, hair loss, tiredness and weakness. What will be the investigation of choice?
(a) FSH & LH level
(b) Estrogen level
(c) Free T
3
and free T
4
level
(d) Free T
4
and TSH level







61.
Consider the following statements about erectile dysfunction in diabetic males:

1. It affects 60 % of males
2. Its common cause is an underlying neuro-vascular pathology
3. It may be aggravated by beta-adrenergic agonist drugs
4. Endocrine disorders like hyperprolactinemia may cause it

Which of the statements given above are true?
(a) 1 and 2
(b) 2 and 3
(c) 2 and 4
(d) 3 and 4
62.
All of the following can cause Hyperprolactinemia, EXCEPT:
(a) Craniopharyngioma
(b) Hyperthyroidism
(c) Stress
(d) Chronic renal failure
63.
All of the following can be seen in a case of thyrotoxicosis, EXCEPT:
(a) Atrial fibrillation
(b) Periodic paralysis
(c) Deafness
(d) Osteoporosis
64.
Which one of the following hormones acts via the receptor tyrosine kinase?
(a) FSH
(b) TSH
(c) IGF-1
(d) PTH
65.
Consider the following disorders as constituent of Multiple Endocrine Neoplasia Type 2a
(MEN 2a):

1. Primary Hyperparathyroidism
2. Pituitary tumours
3. Medullary carcinoma of thyroid
4. Pheochromocytoma

Which of the above are correct?:

(a) 1, 2 and 4
(b) 1, 2 and 3
(c) 1, 3 and 4
(d) 2, 3 and 4




66.
Consider the following statements with regard to Graves? ophthalmopathy :

1. Proptosis is often asymmetric and can even appear to be unilateral
2. It is a clinical diagnosis
3. Worsening of symptoms upon glucocorticoid withdrawal is common
4. Radiation therapy is very effective in treatment

Which of the above statements are correct?

(a) 1 and 2 only
(b) 1, 2 and 3
(c) 2, 3 and 4
(d) 1 and 4 only
67.
All of the following are common aetiologies of delirium EXCEPT:
(a) Hyperthyroidism
(b) Hypothyroidism
(c) Hyperparathyroidism
(d) Hypoparathyroidism
68.
Key anthropometric measurements important for evaluating the degree of obesity are:

(a) Weight, height and waist circumference
(b) Weight, height and hip circumference
(c) Weight, height and mid-thigh circumference
(d) Weight, height and chest circumference
69.
What is the minimum BMI (Body Mass Index) recommended for considering adjuvant
pharmacologic treatment in obesity with comorbidity?
(a) 23 kg/m
2

(b) 25 kg/m
2

(c) 27 kg/m
2

(d) 30 kg/m
2

70.
A young medical student while working in HIV ward got pricked accidently on the finger by
a syringe needle which was used to draw blood from a patient. Patient?s infection status is not
known. Which one of the following therapeutic interventions will be the best interest of this
student?
(a) To start Dolultegravir immediately
(b) To wash wound with soap and water and start Emtricitabine
(c) To give Dolutegravir + Tenofovir + Emtricitabine
(d) To give Dolutegravir + Tenofovir + Emtricitabine + HBV Immunoglobulin
71.
Cryptococcal infection is acquired through:
(a) Inhalation route
(b) Bite of Culex mosquito
(c) Direct skin contact
(d) Ingestion of spores

72.
Which one of the following is NOT appropriate treatment for Echinococcus granulosus and
hydatid disease?
(a) Albendazole 400 mg twice in a day for 3 months
(b) Diethylcarbamazine 2 mg/kg thrice in a day for 12 days
(c) PAIR (percutaneous puncture, aspiration, injection of scolicide, reaspiration)
(d) Praziquantel (20 mg/kg twice daily for 14 days)
73.
Which one of the following infections is NOT amenable to post-exposure prophylaxis with
specific immunoglobulins?
(a) Hepatitis B
(b) HIV
(c) Tetanus
(d) Rabies
74.
Echinocandins are a class of:
(a) Antiviral agents
(b) Antiparasitic agents
(c) Antitubercular agents
(d) Antifungal agents
75.
Recommended first time drug for initial treatment of tuberculosis include all EXCEPT:
(a) Isoniazid
(b) Rifampin
(c) Ethambutol
(d) Ethionamide
76.
The most frequent adverse reaction of significance among people treated for drug-susceptible
tuberculosis is:
(a) Colitis
(b) Hepatitis
(c) Cystitis
(d) Enteritis
77.
All adult patients in whom anti-tubercular treatment is being started should undergo baseline
assessment of:
(a) Complete blood count
(b) Liver function test
(c) Lipid profile
(d) Pulmonary function test
78.
A 30 year old male presents with fever, headache, anorexia, nausea, vomiting and diarrhoea.
On examination tongue is coated with abdominal tenderness, soft splenomegaly, relative
bradycardia and rose spots rash. The most likely diagnosis is:

(a) Malaria fever
(b) Enteric fever
(c) Leptospirosis
(d) Dengue fever

79.
The extrapulmonary sites most commonly involved in tuberculosis are all EXCEPT:
(a) Lymph node TB
(b) Genitourinary TB
(c) Skeletal TB
(d) Skull TB
80.
Which of the following is NOT a clinical feature of Botulism?
(a) Impaired alertness
(b) Dysphagia
(c) Diplopia
(d) Ptosis
81.
Post exposure prophylaxis (PEP) for HIV should contain:
(a) 2 antiretroviral drugs administered for 4 weeks
(b) 2 antiretroviral drugs administered for 6 weeks
(c) 3 antiretroviral drugs administered for 4 weeks
(d) 3 antiretroviral drugs administered for 6 weeks
82.
Paralytic rabies is characterized by:
(a) Maculopapular rash
(b) Flaccid muscle weakness
(c) Spastic muscle weakness
(d) Hyperexcitability and facial weakness
83.
All of the following are seen in Marasmus EXCEPT:

(a) Reduced triceps skinfold
(b) Reduced mid arm circumference
(c) Protein wasting
(d) Decreased body mass index
84.
A 20 year young lady presents with high grade fever for five days and palpable purpura over
extremities. She is found to be confused, with presence of neck stiffness. Which of the
following is the most likely diagnosis?
(a) Disseminated intravascular coagulation
(b) Acute meningococcemia
(c) Antiphospholipid antibody syndrome
(d) Thrombotic thrombocytopenic purpura
85.
Fibroblast Growth Factor 23 (FGF 23) is increased in which one of the following conditions?
(a) Osteomalacia
(b) Osteoporosis
(c) Paget?s disease
(d) Renal Osteodystrophy





86.
Which of the following crystals are deposited in a case of pseudogout?
(a) Monosodium urate
(b) Calcium phosphate
(c) Cholestrol crystals
(d) Calcium pyrophosphate dehydrate (CPPD)
87.
Which of the following features are associated with poor prognosis in Diffuse Cutaneous
Systemic Sclerosis?

1. Older age
2. A high gas transfer factor for carbon monoxide (TLCO)
3. Proteinuria
4. Diffuse skin disease

Select the correct answer using the code given below:
(a) 1, 2 and 4
(b) 3 and 4 only
(c) 1, 3 and 4
(d) 1, 2 and 3

88.
Which of the following statements regarding pegloticase are correct?

1. It is enzyme conjugate
2. It is useful in chronic Gout management
3. The main adverse effects are infusion reactions and development of antibodies

Select the correct answer using the code given below:
(a) 1 and 2 only
(b) 2 and 3 only
(c) 1 and 3 only
(d) 1, 2 and 3
89.
The most common cause of sensorineural hearing loss in adults is:
(a) Otosclerosis
(b) Meniere?s disease
(c) Presbycusis
(d) Meningitis
90.
A 50-year old male presents with dizziness. Which one of the following clinical findings does
NOT favour a central cause?
(a) Poor saccades
(b) Gaze evoked nystagmus
(c) Inhibition by visual fixation
(d) Presence of diplopia, dysarthria




91.
A young primigravida with 11 weeks pregnancy presents with sudden onset pain and
swelling of left lower limb. Homan?s sign is positive. Duplex ultrasonogram shows non-
compressibility of veins in the affected region, with absent flow. The drug of choice for
treatment is:
(a) Enoxaparin
(b) Amoxycillin?Clavulanic acid
(c) Warfarin
(d) Limb elevation only
92.
All of the following are clinical features of hypovolaemic shock EXCEPT:
(a) Cold clammy skin
(b) Bradycardia
(c) Hypotension
(d) Oliguria
93.
Which one of the following is most appropriate to describe classic heat stroke?
(a) Patients core body temperature rises above 44?C with headache, nausea and loss of
consciousness
(b) Patients core body temperature rises above 40 ?C with muscle tremors, confusion,
loss of consciousness and loss of sweating
(c) Patients core body temperature rises above 38 ?C with increased sweating,
tachypnoea, tachycardia and loss of consciousness
(d) Patients core body temperature is above 40 ?C with disorientation, increased
sweating, peripheral vasodilation, tachycardia and loss of consciousness
94.
Which of the following is NOT a priority in the initial management of shock from venomous
snake?

(a) Aggressive volume resuscitation
(b) Vasopressor support
(c) Anti-venom administration
(d) All of these are undertaken simultaneously

95.
Which one of the following is NOT an appropriate criterion for diagnosis of Systemic
Inflammatory Response Syndrome (SIRS)?
(a) Respiratory rate > 20/mm
(b) Heart rate > 90/min
(c) Total leucocyte count > 16 ? 10
9
per Litre
(d) Temperature > 38
0
C








96.
All of the following predispose to Acute Respiratory Distress Syndrome EXCEPT:
(a) Acute Pancreatitis
(b) Toxic gas inhalation
(c) Aspiration of gastric contents
(d) Severe Bronchial asthma
97.
An 8 year old child has consumed a few tablets of a drug that were being taken by his mother
on a regular basis. Following this the child develops gastric necrosis, acidosis, shock and
hepatic necrosis. Which is the most likely drug?
(a) Multivitamins
(b) Calcium carbonate
(c) Folic acid
(d) Iron
98.
A newborn weighing 1.5 kg at birth presents at 6 hours of age with lethargy, weak and high
pitched cry and difficulty in feeding. What is the next step in management?

(a) Estimation of blood sugar
(b) Estimation of serum calcium
(c) Supplementary oxygen
(d) Bedside electrocardiogram

99.
In a child with acute organophosphorus poisoning, which one of the following drugs will act
as anti-sialogogue and peripheral parasympatholytic agent?

(a) Carbamates
(b) Atropine
(c) Benzodiazepine
(d) Pralidoxime
100.
A 6-year old child needs to undergo a central Catheter placement for total parental nutrition.
The procedure is likely to evoke pain and anxiety. Which of the following is the most
appropriate drug for sedation and analgesia in this child?

(a) Chloral hydrate
(b) Midozolam
(c) Propofol
(d) Ketamine
101.
Which one of the following is the drug of choice in a child presenting with supraventricular
tachycardia?

(a) Atropine
(b) Lidocaine
(c) Adenosine
(d) Nalaxone


102.
Which one of the following drugs is used in cardiopulmonary resuscitation for pulseless
ventricular fibrillation?

(a) Atropine
(b) Adenosine
(c) Lidocaine
(d) Amiodarone
103.
Which one of the following is a sign of good attachment during breastfeeding?
(a) The baby?s nose is at the level of the nipple
(b) The baby?s chin touches the breast
(c) Most of the nipple is in the mouth and the lower areola is visible
(d) The baby?s lower lip is inverted
104.
Which one of the following signs is NOT included as an indicator of a vigorous baby
immediately after delivery of an infant born through meconium strained liquor?
(a) Pink colour
(b) Strong respiratory efforts
(c) Good muscle tone
(d) Active movements of the baby
105.
All are true for a Cephalohematoma in a newborn EXCEPT:
(a) It is located over the parietal bones in the subcutaneous plane
(b) It may be associated with hyperbilirubinemia
(c) It increases in size in the first 12?24 hours
(d) It can take 3?6 weeks to resolve
106.
Which of the following sick neonates can be started on ?Minimal Enteral Feeding?, if
hemodynamically stable?
(a) Septic neonate with sclerema
(b) Neonate with necrotizing enterocolitis (NEC)
(c) Neonate on mechanical ventilation
(d) Symptomatic hypoglycaemia
107.
Which one of the following drugs is contraindicated in a mother who is breastfeeding her
infant?
(a) Bromocriptine
(b) Chlorpromazine
(c) Acyclovir
(d) Morphine
108.
A term neonate presents with tachypnea on day 1 after birth. The chest X-ray reveals
hyperexpanded lung fields, prominent vascular markings, and prominent interlobar fissure.
Which of the following is the most probable diagnosis?
(a) Congenital lobar emphysema
(b) Diaphragmatic hernia
(c) Transient tachypnea of the newborn
(d) Tracheoesophageal fistula

FirstRanker.com - FirstRanker's Choice
Combined Medical Services Examination-2020
Paper-I
1.
A chronic alcoholic develops a paroxysm of palpitations after alcohol binge. Which of the
following Arrhythmia is most likely?
(a) Ventricular fibrillation
(b) Ventricular premature complex
(c) Atrial flutter
(d) Atrial fibrillation
2.
An elderly-man with history of Diabetes mellitus and Coronary Artery Disease comes for
follow-up, with complaints of muscle pains. Which one of the following drugs could be the
most likely cause?
(a) Aspirin
(b) Glimepiride
(c) Enalapril
(d) Atorvastatin
3.
A 40-year old lady comes to outdoor clinic with complaints of sudden onset chest pain. The
chest X-ray shows bilateral Pneumothorax. Examination reveals abnormalities of body
habitus-including long arms, legs and finger (arachnodactyly), scoliosis, high arched palate,
joint hypermobility, and a pansystolic murmur at cardiac apex. Which one of the following
diseases is most likely?
(a) Takayasu?s arteritis
(b) Raynaud?s syndrome
(c) Marfan?s syndrome
(d) Rheumatic heart disease-Mitral regurgitation
4.
All of the following are indications for treadmill testing/exercise-testing EXCEPT:
(a) To confirm the diagnosis of angina
(b) To evaluate stable angina
(c) To assess outcome after coronary revascularization
(d) To evaluate the treatment efficacy of antianginal drugs
5.
Kussmaul?s sign is present in all of the following conditions, EXCEPT:

(a) Massive Pulmonary Embolism
(b) Restrictive Cardiomyopathy
(c) Hypertrophic Cardiomyopathy
(d) Right Ventricular Infarction

6.
Which one of the following is NOT a common cause of atrial fibrillation?

(a) Mitral regurgitation
(b) Hypothyroidism
(c) Hypertension
(d) Acute myocardial infarction

7.
In an ECG recording, P wave is produced by:

(a) Atrial depolarisation
(b) Atrial repolarisation
(c) Ventricular depolarisation
(d) Ventricular repolarisation
8.
Which one of the following modalities is NOT used in the treatment of ventricular
tachycardia?

(a) DC Cardioversion
(b) Injection Lignocaine
(c) Injection Amiodarone
(d) Injection Adenosine
9.
Which one of the following drugs is a Direct Renin Inhibitor?

(a) Benedipine
(b) Azilsartan
(c) Aliskiren
(d) Lisinopril
10.
All of the following are causes of systolic hypertension with wide pulse pressure EXCEPT:

(a) Aortic regurgitation
(b) Thyrotoxicosis
(c) Patent ductus arteriosus
(d) Aortic stenosis
11.
Signet-ring sign on CT-chest is suggestive of:
(a) Bronchiectasis
(b) Active alveolitis
(c) Aspergilloma
(d) Sarcoidosis
12.
Oral corticosteroids are best introduced in the treatment of chronic bronchial asthma when it
is:
(a) Mild persistent
(b) Moderate persistent
(c) Severe persistent
(d) Very severe persistent
13.
An 18-year old male has insulin dependent diabetes, with malabsorption syndrome and
bilateral upper lobe bronchiectasis. A diagnostic work-up will include all EXCEPT:
(a) 24 hour fecal fat-estimation
(b) CT-chest
(c) Lung biopsy
(d) DNA sequencing study
14.
Which one of the following antigens is commonly associated with causation of Farmer?s
lung?
(a) Penicillium
(b) Actinomycetes
(c) Aspergillus
(d) Candida
15.
The Gene X-pert test used for MTB detection has the additional advantage of detection of
which of the following?

(a) INH resistance
(b) Rifampicin resistance
(c) Multi drug resistance
(d) Ethambutol resistance
16.
The most common cause of chronic type II respiratory failure is:
(a) Severe pneumonia
(b) Severe COPD
(c) Severe bronchial asthma
(d) Severe pulmonary thromboembolism
17.
The most frequent symptom in respiratory diseases is:
(a) Breathlessness
(b) Chest pain
(c) Hemoptysis
(d) Cough
18.
A 26-year old young lady attends medical emergency and is labelled as acute severe asthma
by the emergency physician. Which one of the following is unlikely to be a part of the
prescription?

(a) Albuterol
(b) Anti-leukotrienes
(c) Aminophylline
(d) Magnesium sulfate
19.
Consider the following statements with regard to respiratory examination:

1. Change in note, when patient phonates ?EEE? (Egophony) is characteristic of
interstitial fibrosis
2. Whispered pectoriloquy is characteristic of lung consolidation
3. Monophonic wheeze is characteristic of asthma
4. Hyper-resonant note on percussion is characteristic of pnemothorax

Which of the above statements are correct?
(a) 1 and 3
(b) 2 and 4 only
(c) 1, 2 and 4
(d) 2, 3 and 4
20.
All of the following are the causes of exudative Pleural Effusion, EXCEPT:
(a) SVC obstruction
(b) Fungal infection
(c) SLE
(d) Meig syndrome
21.
Which one of the following is NOT true about Ghrelin?
(a) It stimulates appetite
(b) It decreases gastric emptying
(c) It increases acid secretion
(d) Fasting increases its secretion
22.
Fecal elastase test is used for diagnosing which one of the following conditions?
(a) Lactose intolerance
(b) Mucosal inflammation
(c) Bile acid secretory defects
(d) Pancreatic dysfunction
23.
The recent classification system for listing a patient as a candidate for liver transplantation is:
(a) Child Pugh score
(b) APACHE score
(c) MELD score
(d) Metavir score
24.
The blood supply of liver consists of:
(a) 50 % hepatic artery and 50 % portal vein
(b) 80 % portal vein and 20 % hepatic artery
(c) 80 % hepatic artery and 20 % portal vein
(d) 70 % hepatic artery and 30 % portal vein
25.
All of the following conditions are associated with Glomerulonephritis with low complement
level, EXCEPT:
(a) Subacute bacterial endocarditis
(b) IgA nephropathy
(c) Systemic lupus erythematosus
(d) Cryoglobulinaemia













26.
Consider the following statements with regard to oral aphthous ulcers:

1. They are superficial and painful
2. They may occur in women just prior to menstruation
3. Oral glucocorticoids may be needed for the treatment of severe, recurrent cases
4. They are pre-malignant and progress over few years to squamous cell carcinoma of
the oral cavity

Which of the above statements are correct?

(a) 1 and 4 only
(b) 2 and 3 only
(c) 1, 2 and 3 only
(d) 1, 2, 3 and 4
27.
Which one of the following statements about Barrett?s Oesophagus is NOT correct?

(a) It is a pre-malignant condition
(b) Normal squamous cells lining the lower oesophagus are replaced by columnar
cells
(c) Normal columnar cells lining the lower oesophagus are replaced by squamous
cells
(d) It is an adaptive response to chronic gastro-oesophageal reflux

28.
The most common cause of acute hepatitis outbreaks in India is:

(a) Hepatitis E
(b) Hepatitis C
(c) Hepatitis B
(d) Hepatitis A

29.
Spontaneous oesophageal perforation after a bout of forceful vomiting or retching is
characteristically seen in which one of the following conditions?

(a) Menetrier?s disease
(b) Boerhaave?s syndrome
(c) Achalasia of the oesophagus
(d) Barrett?s oesophagus
30.
Eradication of helicobacter pylori infection may prove beneficial in the following extra-
gastric disorders EXCEPT:

(a) Unexplained vitamin B12 deficiency
(b) Idiopathic thrombocytopenic purpura
(c) Iron deficiency anaemia without gastro-intestinal bleeding
(d) Acute glomerulonephritis

31.
What is the target blood Hemoglobin level when treating anaemia in Stage-4 chronic kidney
disease patient?
(a) 8 to 10 gm/dL
(b) 10 to 12 gm/dL
(c) 12 to 13 gm/dL
(d) 13 to 14 gm/dL
32.
Which one of the following complications of chronic kidney disease is observed in patients
with low parathyroid hormone levels?
(a) Adynamic bone disease
(b) Tumoral calcinosis
(c) Osteitis fibrosa cystic
(d) Calciphylaxis
33.
All of the following statements about adult polycystic kidney disease are true, EXCEPT:
(a) It is inherited as autosomal dominant trait
(b) It is usually associated with marked proteinuria
(c) It is usually associated with systematic hypertension, from young age
(d) It is associated with Berry aneurysm
34.
Consider the following statements in relation to an adult patient:

1. Oliguria is defined as passage of urine less than 300 ml per day
2. Anuria is said to exist when less than 50 ml urine is passed per day
3. Polyuria is defined as urine volume in excess of 3 litre per day

Which of the statements given above are correct?
(a) 1 and 3 only
(b) 2 and 3 only
(c) 1 and 2 only
(d) 1, 2 and 3

35.
White cell casts in urine examination are strongly suggestive of:
(a) Nephritis
(b) Pyelonephritis
(c) Renal stone disease
(d) Papillitis
36.
A 13-year old boy presents with hematuria, oliguria, edema and hypertension. He has history
of sore throat two weeks prior to presentation. Laboratory investigations are remarkable for
low C
3
and increased titres of ASO and antiDNase. Which one of the following statements is
NOT correct about management for this condition?

(a) Renal biopsy is rarely required for making diagnosis
(b) Antibiotic treatment is given for streptococcal infection
(c) Treatment is largely supportive
(d) Immunosuppressants are to be used for crescentic glomerulonephritis
37.
Which one of the following statements is NOT correct regarding the risk factors for
nephrolithiasis?

(a) Higher dietary calcium increases risk
(b) Higher animal protein intake increases risk
(c) Higher dietary potassium intake lowers risk
(d) Lower urine citrate increases the risk
38.
A 60-year old patient is admitted in emergency with seizure, aphasia and altered sensorium.
He has history of fever and headache for the preceding 3 days. The CSF examination is
unremarkable. What would be the probable diagnosis?
(a) Multiple sclerosis
(b) Tubercular meningitis
(c) Pyogenic (bacterial) meningitis
(d) Viral encephalitis
39.
Drug of choice for the treatment of Trigeminal Neuralgia is:
(a) Carbamazepine
(b) Aceclofenac
(c) Thiamine
(d) Prednisdone
40.
Constructional skills is a function of which lobe?
(a) Dominant temporal lobe
(b) Non dominant parietal lobe
(c) Frontal lobe
(d) Non dominant temporal lobe

41.
Vertical gaze palsy with convergence retraction nystagmus is seen in:
(a) Weber syndrome
(b) Millard Gubler syndrome
(c) Claude syndrome
(d) Parinaud syndrome
42.
A patient on looking forward was found to have his right eye deviated downwards and
outwards with pupil dilated. He is suffering from:
(a) Left 3
rd
nerve palsy
(b) Left 6
th
nerve palsy
(c) Right 4
th
nerve palsy
(d) Right 3
rd
nerve palsy
43.
All of the following are features of Cauda Equina syndrome EXCEPT:

(a) Low back pain
(b) Lower limb areflexia
(c) Loss of bladder function
(d) Extensor plantar response

44.
A 68-year old male presented in OPD with complaints of progressive small handwriting. On
examination, he had resting tremor, bradykinesia, rigidity and postural instability. This
patient is suffering most likely from:

(a) Parkinson?s disease
(b) Vascular dementia
(c) Alzheimer?s disease
(d) Frunto temporal dementia
45.
An elderly patient presents with abnormal gait. He has a wide base freezing gait with
imbalance, comprising short strides, shuffling along the floor and difficulty with starts and
turns. Heal-Shin test is normal. Which one of the following is most likely disorder?
(a) Cerebellar ataxia
(b) Sensory ataxia
(c) Frontal gait disorder
(d) Parkinson?s disease
46.
Cushing reflex, seen in conditions of raised intra cranial pressure includes all of the following
EXCEPT:
(a) Bradycardia
(b) Hypertension
(c) Irregular respiration
(d) Hypothermia

47.
A 35 year old man presents with history of low grade fever and headache for last five weeks.
Clinical examination is remarkable for signs of meningismus. He undergoes lumbar puncture
(LP) and a day later reports worsening of headache. Which one of the following features is
NOT consistent with diagnosis of ?Post-LP? headache?

(a) Post-LP headache usually begins within 48 hours
(b) Post-LP headache worsens in sitting position
(c) Post-LP headache is most severe upon waking up
(d) Post-LP headache may improve with caffeine intake
48.
Which one of the following investigations is NOT appropriate in a case of recurrent
thrombosis?
(a) Antiphospholipid antibodies
(b) Bcr-Abl assay
(c) Protein C and S assays
(d) Antithrombin level
49.
All of the following are causes of reactive thrombocytosis, EXCEPT:
(a) Hemolytic anemia
(b) Megaloblastic anemia
(c) Post splenectomy
(d) Chronic inflammatory disorders


50.
Cryoprecipitate cannot be used for treatment of which one of the following conditions?
(a) Von Willebrand disease
(b) Hypofibrinogenemia
(c) Hemophilia?B
(d) Hemphilia?A
51.
Pyruvate kinase deficiency results in deficiency of ATP production and a chronic haemolytic
anaemia. The disorder is inherited as an:

(a) Autosomal recessive trait
(b) X-linked recessive trait
(c) Autosomal dominant trait
(d) X-linked dominant trait
52.
Sickle cell disease may be associated with any of the following EXCEPT:

(a) Acute chest syndrome
(b) Aplastic crisis
(c) Splenic sequestration crisis
(d) Conn?s syndrome

53.
Fever with splenomegaly and lymphadenopathy can be seen in
(a) Infectious mononucleosis
(b) Chronic leukaemia
(c) Both infectious mononucleosis and chronic leukaemia
(d) Neither infectious mononucleosis nor chronic leukaemia
54.
Every individual carries four alpha gene alleles. Deletion of three alleles leads to
development of:
(a) Haemoglobin X disease
(b) Haemoglobin H disease
(c) Haemoglobin F disease
(d) Hydrops fetalis
55.
Which one of the following is NOT true regarding Chronic Myeloid Leukaemia (CML)?
(a) It is a clonal malignancy of haematopoetic stem
(b) Risk of developing CML is increased in monozygotic turns
(c) CML is defined by presence of BCR-ABL 1 fusion gene
(d) 10 year survival with TKI therapy is 85%









56.
Diagnostic criteria of multiple myeloma includes which of the following?

1. Increased malignant plasma cells in the bone marrow
2. Serum and/or urinary M protein
3. Skeletal lytic lesions
4. Skeletal blastic lesions

Select the correct answer using the code given below:
(a) 1, 2 and 3
(b) 1, 3 and 4
(c) 1, 2 and 4
(d) 2, 3 and 4
57.
The most predominant type of serum immunoglobulin involved in patients with plasma cell
disorder is:
(a) Ig G
(b) Ig A
(c) Ig D
(d) Light chains
58.
Which one of the following statements is NOT true regarding the thyroid function/disorder in
pregnancy?
(a) There is an increase in the metabolism of thyroxine by placenta
(b) Hyperemesis gravidarum may be associated with thyrotoxicosis in pregnancy
(c) Levothyroxine replacement therapy dose should be decreased by 30?50% early in
pregnancy
(d) Impaired cognitive impairment may be seen in the offspring of mother with
subclinical hypothyroidism

59.
Which one of the following is NOT the criterion for the diagnosis of Diabetes mellitus?
(a) Symptoms of diabetes and random plasma sugar concentration ? 200 mg/dL
(b) HbA1c ? 6.5 %
(c) 2-hours plasma glucose ? 200 mg/dL during an oral glucose tolerance test
(d) Fasting plasma glucose ? 110 mg/dL
60.
A young lady presents in outdoor clinic with complaints of menstrual irregularity, weight
gain, hair loss, tiredness and weakness. What will be the investigation of choice?
(a) FSH & LH level
(b) Estrogen level
(c) Free T
3
and free T
4
level
(d) Free T
4
and TSH level







61.
Consider the following statements about erectile dysfunction in diabetic males:

1. It affects 60 % of males
2. Its common cause is an underlying neuro-vascular pathology
3. It may be aggravated by beta-adrenergic agonist drugs
4. Endocrine disorders like hyperprolactinemia may cause it

Which of the statements given above are true?
(a) 1 and 2
(b) 2 and 3
(c) 2 and 4
(d) 3 and 4
62.
All of the following can cause Hyperprolactinemia, EXCEPT:
(a) Craniopharyngioma
(b) Hyperthyroidism
(c) Stress
(d) Chronic renal failure
63.
All of the following can be seen in a case of thyrotoxicosis, EXCEPT:
(a) Atrial fibrillation
(b) Periodic paralysis
(c) Deafness
(d) Osteoporosis
64.
Which one of the following hormones acts via the receptor tyrosine kinase?
(a) FSH
(b) TSH
(c) IGF-1
(d) PTH
65.
Consider the following disorders as constituent of Multiple Endocrine Neoplasia Type 2a
(MEN 2a):

1. Primary Hyperparathyroidism
2. Pituitary tumours
3. Medullary carcinoma of thyroid
4. Pheochromocytoma

Which of the above are correct?:

(a) 1, 2 and 4
(b) 1, 2 and 3
(c) 1, 3 and 4
(d) 2, 3 and 4




66.
Consider the following statements with regard to Graves? ophthalmopathy :

1. Proptosis is often asymmetric and can even appear to be unilateral
2. It is a clinical diagnosis
3. Worsening of symptoms upon glucocorticoid withdrawal is common
4. Radiation therapy is very effective in treatment

Which of the above statements are correct?

(a) 1 and 2 only
(b) 1, 2 and 3
(c) 2, 3 and 4
(d) 1 and 4 only
67.
All of the following are common aetiologies of delirium EXCEPT:
(a) Hyperthyroidism
(b) Hypothyroidism
(c) Hyperparathyroidism
(d) Hypoparathyroidism
68.
Key anthropometric measurements important for evaluating the degree of obesity are:

(a) Weight, height and waist circumference
(b) Weight, height and hip circumference
(c) Weight, height and mid-thigh circumference
(d) Weight, height and chest circumference
69.
What is the minimum BMI (Body Mass Index) recommended for considering adjuvant
pharmacologic treatment in obesity with comorbidity?
(a) 23 kg/m
2

(b) 25 kg/m
2

(c) 27 kg/m
2

(d) 30 kg/m
2

70.
A young medical student while working in HIV ward got pricked accidently on the finger by
a syringe needle which was used to draw blood from a patient. Patient?s infection status is not
known. Which one of the following therapeutic interventions will be the best interest of this
student?
(a) To start Dolultegravir immediately
(b) To wash wound with soap and water and start Emtricitabine
(c) To give Dolutegravir + Tenofovir + Emtricitabine
(d) To give Dolutegravir + Tenofovir + Emtricitabine + HBV Immunoglobulin
71.
Cryptococcal infection is acquired through:
(a) Inhalation route
(b) Bite of Culex mosquito
(c) Direct skin contact
(d) Ingestion of spores

72.
Which one of the following is NOT appropriate treatment for Echinococcus granulosus and
hydatid disease?
(a) Albendazole 400 mg twice in a day for 3 months
(b) Diethylcarbamazine 2 mg/kg thrice in a day for 12 days
(c) PAIR (percutaneous puncture, aspiration, injection of scolicide, reaspiration)
(d) Praziquantel (20 mg/kg twice daily for 14 days)
73.
Which one of the following infections is NOT amenable to post-exposure prophylaxis with
specific immunoglobulins?
(a) Hepatitis B
(b) HIV
(c) Tetanus
(d) Rabies
74.
Echinocandins are a class of:
(a) Antiviral agents
(b) Antiparasitic agents
(c) Antitubercular agents
(d) Antifungal agents
75.
Recommended first time drug for initial treatment of tuberculosis include all EXCEPT:
(a) Isoniazid
(b) Rifampin
(c) Ethambutol
(d) Ethionamide
76.
The most frequent adverse reaction of significance among people treated for drug-susceptible
tuberculosis is:
(a) Colitis
(b) Hepatitis
(c) Cystitis
(d) Enteritis
77.
All adult patients in whom anti-tubercular treatment is being started should undergo baseline
assessment of:
(a) Complete blood count
(b) Liver function test
(c) Lipid profile
(d) Pulmonary function test
78.
A 30 year old male presents with fever, headache, anorexia, nausea, vomiting and diarrhoea.
On examination tongue is coated with abdominal tenderness, soft splenomegaly, relative
bradycardia and rose spots rash. The most likely diagnosis is:

(a) Malaria fever
(b) Enteric fever
(c) Leptospirosis
(d) Dengue fever

79.
The extrapulmonary sites most commonly involved in tuberculosis are all EXCEPT:
(a) Lymph node TB
(b) Genitourinary TB
(c) Skeletal TB
(d) Skull TB
80.
Which of the following is NOT a clinical feature of Botulism?
(a) Impaired alertness
(b) Dysphagia
(c) Diplopia
(d) Ptosis
81.
Post exposure prophylaxis (PEP) for HIV should contain:
(a) 2 antiretroviral drugs administered for 4 weeks
(b) 2 antiretroviral drugs administered for 6 weeks
(c) 3 antiretroviral drugs administered for 4 weeks
(d) 3 antiretroviral drugs administered for 6 weeks
82.
Paralytic rabies is characterized by:
(a) Maculopapular rash
(b) Flaccid muscle weakness
(c) Spastic muscle weakness
(d) Hyperexcitability and facial weakness
83.
All of the following are seen in Marasmus EXCEPT:

(a) Reduced triceps skinfold
(b) Reduced mid arm circumference
(c) Protein wasting
(d) Decreased body mass index
84.
A 20 year young lady presents with high grade fever for five days and palpable purpura over
extremities. She is found to be confused, with presence of neck stiffness. Which of the
following is the most likely diagnosis?
(a) Disseminated intravascular coagulation
(b) Acute meningococcemia
(c) Antiphospholipid antibody syndrome
(d) Thrombotic thrombocytopenic purpura
85.
Fibroblast Growth Factor 23 (FGF 23) is increased in which one of the following conditions?
(a) Osteomalacia
(b) Osteoporosis
(c) Paget?s disease
(d) Renal Osteodystrophy





86.
Which of the following crystals are deposited in a case of pseudogout?
(a) Monosodium urate
(b) Calcium phosphate
(c) Cholestrol crystals
(d) Calcium pyrophosphate dehydrate (CPPD)
87.
Which of the following features are associated with poor prognosis in Diffuse Cutaneous
Systemic Sclerosis?

1. Older age
2. A high gas transfer factor for carbon monoxide (TLCO)
3. Proteinuria
4. Diffuse skin disease

Select the correct answer using the code given below:
(a) 1, 2 and 4
(b) 3 and 4 only
(c) 1, 3 and 4
(d) 1, 2 and 3

88.
Which of the following statements regarding pegloticase are correct?

1. It is enzyme conjugate
2. It is useful in chronic Gout management
3. The main adverse effects are infusion reactions and development of antibodies

Select the correct answer using the code given below:
(a) 1 and 2 only
(b) 2 and 3 only
(c) 1 and 3 only
(d) 1, 2 and 3
89.
The most common cause of sensorineural hearing loss in adults is:
(a) Otosclerosis
(b) Meniere?s disease
(c) Presbycusis
(d) Meningitis
90.
A 50-year old male presents with dizziness. Which one of the following clinical findings does
NOT favour a central cause?
(a) Poor saccades
(b) Gaze evoked nystagmus
(c) Inhibition by visual fixation
(d) Presence of diplopia, dysarthria




91.
A young primigravida with 11 weeks pregnancy presents with sudden onset pain and
swelling of left lower limb. Homan?s sign is positive. Duplex ultrasonogram shows non-
compressibility of veins in the affected region, with absent flow. The drug of choice for
treatment is:
(a) Enoxaparin
(b) Amoxycillin?Clavulanic acid
(c) Warfarin
(d) Limb elevation only
92.
All of the following are clinical features of hypovolaemic shock EXCEPT:
(a) Cold clammy skin
(b) Bradycardia
(c) Hypotension
(d) Oliguria
93.
Which one of the following is most appropriate to describe classic heat stroke?
(a) Patients core body temperature rises above 44?C with headache, nausea and loss of
consciousness
(b) Patients core body temperature rises above 40 ?C with muscle tremors, confusion,
loss of consciousness and loss of sweating
(c) Patients core body temperature rises above 38 ?C with increased sweating,
tachypnoea, tachycardia and loss of consciousness
(d) Patients core body temperature is above 40 ?C with disorientation, increased
sweating, peripheral vasodilation, tachycardia and loss of consciousness
94.
Which of the following is NOT a priority in the initial management of shock from venomous
snake?

(a) Aggressive volume resuscitation
(b) Vasopressor support
(c) Anti-venom administration
(d) All of these are undertaken simultaneously

95.
Which one of the following is NOT an appropriate criterion for diagnosis of Systemic
Inflammatory Response Syndrome (SIRS)?
(a) Respiratory rate > 20/mm
(b) Heart rate > 90/min
(c) Total leucocyte count > 16 ? 10
9
per Litre
(d) Temperature > 38
0
C








96.
All of the following predispose to Acute Respiratory Distress Syndrome EXCEPT:
(a) Acute Pancreatitis
(b) Toxic gas inhalation
(c) Aspiration of gastric contents
(d) Severe Bronchial asthma
97.
An 8 year old child has consumed a few tablets of a drug that were being taken by his mother
on a regular basis. Following this the child develops gastric necrosis, acidosis, shock and
hepatic necrosis. Which is the most likely drug?
(a) Multivitamins
(b) Calcium carbonate
(c) Folic acid
(d) Iron
98.
A newborn weighing 1.5 kg at birth presents at 6 hours of age with lethargy, weak and high
pitched cry and difficulty in feeding. What is the next step in management?

(a) Estimation of blood sugar
(b) Estimation of serum calcium
(c) Supplementary oxygen
(d) Bedside electrocardiogram

99.
In a child with acute organophosphorus poisoning, which one of the following drugs will act
as anti-sialogogue and peripheral parasympatholytic agent?

(a) Carbamates
(b) Atropine
(c) Benzodiazepine
(d) Pralidoxime
100.
A 6-year old child needs to undergo a central Catheter placement for total parental nutrition.
The procedure is likely to evoke pain and anxiety. Which of the following is the most
appropriate drug for sedation and analgesia in this child?

(a) Chloral hydrate
(b) Midozolam
(c) Propofol
(d) Ketamine
101.
Which one of the following is the drug of choice in a child presenting with supraventricular
tachycardia?

(a) Atropine
(b) Lidocaine
(c) Adenosine
(d) Nalaxone


102.
Which one of the following drugs is used in cardiopulmonary resuscitation for pulseless
ventricular fibrillation?

(a) Atropine
(b) Adenosine
(c) Lidocaine
(d) Amiodarone
103.
Which one of the following is a sign of good attachment during breastfeeding?
(a) The baby?s nose is at the level of the nipple
(b) The baby?s chin touches the breast
(c) Most of the nipple is in the mouth and the lower areola is visible
(d) The baby?s lower lip is inverted
104.
Which one of the following signs is NOT included as an indicator of a vigorous baby
immediately after delivery of an infant born through meconium strained liquor?
(a) Pink colour
(b) Strong respiratory efforts
(c) Good muscle tone
(d) Active movements of the baby
105.
All are true for a Cephalohematoma in a newborn EXCEPT:
(a) It is located over the parietal bones in the subcutaneous plane
(b) It may be associated with hyperbilirubinemia
(c) It increases in size in the first 12?24 hours
(d) It can take 3?6 weeks to resolve
106.
Which of the following sick neonates can be started on ?Minimal Enteral Feeding?, if
hemodynamically stable?
(a) Septic neonate with sclerema
(b) Neonate with necrotizing enterocolitis (NEC)
(c) Neonate on mechanical ventilation
(d) Symptomatic hypoglycaemia
107.
Which one of the following drugs is contraindicated in a mother who is breastfeeding her
infant?
(a) Bromocriptine
(b) Chlorpromazine
(c) Acyclovir
(d) Morphine
108.
A term neonate presents with tachypnea on day 1 after birth. The chest X-ray reveals
hyperexpanded lung fields, prominent vascular markings, and prominent interlobar fissure.
Which of the following is the most probable diagnosis?
(a) Congenital lobar emphysema
(b) Diaphragmatic hernia
(c) Transient tachypnea of the newborn
(d) Tracheoesophageal fistula

109.
A pre-school boy can go upstairs on alternate feet and has also started copying a circle. How
many blocks should he be able to build a tower with, without toppling over?

(a) 5
(b) 7
(c) 9
(d) More than 10

110.
Which one of the following drugs is the antidote for benzodiazepine poisoning?

(a) Flumazenil
(b) Atropine
(c) Naloxone
(d) Pralidoxime aldoxime methiodide

111.
A 3-year old child develops severe respiratory distress after inhalation of fumes arising from
burning plastic. The child is started on supportive ventilation. Which one of the following
antidotes needs to be given?
(a) Hydroxycobalamin
(b) Amylnitrate
(c) Sodium nitrite
(d) N-acetyl cysteine
112.
A child has an incurving of the little finger. What is this condition known as?
(a) Camptodactyly
(b) Brachydactyly
(c) Partial cutaneous syndactyly
(d) Clinodactyly
113.
A 4-week old infant presents with repeated episodes of non-bilious vomiting since 7 days.
The infant seems active and hungry after the vomiting. Examination reveals an olive-shaped
mass in the epigartrium. Which metabolic abnormality is expected in this infant?
(a) Hyperchloremia
(b) Metabolic alkalosis
(c) Hyperkalemia
(d) Hyponatramia
114.
Which one of the following is the correct dose and route of administration of hepatitis B
vaccine in children?

(a) 0.5 mL dose with 10 microgram of antigen subcutaneously
(b) 0.5 mL dose with 10 microgram of antigen intramuscularly
(c) 1 mL dose with 5 microgram of antigen subcutaneously
(d) 1 mL dose with 5 microgram of antigen intramuscularly


FirstRanker.com - FirstRanker's Choice
Combined Medical Services Examination-2020
Paper-I
1.
A chronic alcoholic develops a paroxysm of palpitations after alcohol binge. Which of the
following Arrhythmia is most likely?
(a) Ventricular fibrillation
(b) Ventricular premature complex
(c) Atrial flutter
(d) Atrial fibrillation
2.
An elderly-man with history of Diabetes mellitus and Coronary Artery Disease comes for
follow-up, with complaints of muscle pains. Which one of the following drugs could be the
most likely cause?
(a) Aspirin
(b) Glimepiride
(c) Enalapril
(d) Atorvastatin
3.
A 40-year old lady comes to outdoor clinic with complaints of sudden onset chest pain. The
chest X-ray shows bilateral Pneumothorax. Examination reveals abnormalities of body
habitus-including long arms, legs and finger (arachnodactyly), scoliosis, high arched palate,
joint hypermobility, and a pansystolic murmur at cardiac apex. Which one of the following
diseases is most likely?
(a) Takayasu?s arteritis
(b) Raynaud?s syndrome
(c) Marfan?s syndrome
(d) Rheumatic heart disease-Mitral regurgitation
4.
All of the following are indications for treadmill testing/exercise-testing EXCEPT:
(a) To confirm the diagnosis of angina
(b) To evaluate stable angina
(c) To assess outcome after coronary revascularization
(d) To evaluate the treatment efficacy of antianginal drugs
5.
Kussmaul?s sign is present in all of the following conditions, EXCEPT:

(a) Massive Pulmonary Embolism
(b) Restrictive Cardiomyopathy
(c) Hypertrophic Cardiomyopathy
(d) Right Ventricular Infarction

6.
Which one of the following is NOT a common cause of atrial fibrillation?

(a) Mitral regurgitation
(b) Hypothyroidism
(c) Hypertension
(d) Acute myocardial infarction

7.
In an ECG recording, P wave is produced by:

(a) Atrial depolarisation
(b) Atrial repolarisation
(c) Ventricular depolarisation
(d) Ventricular repolarisation
8.
Which one of the following modalities is NOT used in the treatment of ventricular
tachycardia?

(a) DC Cardioversion
(b) Injection Lignocaine
(c) Injection Amiodarone
(d) Injection Adenosine
9.
Which one of the following drugs is a Direct Renin Inhibitor?

(a) Benedipine
(b) Azilsartan
(c) Aliskiren
(d) Lisinopril
10.
All of the following are causes of systolic hypertension with wide pulse pressure EXCEPT:

(a) Aortic regurgitation
(b) Thyrotoxicosis
(c) Patent ductus arteriosus
(d) Aortic stenosis
11.
Signet-ring sign on CT-chest is suggestive of:
(a) Bronchiectasis
(b) Active alveolitis
(c) Aspergilloma
(d) Sarcoidosis
12.
Oral corticosteroids are best introduced in the treatment of chronic bronchial asthma when it
is:
(a) Mild persistent
(b) Moderate persistent
(c) Severe persistent
(d) Very severe persistent
13.
An 18-year old male has insulin dependent diabetes, with malabsorption syndrome and
bilateral upper lobe bronchiectasis. A diagnostic work-up will include all EXCEPT:
(a) 24 hour fecal fat-estimation
(b) CT-chest
(c) Lung biopsy
(d) DNA sequencing study
14.
Which one of the following antigens is commonly associated with causation of Farmer?s
lung?
(a) Penicillium
(b) Actinomycetes
(c) Aspergillus
(d) Candida
15.
The Gene X-pert test used for MTB detection has the additional advantage of detection of
which of the following?

(a) INH resistance
(b) Rifampicin resistance
(c) Multi drug resistance
(d) Ethambutol resistance
16.
The most common cause of chronic type II respiratory failure is:
(a) Severe pneumonia
(b) Severe COPD
(c) Severe bronchial asthma
(d) Severe pulmonary thromboembolism
17.
The most frequent symptom in respiratory diseases is:
(a) Breathlessness
(b) Chest pain
(c) Hemoptysis
(d) Cough
18.
A 26-year old young lady attends medical emergency and is labelled as acute severe asthma
by the emergency physician. Which one of the following is unlikely to be a part of the
prescription?

(a) Albuterol
(b) Anti-leukotrienes
(c) Aminophylline
(d) Magnesium sulfate
19.
Consider the following statements with regard to respiratory examination:

1. Change in note, when patient phonates ?EEE? (Egophony) is characteristic of
interstitial fibrosis
2. Whispered pectoriloquy is characteristic of lung consolidation
3. Monophonic wheeze is characteristic of asthma
4. Hyper-resonant note on percussion is characteristic of pnemothorax

Which of the above statements are correct?
(a) 1 and 3
(b) 2 and 4 only
(c) 1, 2 and 4
(d) 2, 3 and 4
20.
All of the following are the causes of exudative Pleural Effusion, EXCEPT:
(a) SVC obstruction
(b) Fungal infection
(c) SLE
(d) Meig syndrome
21.
Which one of the following is NOT true about Ghrelin?
(a) It stimulates appetite
(b) It decreases gastric emptying
(c) It increases acid secretion
(d) Fasting increases its secretion
22.
Fecal elastase test is used for diagnosing which one of the following conditions?
(a) Lactose intolerance
(b) Mucosal inflammation
(c) Bile acid secretory defects
(d) Pancreatic dysfunction
23.
The recent classification system for listing a patient as a candidate for liver transplantation is:
(a) Child Pugh score
(b) APACHE score
(c) MELD score
(d) Metavir score
24.
The blood supply of liver consists of:
(a) 50 % hepatic artery and 50 % portal vein
(b) 80 % portal vein and 20 % hepatic artery
(c) 80 % hepatic artery and 20 % portal vein
(d) 70 % hepatic artery and 30 % portal vein
25.
All of the following conditions are associated with Glomerulonephritis with low complement
level, EXCEPT:
(a) Subacute bacterial endocarditis
(b) IgA nephropathy
(c) Systemic lupus erythematosus
(d) Cryoglobulinaemia













26.
Consider the following statements with regard to oral aphthous ulcers:

1. They are superficial and painful
2. They may occur in women just prior to menstruation
3. Oral glucocorticoids may be needed for the treatment of severe, recurrent cases
4. They are pre-malignant and progress over few years to squamous cell carcinoma of
the oral cavity

Which of the above statements are correct?

(a) 1 and 4 only
(b) 2 and 3 only
(c) 1, 2 and 3 only
(d) 1, 2, 3 and 4
27.
Which one of the following statements about Barrett?s Oesophagus is NOT correct?

(a) It is a pre-malignant condition
(b) Normal squamous cells lining the lower oesophagus are replaced by columnar
cells
(c) Normal columnar cells lining the lower oesophagus are replaced by squamous
cells
(d) It is an adaptive response to chronic gastro-oesophageal reflux

28.
The most common cause of acute hepatitis outbreaks in India is:

(a) Hepatitis E
(b) Hepatitis C
(c) Hepatitis B
(d) Hepatitis A

29.
Spontaneous oesophageal perforation after a bout of forceful vomiting or retching is
characteristically seen in which one of the following conditions?

(a) Menetrier?s disease
(b) Boerhaave?s syndrome
(c) Achalasia of the oesophagus
(d) Barrett?s oesophagus
30.
Eradication of helicobacter pylori infection may prove beneficial in the following extra-
gastric disorders EXCEPT:

(a) Unexplained vitamin B12 deficiency
(b) Idiopathic thrombocytopenic purpura
(c) Iron deficiency anaemia without gastro-intestinal bleeding
(d) Acute glomerulonephritis

31.
What is the target blood Hemoglobin level when treating anaemia in Stage-4 chronic kidney
disease patient?
(a) 8 to 10 gm/dL
(b) 10 to 12 gm/dL
(c) 12 to 13 gm/dL
(d) 13 to 14 gm/dL
32.
Which one of the following complications of chronic kidney disease is observed in patients
with low parathyroid hormone levels?
(a) Adynamic bone disease
(b) Tumoral calcinosis
(c) Osteitis fibrosa cystic
(d) Calciphylaxis
33.
All of the following statements about adult polycystic kidney disease are true, EXCEPT:
(a) It is inherited as autosomal dominant trait
(b) It is usually associated with marked proteinuria
(c) It is usually associated with systematic hypertension, from young age
(d) It is associated with Berry aneurysm
34.
Consider the following statements in relation to an adult patient:

1. Oliguria is defined as passage of urine less than 300 ml per day
2. Anuria is said to exist when less than 50 ml urine is passed per day
3. Polyuria is defined as urine volume in excess of 3 litre per day

Which of the statements given above are correct?
(a) 1 and 3 only
(b) 2 and 3 only
(c) 1 and 2 only
(d) 1, 2 and 3

35.
White cell casts in urine examination are strongly suggestive of:
(a) Nephritis
(b) Pyelonephritis
(c) Renal stone disease
(d) Papillitis
36.
A 13-year old boy presents with hematuria, oliguria, edema and hypertension. He has history
of sore throat two weeks prior to presentation. Laboratory investigations are remarkable for
low C
3
and increased titres of ASO and antiDNase. Which one of the following statements is
NOT correct about management for this condition?

(a) Renal biopsy is rarely required for making diagnosis
(b) Antibiotic treatment is given for streptococcal infection
(c) Treatment is largely supportive
(d) Immunosuppressants are to be used for crescentic glomerulonephritis
37.
Which one of the following statements is NOT correct regarding the risk factors for
nephrolithiasis?

(a) Higher dietary calcium increases risk
(b) Higher animal protein intake increases risk
(c) Higher dietary potassium intake lowers risk
(d) Lower urine citrate increases the risk
38.
A 60-year old patient is admitted in emergency with seizure, aphasia and altered sensorium.
He has history of fever and headache for the preceding 3 days. The CSF examination is
unremarkable. What would be the probable diagnosis?
(a) Multiple sclerosis
(b) Tubercular meningitis
(c) Pyogenic (bacterial) meningitis
(d) Viral encephalitis
39.
Drug of choice for the treatment of Trigeminal Neuralgia is:
(a) Carbamazepine
(b) Aceclofenac
(c) Thiamine
(d) Prednisdone
40.
Constructional skills is a function of which lobe?
(a) Dominant temporal lobe
(b) Non dominant parietal lobe
(c) Frontal lobe
(d) Non dominant temporal lobe

41.
Vertical gaze palsy with convergence retraction nystagmus is seen in:
(a) Weber syndrome
(b) Millard Gubler syndrome
(c) Claude syndrome
(d) Parinaud syndrome
42.
A patient on looking forward was found to have his right eye deviated downwards and
outwards with pupil dilated. He is suffering from:
(a) Left 3
rd
nerve palsy
(b) Left 6
th
nerve palsy
(c) Right 4
th
nerve palsy
(d) Right 3
rd
nerve palsy
43.
All of the following are features of Cauda Equina syndrome EXCEPT:

(a) Low back pain
(b) Lower limb areflexia
(c) Loss of bladder function
(d) Extensor plantar response

44.
A 68-year old male presented in OPD with complaints of progressive small handwriting. On
examination, he had resting tremor, bradykinesia, rigidity and postural instability. This
patient is suffering most likely from:

(a) Parkinson?s disease
(b) Vascular dementia
(c) Alzheimer?s disease
(d) Frunto temporal dementia
45.
An elderly patient presents with abnormal gait. He has a wide base freezing gait with
imbalance, comprising short strides, shuffling along the floor and difficulty with starts and
turns. Heal-Shin test is normal. Which one of the following is most likely disorder?
(a) Cerebellar ataxia
(b) Sensory ataxia
(c) Frontal gait disorder
(d) Parkinson?s disease
46.
Cushing reflex, seen in conditions of raised intra cranial pressure includes all of the following
EXCEPT:
(a) Bradycardia
(b) Hypertension
(c) Irregular respiration
(d) Hypothermia

47.
A 35 year old man presents with history of low grade fever and headache for last five weeks.
Clinical examination is remarkable for signs of meningismus. He undergoes lumbar puncture
(LP) and a day later reports worsening of headache. Which one of the following features is
NOT consistent with diagnosis of ?Post-LP? headache?

(a) Post-LP headache usually begins within 48 hours
(b) Post-LP headache worsens in sitting position
(c) Post-LP headache is most severe upon waking up
(d) Post-LP headache may improve with caffeine intake
48.
Which one of the following investigations is NOT appropriate in a case of recurrent
thrombosis?
(a) Antiphospholipid antibodies
(b) Bcr-Abl assay
(c) Protein C and S assays
(d) Antithrombin level
49.
All of the following are causes of reactive thrombocytosis, EXCEPT:
(a) Hemolytic anemia
(b) Megaloblastic anemia
(c) Post splenectomy
(d) Chronic inflammatory disorders


50.
Cryoprecipitate cannot be used for treatment of which one of the following conditions?
(a) Von Willebrand disease
(b) Hypofibrinogenemia
(c) Hemophilia?B
(d) Hemphilia?A
51.
Pyruvate kinase deficiency results in deficiency of ATP production and a chronic haemolytic
anaemia. The disorder is inherited as an:

(a) Autosomal recessive trait
(b) X-linked recessive trait
(c) Autosomal dominant trait
(d) X-linked dominant trait
52.
Sickle cell disease may be associated with any of the following EXCEPT:

(a) Acute chest syndrome
(b) Aplastic crisis
(c) Splenic sequestration crisis
(d) Conn?s syndrome

53.
Fever with splenomegaly and lymphadenopathy can be seen in
(a) Infectious mononucleosis
(b) Chronic leukaemia
(c) Both infectious mononucleosis and chronic leukaemia
(d) Neither infectious mononucleosis nor chronic leukaemia
54.
Every individual carries four alpha gene alleles. Deletion of three alleles leads to
development of:
(a) Haemoglobin X disease
(b) Haemoglobin H disease
(c) Haemoglobin F disease
(d) Hydrops fetalis
55.
Which one of the following is NOT true regarding Chronic Myeloid Leukaemia (CML)?
(a) It is a clonal malignancy of haematopoetic stem
(b) Risk of developing CML is increased in monozygotic turns
(c) CML is defined by presence of BCR-ABL 1 fusion gene
(d) 10 year survival with TKI therapy is 85%









56.
Diagnostic criteria of multiple myeloma includes which of the following?

1. Increased malignant plasma cells in the bone marrow
2. Serum and/or urinary M protein
3. Skeletal lytic lesions
4. Skeletal blastic lesions

Select the correct answer using the code given below:
(a) 1, 2 and 3
(b) 1, 3 and 4
(c) 1, 2 and 4
(d) 2, 3 and 4
57.
The most predominant type of serum immunoglobulin involved in patients with plasma cell
disorder is:
(a) Ig G
(b) Ig A
(c) Ig D
(d) Light chains
58.
Which one of the following statements is NOT true regarding the thyroid function/disorder in
pregnancy?
(a) There is an increase in the metabolism of thyroxine by placenta
(b) Hyperemesis gravidarum may be associated with thyrotoxicosis in pregnancy
(c) Levothyroxine replacement therapy dose should be decreased by 30?50% early in
pregnancy
(d) Impaired cognitive impairment may be seen in the offspring of mother with
subclinical hypothyroidism

59.
Which one of the following is NOT the criterion for the diagnosis of Diabetes mellitus?
(a) Symptoms of diabetes and random plasma sugar concentration ? 200 mg/dL
(b) HbA1c ? 6.5 %
(c) 2-hours plasma glucose ? 200 mg/dL during an oral glucose tolerance test
(d) Fasting plasma glucose ? 110 mg/dL
60.
A young lady presents in outdoor clinic with complaints of menstrual irregularity, weight
gain, hair loss, tiredness and weakness. What will be the investigation of choice?
(a) FSH & LH level
(b) Estrogen level
(c) Free T
3
and free T
4
level
(d) Free T
4
and TSH level







61.
Consider the following statements about erectile dysfunction in diabetic males:

1. It affects 60 % of males
2. Its common cause is an underlying neuro-vascular pathology
3. It may be aggravated by beta-adrenergic agonist drugs
4. Endocrine disorders like hyperprolactinemia may cause it

Which of the statements given above are true?
(a) 1 and 2
(b) 2 and 3
(c) 2 and 4
(d) 3 and 4
62.
All of the following can cause Hyperprolactinemia, EXCEPT:
(a) Craniopharyngioma
(b) Hyperthyroidism
(c) Stress
(d) Chronic renal failure
63.
All of the following can be seen in a case of thyrotoxicosis, EXCEPT:
(a) Atrial fibrillation
(b) Periodic paralysis
(c) Deafness
(d) Osteoporosis
64.
Which one of the following hormones acts via the receptor tyrosine kinase?
(a) FSH
(b) TSH
(c) IGF-1
(d) PTH
65.
Consider the following disorders as constituent of Multiple Endocrine Neoplasia Type 2a
(MEN 2a):

1. Primary Hyperparathyroidism
2. Pituitary tumours
3. Medullary carcinoma of thyroid
4. Pheochromocytoma

Which of the above are correct?:

(a) 1, 2 and 4
(b) 1, 2 and 3
(c) 1, 3 and 4
(d) 2, 3 and 4




66.
Consider the following statements with regard to Graves? ophthalmopathy :

1. Proptosis is often asymmetric and can even appear to be unilateral
2. It is a clinical diagnosis
3. Worsening of symptoms upon glucocorticoid withdrawal is common
4. Radiation therapy is very effective in treatment

Which of the above statements are correct?

(a) 1 and 2 only
(b) 1, 2 and 3
(c) 2, 3 and 4
(d) 1 and 4 only
67.
All of the following are common aetiologies of delirium EXCEPT:
(a) Hyperthyroidism
(b) Hypothyroidism
(c) Hyperparathyroidism
(d) Hypoparathyroidism
68.
Key anthropometric measurements important for evaluating the degree of obesity are:

(a) Weight, height and waist circumference
(b) Weight, height and hip circumference
(c) Weight, height and mid-thigh circumference
(d) Weight, height and chest circumference
69.
What is the minimum BMI (Body Mass Index) recommended for considering adjuvant
pharmacologic treatment in obesity with comorbidity?
(a) 23 kg/m
2

(b) 25 kg/m
2

(c) 27 kg/m
2

(d) 30 kg/m
2

70.
A young medical student while working in HIV ward got pricked accidently on the finger by
a syringe needle which was used to draw blood from a patient. Patient?s infection status is not
known. Which one of the following therapeutic interventions will be the best interest of this
student?
(a) To start Dolultegravir immediately
(b) To wash wound with soap and water and start Emtricitabine
(c) To give Dolutegravir + Tenofovir + Emtricitabine
(d) To give Dolutegravir + Tenofovir + Emtricitabine + HBV Immunoglobulin
71.
Cryptococcal infection is acquired through:
(a) Inhalation route
(b) Bite of Culex mosquito
(c) Direct skin contact
(d) Ingestion of spores

72.
Which one of the following is NOT appropriate treatment for Echinococcus granulosus and
hydatid disease?
(a) Albendazole 400 mg twice in a day for 3 months
(b) Diethylcarbamazine 2 mg/kg thrice in a day for 12 days
(c) PAIR (percutaneous puncture, aspiration, injection of scolicide, reaspiration)
(d) Praziquantel (20 mg/kg twice daily for 14 days)
73.
Which one of the following infections is NOT amenable to post-exposure prophylaxis with
specific immunoglobulins?
(a) Hepatitis B
(b) HIV
(c) Tetanus
(d) Rabies
74.
Echinocandins are a class of:
(a) Antiviral agents
(b) Antiparasitic agents
(c) Antitubercular agents
(d) Antifungal agents
75.
Recommended first time drug for initial treatment of tuberculosis include all EXCEPT:
(a) Isoniazid
(b) Rifampin
(c) Ethambutol
(d) Ethionamide
76.
The most frequent adverse reaction of significance among people treated for drug-susceptible
tuberculosis is:
(a) Colitis
(b) Hepatitis
(c) Cystitis
(d) Enteritis
77.
All adult patients in whom anti-tubercular treatment is being started should undergo baseline
assessment of:
(a) Complete blood count
(b) Liver function test
(c) Lipid profile
(d) Pulmonary function test
78.
A 30 year old male presents with fever, headache, anorexia, nausea, vomiting and diarrhoea.
On examination tongue is coated with abdominal tenderness, soft splenomegaly, relative
bradycardia and rose spots rash. The most likely diagnosis is:

(a) Malaria fever
(b) Enteric fever
(c) Leptospirosis
(d) Dengue fever

79.
The extrapulmonary sites most commonly involved in tuberculosis are all EXCEPT:
(a) Lymph node TB
(b) Genitourinary TB
(c) Skeletal TB
(d) Skull TB
80.
Which of the following is NOT a clinical feature of Botulism?
(a) Impaired alertness
(b) Dysphagia
(c) Diplopia
(d) Ptosis
81.
Post exposure prophylaxis (PEP) for HIV should contain:
(a) 2 antiretroviral drugs administered for 4 weeks
(b) 2 antiretroviral drugs administered for 6 weeks
(c) 3 antiretroviral drugs administered for 4 weeks
(d) 3 antiretroviral drugs administered for 6 weeks
82.
Paralytic rabies is characterized by:
(a) Maculopapular rash
(b) Flaccid muscle weakness
(c) Spastic muscle weakness
(d) Hyperexcitability and facial weakness
83.
All of the following are seen in Marasmus EXCEPT:

(a) Reduced triceps skinfold
(b) Reduced mid arm circumference
(c) Protein wasting
(d) Decreased body mass index
84.
A 20 year young lady presents with high grade fever for five days and palpable purpura over
extremities. She is found to be confused, with presence of neck stiffness. Which of the
following is the most likely diagnosis?
(a) Disseminated intravascular coagulation
(b) Acute meningococcemia
(c) Antiphospholipid antibody syndrome
(d) Thrombotic thrombocytopenic purpura
85.
Fibroblast Growth Factor 23 (FGF 23) is increased in which one of the following conditions?
(a) Osteomalacia
(b) Osteoporosis
(c) Paget?s disease
(d) Renal Osteodystrophy





86.
Which of the following crystals are deposited in a case of pseudogout?
(a) Monosodium urate
(b) Calcium phosphate
(c) Cholestrol crystals
(d) Calcium pyrophosphate dehydrate (CPPD)
87.
Which of the following features are associated with poor prognosis in Diffuse Cutaneous
Systemic Sclerosis?

1. Older age
2. A high gas transfer factor for carbon monoxide (TLCO)
3. Proteinuria
4. Diffuse skin disease

Select the correct answer using the code given below:
(a) 1, 2 and 4
(b) 3 and 4 only
(c) 1, 3 and 4
(d) 1, 2 and 3

88.
Which of the following statements regarding pegloticase are correct?

1. It is enzyme conjugate
2. It is useful in chronic Gout management
3. The main adverse effects are infusion reactions and development of antibodies

Select the correct answer using the code given below:
(a) 1 and 2 only
(b) 2 and 3 only
(c) 1 and 3 only
(d) 1, 2 and 3
89.
The most common cause of sensorineural hearing loss in adults is:
(a) Otosclerosis
(b) Meniere?s disease
(c) Presbycusis
(d) Meningitis
90.
A 50-year old male presents with dizziness. Which one of the following clinical findings does
NOT favour a central cause?
(a) Poor saccades
(b) Gaze evoked nystagmus
(c) Inhibition by visual fixation
(d) Presence of diplopia, dysarthria




91.
A young primigravida with 11 weeks pregnancy presents with sudden onset pain and
swelling of left lower limb. Homan?s sign is positive. Duplex ultrasonogram shows non-
compressibility of veins in the affected region, with absent flow. The drug of choice for
treatment is:
(a) Enoxaparin
(b) Amoxycillin?Clavulanic acid
(c) Warfarin
(d) Limb elevation only
92.
All of the following are clinical features of hypovolaemic shock EXCEPT:
(a) Cold clammy skin
(b) Bradycardia
(c) Hypotension
(d) Oliguria
93.
Which one of the following is most appropriate to describe classic heat stroke?
(a) Patients core body temperature rises above 44?C with headache, nausea and loss of
consciousness
(b) Patients core body temperature rises above 40 ?C with muscle tremors, confusion,
loss of consciousness and loss of sweating
(c) Patients core body temperature rises above 38 ?C with increased sweating,
tachypnoea, tachycardia and loss of consciousness
(d) Patients core body temperature is above 40 ?C with disorientation, increased
sweating, peripheral vasodilation, tachycardia and loss of consciousness
94.
Which of the following is NOT a priority in the initial management of shock from venomous
snake?

(a) Aggressive volume resuscitation
(b) Vasopressor support
(c) Anti-venom administration
(d) All of these are undertaken simultaneously

95.
Which one of the following is NOT an appropriate criterion for diagnosis of Systemic
Inflammatory Response Syndrome (SIRS)?
(a) Respiratory rate > 20/mm
(b) Heart rate > 90/min
(c) Total leucocyte count > 16 ? 10
9
per Litre
(d) Temperature > 38
0
C








96.
All of the following predispose to Acute Respiratory Distress Syndrome EXCEPT:
(a) Acute Pancreatitis
(b) Toxic gas inhalation
(c) Aspiration of gastric contents
(d) Severe Bronchial asthma
97.
An 8 year old child has consumed a few tablets of a drug that were being taken by his mother
on a regular basis. Following this the child develops gastric necrosis, acidosis, shock and
hepatic necrosis. Which is the most likely drug?
(a) Multivitamins
(b) Calcium carbonate
(c) Folic acid
(d) Iron
98.
A newborn weighing 1.5 kg at birth presents at 6 hours of age with lethargy, weak and high
pitched cry and difficulty in feeding. What is the next step in management?

(a) Estimation of blood sugar
(b) Estimation of serum calcium
(c) Supplementary oxygen
(d) Bedside electrocardiogram

99.
In a child with acute organophosphorus poisoning, which one of the following drugs will act
as anti-sialogogue and peripheral parasympatholytic agent?

(a) Carbamates
(b) Atropine
(c) Benzodiazepine
(d) Pralidoxime
100.
A 6-year old child needs to undergo a central Catheter placement for total parental nutrition.
The procedure is likely to evoke pain and anxiety. Which of the following is the most
appropriate drug for sedation and analgesia in this child?

(a) Chloral hydrate
(b) Midozolam
(c) Propofol
(d) Ketamine
101.
Which one of the following is the drug of choice in a child presenting with supraventricular
tachycardia?

(a) Atropine
(b) Lidocaine
(c) Adenosine
(d) Nalaxone


102.
Which one of the following drugs is used in cardiopulmonary resuscitation for pulseless
ventricular fibrillation?

(a) Atropine
(b) Adenosine
(c) Lidocaine
(d) Amiodarone
103.
Which one of the following is a sign of good attachment during breastfeeding?
(a) The baby?s nose is at the level of the nipple
(b) The baby?s chin touches the breast
(c) Most of the nipple is in the mouth and the lower areola is visible
(d) The baby?s lower lip is inverted
104.
Which one of the following signs is NOT included as an indicator of a vigorous baby
immediately after delivery of an infant born through meconium strained liquor?
(a) Pink colour
(b) Strong respiratory efforts
(c) Good muscle tone
(d) Active movements of the baby
105.
All are true for a Cephalohematoma in a newborn EXCEPT:
(a) It is located over the parietal bones in the subcutaneous plane
(b) It may be associated with hyperbilirubinemia
(c) It increases in size in the first 12?24 hours
(d) It can take 3?6 weeks to resolve
106.
Which of the following sick neonates can be started on ?Minimal Enteral Feeding?, if
hemodynamically stable?
(a) Septic neonate with sclerema
(b) Neonate with necrotizing enterocolitis (NEC)
(c) Neonate on mechanical ventilation
(d) Symptomatic hypoglycaemia
107.
Which one of the following drugs is contraindicated in a mother who is breastfeeding her
infant?
(a) Bromocriptine
(b) Chlorpromazine
(c) Acyclovir
(d) Morphine
108.
A term neonate presents with tachypnea on day 1 after birth. The chest X-ray reveals
hyperexpanded lung fields, prominent vascular markings, and prominent interlobar fissure.
Which of the following is the most probable diagnosis?
(a) Congenital lobar emphysema
(b) Diaphragmatic hernia
(c) Transient tachypnea of the newborn
(d) Tracheoesophageal fistula

109.
A pre-school boy can go upstairs on alternate feet and has also started copying a circle. How
many blocks should he be able to build a tower with, without toppling over?

(a) 5
(b) 7
(c) 9
(d) More than 10

110.
Which one of the following drugs is the antidote for benzodiazepine poisoning?

(a) Flumazenil
(b) Atropine
(c) Naloxone
(d) Pralidoxime aldoxime methiodide

111.
A 3-year old child develops severe respiratory distress after inhalation of fumes arising from
burning plastic. The child is started on supportive ventilation. Which one of the following
antidotes needs to be given?
(a) Hydroxycobalamin
(b) Amylnitrate
(c) Sodium nitrite
(d) N-acetyl cysteine
112.
A child has an incurving of the little finger. What is this condition known as?
(a) Camptodactyly
(b) Brachydactyly
(c) Partial cutaneous syndactyly
(d) Clinodactyly
113.
A 4-week old infant presents with repeated episodes of non-bilious vomiting since 7 days.
The infant seems active and hungry after the vomiting. Examination reveals an olive-shaped
mass in the epigartrium. Which metabolic abnormality is expected in this infant?
(a) Hyperchloremia
(b) Metabolic alkalosis
(c) Hyperkalemia
(d) Hyponatramia
114.
Which one of the following is the correct dose and route of administration of hepatitis B
vaccine in children?

(a) 0.5 mL dose with 10 microgram of antigen subcutaneously
(b) 0.5 mL dose with 10 microgram of antigen intramuscularly
(c) 1 mL dose with 5 microgram of antigen subcutaneously
(d) 1 mL dose with 5 microgram of antigen intramuscularly


115.
Which one of the following statements is correct for approved acellular pertussis vaccine?
(a) It contains at least 3 IU of inactivated pertussis toxin
(b) It contains at least 3 pathogenic pertussis antigens
(c) It?s efficacy is superior to the whole cell pertussis vaccine
(d) Presence of nonfimbrial protein is a mandatory component
116.
Which of the following is recommended in the follow up health visits of a child with Down
syndrome?
(a) Growth assessment twice a year in the first year and annually till 5 years
(b) Annual auditory screening from 1 to 5 years
(c) Screening for refractory error if visual impairment is suspected
(d) Annual thyroid function test from birth to 2 years
117.
A 12 year old child presents with delayed puberty and delayed bone age. His growth records
reveal that he was growing normally till 1 year of age, after which both his height and weight
fell to below the third centile for age. Subsequently, he continued to grow below the third
centile with normal height velocity. What is the most likely diagnosis?
(a) Familial short stature
(b) Constitutional growth delay
(c) Growth hormone deficiency
(d) Skeletal dysplasia
118.
According to the ?Integrated Management of Neonatal and Childhood Illnesses? (IMNCI)
classification of young infants upto 2 months, which one of the following signs is NOT
included in ?Possible Serious Bacterial Infection?
(a) Fast breathing (50 breaths or more per minute)
(b) Severe chest indrawing
(c) Axillary temperature less than 36.5
0
C
(d) Movement only when stimulated or no movement at all
119.
Neonatal tetanus elimination in India is defined as less than one case per:

(a) 100 live births per year
(b) 1000 live births per year
(c) 10000 live births per year
(d) 100000 live births per year
120.
Which one of the following facilities has been established at district hospitals in India to
strengthen the care of low birth weight babies and sick newborns?
(a) Special Newborn Care Units (SNCU)
(b) Newborn Treatment Units (NTU)
(c) Newborn Stabilization Units (NSU)
(d) Shishu Suraksha Units (SSU)



FirstRanker.com - FirstRanker's Choice

This post was last modified on 02 August 2021